Đến nội dung

Ban Biên Tập nội dung

Có 70 mục bởi Ban Biên Tập (Tìm giới hạn từ 20-04-2020)



Sắp theo                Sắp xếp  

#617579 Ngày 29 tháng 02

Đã gửi bởi Ban Biên Tập on 29-02-2016 - 00:22 trong Toán học lý thú

Ngày 29 tháng 2 chỉ xảy ra vào những năm nhuận. Ngày 29 tháng 2 là ngày thứ 60 trong một năm nhuận của lịch Gregory. Theo định nghĩa, năm nào có ngày này là một năm nhuận. Nó chỉ xuất hiện mỗi bốn năm một lần như 1996, 2000, 2004, 2008, 2012 và 2016.

 

Liên quan đến vấn đề này, một câu hỏi khác được đặt ra: Tại sao tháng 2 lại có 28 ngày(năm nhuận là 29 ngày) trong khi các tháng khác trong năm đều có 30 hoặc 31 ngày?

 

Vì những năm 46 trước Công nguyên, thống soái La Mã Julius César khi định ra lịch dương, quy định ban đầu là mỗi năm có 12 tháng, tháng lẻ là tháng đủ, có 31 ngày; Tháng chẵn là tháng thiếu, có 30 ngày. Tháng 2 là tháng chẵn lẽ ra cũng phải có 30 ngày. Nhưng nếu tính như vậy thì một năm không phải có 365 ngày mà là 366 ngày. Do đó phải tìm cách bớt đi một ngày trong một năm.

 

Vậy thì bớt đi một ngày trong tháng nào?

 

Lúc đó, theo tập tục của La Mã, rất nhiều phạm nhân đã bị xử tử hình, đều bị chấp hành hình phạt vào tháng 2, cho nên mọi người cho rằng tháng đó là tháng không may mắn. Trong một năm đã phải bớt đi một ngày, vậy thì bớt đi một ngày trong tháng 2 , làm cho tháng không may mắn này bớt đi một ngày là tốt hơn. Do đó tháng 2 còn lại 29 ngày, đó chính là lịch Julius.

 

Sau này, khi Augustus kế tục Julius César lên làm Hoàng đế La Mã. Augustus đã phát hiện ra Julius César sinh vào tháng 7, theo lịch Julius thì tháng 7 là tháng đủ, có 31 ngày, Augustus sinh vào tháng 8, tháng 8 lại luôn là tháng thiếu, chỉ có 30 ngày. Để biểu thị sự tôn nghiêm như Julius César, Augustus đã quyết định sửa tháng 8 thành 31 ngày. Đồng thời cũng sửa lại các tháng khác của nửa năm sau. Tháng 9 và tháng 11 ban đầu là tháng đủ thì sửa thành tháng thiếu. Tháng 10 và tháng 12 ban đầu là tháng thiếu sửa thành tháng đủ. Như vậy lại nhiều thêm một ngày, làm thế nào đây? Lại lấy bớt đi một ngày trong tháng 2 không may mắn nữa, thế là tháng 2 chỉ còn 28 ngày.

 

Hơn 2.000 năm trở lại đây, sở dĩ mọi người vẫn tiếp tục dùng cái quy định không hợp lý này chỉ vì nó là một thói quen. Những người nghiên cứu lịch sử trên thế giới đã đưa ra rất nhiều phương án cải tiến cách làm lịch, họ muốn làm cho lịch được hợp lý hơn.

 

Trở lại Lịch Gregory. Như chúng ta biết, Lịch Gregory là một bộ lịch mới do Giáo hoàng Grêgôriô XIII đưa ra vào năm 1582. Lịch Gregory chia thành 12 tháng với 365 ngày, cứ 4 năm thì thêm một ngày vào cuối tháng 2 tạo thành năm nhuận. Vì vậy theo lịch Julius thì một năm có 365,25 ngày. Nhưng độ dài của năm mặt trời là 365,242216 ngày cho nên lịch Julius dài hơn khoảng 0,0078 ngày trong một năm, tức là khoảng 11 phút 14 giây.

 

Để bù vào sự khác biệt này thì cứ 400 năm ta sẽ bỏ bớt đi 3 ngày năm nhuận. Cho đến năm 1582, thì sự sai biệt đã lên đến 10 ngày. Giáo Hoàng Gregory XIII quyết định bỏ 10 ngày trong tháng 10 năm đó để cho lịch và mùa màng tương ứng trở lại. Sau ngày 4 tháng 10 năm 1582 là ngày 15 tháng 10. Và để tránh sai biệt, lịch lấy năm nhuận là năm có số thứ tự chia hết cho 4 (như năm 1964, 1980, 2004, ...) và các năm tận cùng bằng 00 phải chia hết cho 400 mới là năm nhuận (năm 2000 chia hết cho 4 và 400 nên là năm nhuận, những năm 1700 1800 và 1900 chia hết cho 4 nhưng không chia hết cho 400 nên không phải là năm nhuận, ...). Lịch đã sửa mang tên lịch Gregory và được áp dụng cho đến bây giờ.

 

Ngày 29 tháng 2 ngày nay ra sao?

 

Nói về ngày này thì trước tiên : Đứa trẻ nào chào đời vào đúng ngày 29 tháng 2 mà muốn mừng sinh nhật đúng ngày,thì phải đợi đến 4 năm nữa. Lý do tháng này năm nay có ngày nhuần, có đến 29 ngày chứ không phải 28 ngày như của mọi tháng Hai khác.

 

Tại một số quốc gia, để tiện việc, chính phủ sở tại có thể cho đứa trẻ sinh ra trong ngày 29 tháng Hai được kê trong giấy khai sinh là sinh ngày 28 tháng Hai hay sinh ngày 1 tháng Ba. (Theo tập tục của Pháp. Tuỳ cha mẹ chọn ngày sinh 28 -29- hoặc 1tháng 3).

 

Ngoài ra ngày 29 tháng 2 còn là Ngày phái đẹp tỏ tình. Cứ 4 năm mới có một ngày 29/2, ngày này ở châu Âu được coi là “Ngày phụ nữ tỏ tình” - tức là phái đẹp chủ động cầu hôn với giới mày râu.

 

Đây là phong tục lâu đời ở Anh, người Scottland đã thông qua đạo luật lấy ngày 29/2 là “Ngày quyền lợi phụ nữ”. Hồi đó, Nữ hoàng Margarit đã tuyên bố: Trong ngày này phụ nữ có thể cầu hôn với đàn ông và tiến hành trừng phạt những gã đàn ông thích thả dê nhưng lại chối bỏ trách nhiệm.

 

Từ thế kỷ XVII, phong tục này đã lan ra khắp châu Âu. Ngày 29/2/2004 đã có hơn 7.000 phụ nữ Anh chủ động cầu hôn, trong đó có cô MC một đài truyền hình đã cầu hôn bạn trai ngay trên sóng truyền hình và đã thành công.

 

Hiện nay, những vị mày râu nào từ chối lời cầu hôn của bạn gái trong ngày này sẽ phải nộp 1 Bảng tiền “thế chấp” hoặc phải tặng một tấm áo lụa cho người con gái bị tổn thương, nhưng tình huống này rất hiếm khi xảy ra.

 

Theo thống kê, trong ngày 29/2 lần trước có tới 92% đàn ông được ngỏ lời đã vui mừng chấp nhận tình cảm của bạn gái, 4% lúc đầu không đồng ý vì bất ngờ, nhưng sau khi suy nghĩ đã vui vẻ chấp thuận. Đối với nhiều phụ nữ Anh, 29/2 là một ngày có ý nghĩa quyết định trong cuộc đời.

 

Trong lịch sử đã có rất nhiều nhân vật nổi tiếng đã nên duyên nhờ chủ động cầu hôn trong ngày này. Tiêu biểu là nữ minh tinh điện ảnh người Hungary G. Garbo. Bà từng tuyên bố: Cả 9 người chồng trong cuộc đời đều do bà chủ động cầu hôn và bà giải thích “phụ nữ phải hướng dẫn ý thích của đàn ông”.

a




#598136 Thứ Sáu ngày 13 dưới góc nhìn toán học và văn hóa

Đã gửi bởi Ban Biên Tập on 13-11-2015 - 16:54 trong Toán học lý thú

BÀI TẬP

 

Hãy chứng minh:

 

1) Một năm bất kỳ có ít nhất một thứ sáu ngày 13 và nhiều nhất ba thứ sáu ngày 13.

 

2) Một năm có ba thứ sáu ngày 13 khi và chỉ khi ngày đầu năm là thứ năm (đối với năm không nhuận) hoặc chủ nhật (đối với năm nhuận).

 

3) Khoảng cách giữa hai ngày thứ sáu 13 gần nhất chỉ có thể là 27, 90, 181, 244, 272, 335 hoặc 426 ngày.




#598134 Thứ Sáu ngày 13 dưới góc nhìn toán học và văn hóa

Đã gửi bởi Ban Biên Tập on 13-11-2015 - 16:53 trong Toán học lý thú

Ở một số nước phương Tây, người ta cho rằng thứ sáu ngày 13 là ngày rủi ro. Tuy nhiên, báo Le Figaro (Pháp) số ra ngày 11-3-2009 cho biết số người mua lô tô tại Pháp vào thứ sáu ngày 13 cao gấp 3 lần so với những ngày khác.

 

 

Fri13.jpg

 

 

Vậy thứ sáu ngày 13 có đặc điểm gì về mặt toán học và văn hóa? Nó là ngày tốt hay xấu?
Bằng lý thuyết đồng dư, toán học chứng minh được một năm bất kỳ có ít nhất một thứ sáu ngày 13 và nhiều nhất ba thứ sáu ngày 13. Hơn nữa, một năm có ba thứ sáu ngày 13 khi và chỉ khi ngày đầu năm là thứ năm (đối với năm không nhuận) hoặc chủ nhật (đối với năm nhuận). Đó là trường hợp của năm 2009: có ba thứ sáu ngày 13 rơi vào tháng hai, tháng ba và tháng mười một. Sự kiện này đã xảy ra vào năm 1998 và sẽ lặp lại vào các năm 2015, 2026.

 

Năm 2010 và 2011 chỉ có một thứ sáu ngày 13 mỗi năm. Năm 2012 có ba thứ sáu ngày 13 rơi vào tháng giêng, tháng tư và tháng bảy. Bộ ba “giêng, tư, bảy” này ít gặp hơn so với bộ ba “hai, ba, mười một”. Năm 2013 có hai thứ sáu ngày 13 rơi vào tháng 9 và tháng 12. Tổng cộng có 21 thứ sáu ngày 13 từ 2009 - 2019.

 

Cũng bằng toán học, ta tính được khoảng cách giữa hai ngày thứ sáu 13 gần nhất chỉ có thể là 27, 90, 181, 244, 272, 335 hoặc 426 ngày. Như vậy, hai thứ sáu ngày 13 gần nhất có thể cách nhau hơn một năm. Đó chính là trường hợp 13-8-1999 và 13-10-2000.

 

Theo Kinh Thánh, Chúa Jésus bị đóng đinh trên thập tự giá vào thứ sáu. Hơn nữa, bữa ăn cuối cùng của Chúa với các môn đồ có đúng 13 người. Việc này thường được xem là nguồn gốc việc kiêng sợ thứ sáu ngày 13.

 

 

 

 

lastsupperdavinci.jpg

Kiệt tác Bữa tiệc cuối cùng của Léonardo da Vinci

 

 

Ở Ý, số 17 được gắn với sự rủi ro chứ không phải số 13. Còn ở Trung Quốc, con số này là 4 vì được phát âm gần giống với “tử” nghĩa là chết. Ở châu Mỹ Latin, ngày kiêng cữ lại là thứ ba 13.

 

 

Về mặt thống kê, hiện chưa có dữ liệu đáng tin cậy nào để gán cho thứ sáu ngày 13 với “may mắn” hay “rủi ro” theo một nghĩa nào đó. Chẳng hạn, xác suất trúng lô tô ở Pháp vào thứ sáu ngày 13 cũng giống với những ngày khác và xấp xỉ với 1/14.000.000. Xác suất nhỏ bé này không có nghĩa là bạn không thể trúng lô tô và không hề ngăn cản người chơi lô tô nuôi hi vọng!

 

VMF




#465220 Chúc mừng ngày 20/11/2013

Đã gửi bởi Ban Biên Tập on 19-11-2013 - 12:08 trong Góc giao lưu

Topic này dùng để các mem gửi lời chúc đến các thầy cô nhân ngày 20/11


a



#388877 Brachistochrone – Đường đoản thời của John Bernoulli

Đã gửi bởi Ban Biên Tập on 21-01-2013 - 20:51 trong Toán học lý thú

Rất có thể tiêu đề bài viết này đã làm bạn cảm thấy có đôi chút sợ hãi. Ấy vậy mà tôi dám cá rằng các bạn sắp được xem sau đây một trong những chứng minh toán học tuyệt diệu và đặc biệt nhất… Một trong những nguyên lý đẹp nhất của cơ học cổ điển là: Nguyên lý tác dụng tối thiểu. Mà nếu nói một cách nôm na là ” tự nhiên luôn thực hiện mọi việc một cách hết sức tiết kiệm và dè sẻn“. Nói riêng khi ta xét đến hành trình của một tia sáng, nó luôn luôn chọn con đường nào có thời gian đi ngắn nhất. Bây giờ tôi xin được giới thiệu một bài toán cũng tìm con đường đi có thời gian ngắn nhất, chỉ khác một điều: đó không phải là một bài toán quang học nữa, mà lại thuần túy là một bài toán cơ học!

Vào tháng 6 năm 1696, John Bernouilli gửi một lời thách thức đến cho toàn giới Toán học thời bấy giờ (chủ yếu là gửi đến ông anh trai James Bernouilli ) bằng bài toán sau đây:

“Tìm đường cong nối 2 điểm A và B ( trong đó B nằm thấp hơn A và cả hai không cùng nằm trên đường thẳng đứng) sao cho một chất điểm chuyển động không ma sát dưới tác động của trọng lực khởi đầu từ A dọc theo đường cong đó sẽ đến B sớm nhất?”.

Nói cho dễ hiểu là bạn phải làm cái máng trượt có hình thù thế nào để hòn bi lăn đến đích sớm nhất.
Hình đã gửi
Lần này chắc các bạn không nghĩ quỹ đạo cần tìm là đoạn thẳng nối 2 điểm A, B nữa chứ ? Thật ra thì bài toán này không phải là mới. Trong một cuốn sách của mình xuất bản 1638, Galileo cũng đã đề cập đến bài toán này và chứng minh được rằng quỹ đạo là cung tròn thì nhanh hơn quỹ đạo thẳng. Tuy vậy sự lựa chọn đường đi là cung tròn của ông không phải là lời giải đúng.
Hình đã gửi
Về phần mình, John Bernouilli đã đặt tên cho đường cong ngắn nhất đó là ‘Brachistochrone’, mà ông đã nối với nhau hai từ Hi Lạp: ‘Brachistos’ nghĩa là ngắn nhất và ‘chronos’ nghĩa là thời gian. Một cách ‘Việt Nam’ chúng ta có thể gọi nó là ‘đường đoản thời’. Như ta đã biết: trong toán học một hàm số có đồ thị là một đường cong nào đó, và ngược lại nói chung mỗi đường cong sẽ có thể là đồ thị của một hàm số nào đó. Thay vì tìm đường cong, chúng ta sẽ tìm hàm số nhận nó làm đồ thị. Đầu tiên chúng ta đặt đường cong nối từ A đến B vào một hệ tọa độ nhận A làm gốc với trục tung hướng thẳng đứng xuống dưới như sau:
Hình đã gửi
Vì chất điểm chuyển động dưới tác dụng của trọng lực nên ta ‘dễ dàng’ xác định được vận tốc của chất điểm tại điểm C có tung độ $y$ nào đó sẽ là: $v=\sqrt{2gy}$. Xin các bạn yên tâm là tôi sẽ không bắt chước theo các giảng viên đại học bây giờ với các điệp khúc: ” Ồ! dễ thấy rằng… hay không mấy khó khăn ta sẽ…”. Thật ra nhờ đi dạy mà tôi biết rằng trong các trường hợp mà Thầy giáo còn ’khó thấy’ thì dễ dàng nhất là cứ nói: “dễ thấy rằng…” Ngụ ý: chỉ có khờ mới không thấy, mà cuộc đời đâu có ai chịu nhận mình khờ …Trở lại vấn đề, vận tốc tại điểm C có tung độ $y$ sẽ được tìm ra bằng định luật bảo toàn cơ năng (chú ý rằng giả thiết bài toán đã bỏ qua ma sát nên cơ năng bảo toàn):

Đầu tiên để cho đơn giản ta sẽ chọn gốc thế năng tại điểm C. Vì tại A vật không có vận tốc đầu nên cơ năng sẽ chỉ gồm thế năng tại đó: $E=mgy$ (ở đây thế năng sẽ được tính theo độ cao tính từ điểm A xuống gốc thế năng C (tưởng tượng như là mặt đất), mà độ cao này lại chính là $y$. Mặt khác cơ năng tại C chỉ bao gồm động năng ( C là gốc nên thế năng bằng 0): $E=\frac{1}{2}mv^2$. Do cơ năng bảo toàn nên ta có:
$$\frac{1}{2}mv^2 = mgy \Rightarrow v^2 = 2gy \Rightarrow v = \sqrt{2gy}$$
Vận tốc này không hề phụ thuộc vào quỹ đạo của đường cong mà chỉ phụ thuộc vào tung độ y của điểm C đang xét. Và đến đây là điểm quan trọng nhất: Chúng ta chia mặt phẳng chứa đường đi của chất điểm thành vô số những dải lớp mỏng nằm ngang.
Hình đã gửi
Chất điểm sẽ lần lượt đi qua hết lớp này đến lớp khác. Và như trên đã nói vận tốc của nó không phụ thuộc vào quỹ đạo đường đi mà chỉ phụ thuộc vào dải lớp mà nó đi qua ( phụ thuộc vào tung độ y)…Các bạn đã thấy tình huống này ở đâu đó chưa? Đúng vậy! Ánh sáng cũng đã gặp tình huống này khi nó đi từ môi trường không khí vào môi trường nước, hai môi trường mà ở trong đó nó chuyển động với vận tốc khác nhau. Và bởi vì ánh sáng luôn đi theo con đường nhanh nhất nên chúng ta sẽ khôn ngoan đi theo ánh sáng, tức là tuân theo định luật khúc xạ. Bài toán cơ học lúc đầu đã bị chuyển thành một bài toán quang học mất rồi! Thật là một ý tưởng tuyệt diệu.

Trên cơ sở đó chúng ta sẽ nhìn bài toán ban đầu trong một bối cảnh khác: ở đó, có một tia sáng muốn đi từ A đến B. Nó phải đi qua một môi trường biến đổi liên tục mà vận tốc của nó thay đổi theo công thức $v=\sqrt{2gy}$. Vậy nó sẽ đi theo đường nào? Và tất nhiên con đường mà ánh sáng sẽ đi chính là con đường ta cần tìm. Đến đây, dù rằng vẫn còn nhiều việc phải làm nhưng bài toán mới này đã đơn giản hơn bài toán ban đầu biết dường nào rồi!

Gọi $v, v', v'', v''', ...$ lần lượt là vận tốc của ánh sáng trong từng lớp liên tiếp, và $\alpha, \alpha ', \alpha '', \alpha ''', ...$ là các góc tới tương ứng của tia sáng khi đến gặp các mặt phân cách (góc tạo bởi tia sáng với trục thẳng đứng). Theo định luật khúc xạ ánh sáng ta có:
$$\frac{sin\alpha}{v} =\frac{sin\alpha '}{v'} = \frac{sin\alpha''}{v''} = \frac{sin\alpha'''}{v'''} = ...$$
Và bởi vì $v$ nhận các giá trị liên tục theo độ sâu (cũng có nghĩa là các dải lớp môi trường của ta có thể cho mỏng đi một cách tùy ý) ta sẽ có: $\frac{sin\alpha}{v} = const \textbf{ (*)}$ dọc theo đường đi của ánh sáng.

Tiếp theo sẽ là một điểm khá tinh tế (dù không khó khăn để tưởng tượng). Đối với các lớp chúng ta đang xét, chẳng hạn lớp trên cùng có vận tốc $v$, mình sẽ cho bề dày của lớp này tiến dần về 0:

Hình đã gửi
Có nghĩa là cho điểm $P$ tiến dần về điểm $R$, lúc này đoạn $PR$ sẽ tiến dần tới tiếp tuyến của đường cong. Như vậy góc $\alpha$ thực chất sẽ là góc tạo bởi tiếp tuyến của đường cong tại $R$ và trục thẳng đứng.
Hình đã gửi
Ta gọi $\beta$ là góc tạo bởi tiếp tuyến tại $R$ và trục nằm ngang. Mọi người học lớp 7 rồi chắc là còn nhớ $\tan\beta$ chính là hệ số góc của tiếp tuyến của đường cong tại $R$. Mặt khác, nếu bạn đã học lớp 11 rồi thì sẽ biết luôn rằng hệ số góc của tiếp tuyến thì bằng với đạo hàm của hàm số tại đó, tức là: $\tan\beta = y'$. Mặt khác, hiển nhiên là:
$$\alpha + \beta = 90^o \Rightarrow \sin \alpha = \cos \beta = \frac{1}{\sqrt{1+y'^2}}$$.
Cái này được suy ra từ công thức lượng giác cơ bản: $1 + \tan^2\beta = \frac{1}{\cos^2\beta}$. Kết hợp với $(*)$ và và nhớ lại rằng $v = \sqrt{2gy}$ ta được:
$$const = \frac{\sin \alpha}{v} = \frac{1}{\sqrt{1+y'^2}\sqrt{2gy}} \Rightarrow y(1+y'^2) = const = c$$
Cuối cùng thì ta cũng đi đến được phương trình này (là phương trình mà hàm số $y$ của chúng ta phải thõa mãn):
$$y(1+y'^2) = c \textbf{ (**)}$$
Bấy lâu nay mọi người vẫn thường nói: “Toán học là ông hoàng mà cũng là người đầy tớ của khoa học”. Các bạn có biết vì sao không? Là ông hoàng thì xin không giải thích thêm, còn làm “đầy tớ” thì đây là một minh chứng rõ nét. Vật lý chỉ cần dựa vào lý thuyết của mình rút ra phương trình mà đại lượng nào đó phải thõa mãn. Công việc còn lại giao cho toán học xử lý, nó sẽ giải và tìm xem hàm số ấy là gì?

Tất nhiên phương trình $(**)$ là một phương trình vi phân và ẩn của nó làm một hàm số (hàm số chúng ta cần tìm). Nhiều bạn học sinh vẫn chưa biết giải thì cũng đừng bận tâm, vì nói cho cùng thì những phương trình như vậy đều đã có cách giải sẵn. Và thậm chí bạn có thể tìm ra nghiệm của nó chỉ với một phần mềm tính toán mạnh (Maple chẳng hạn). Hàm số tìm được có đồ thị là một đường cong với tên gọi: Cycloid. Tôi xin phép chỉ mô ta đường cong này một chút mà không viết ra đây công thức hàm số chúng ta tìm được. Nó cũng không hẳn phức tạp, chỉ là nó được viết dưới dạng tham số, mà tôi không muốn bài viết của mình dài thêm nữa.

Cùng tìm hiểu một chút về đường cong Cycloid, bạn có thể dễ dàng tưởng tượng ra hình ảnh của nó. Thật vậy, hãy gắn một điểm sáng vào một cái bánh xe đạp rồi lăn nó đi trong bóng tối. Quỹ đạo của điểm sáng đó cũng chính là hình ảnh của đường cong Cycloid.
Hình đã gửi
Và bây giờ là lúc ta tận hưởng thành quả: Đây là tính toán mô phỏng chuyển động của chất điểm theo những quỹ đạo khác nhau. Các bạn hãy xem:
Hình đã gửi
Các bạn chắc biết đâu là đường Cycloid của mình rồi chứ? Lời cuối cùng xin cảm ơn John Bernouilli với chứng minh tuyệt diệu này. Tôi sẽ không quá cường điệu khi nói rằng việc tìm ra chứng minh đó là một nghệ thuật! Chúng ta đã trải qua hai ’ngữ cảnh’ rất khác nhau: Đầu tiên là cơ học, rồi thoắt một cái nó biến thành bài toán quang học. Và quả thật các bạn đã thấy ở đâu một sự liên hệ độc đáo và thú vị đến như vậy hay chưa?

Theo Ngô Minh Đức



#373185 Hình học Tĩnh và động

Đã gửi bởi Ban Biên Tập on 27-11-2012 - 22:28 trong Tài liệu, chuyên đề, phương pháp về Hình học

HÌNH HỌC TĨNH VÀ ĐỘNG

Lê Bá Khánh Trình (Đại học KHTN, ĐHQG Tp. Hồ Chí Minh)

1. HÌNH HỌC TĨNH HAY ĐỘNG

Trong bài này, tôi muốn trình bày một đôi điều riêng tư về môn hình học phổ thông (hay còn được gọi là hình học sơ cấp) dưới hai cách nhìn có phần nào khác biệt nhau. Trước hết, thông dụng hơn cả là cách nhìn của một người quan tâm đến việc giải các bài toán hình học. Cách nhìn này thường yêu cầu xem xét, phân loại các bài toán khác nhau, trình bày kinh nghiệm giải quyết chúng và tìm ra các mối liên quan giữa chúng với các bài toán đã biết. Cách nhìn này thường được quan tâm hàng đầu và thường là nội dung chính trong các bài viết, các tài liệu về toán phổ thông. Bên cạnh đó, tôi cũng muốn trình bày các vấn đề ở đây dưới một cách nhìn khác, cách nhìn của người muốn tìm tòi, phát hiện ra các bài toán mới, những bài toán không chỉ mới về nội dung mà còn có tác dụng tích cực trong việc rèn luyện tư duy và các kỹ năng cần thiết của người học, đặc biệt là đối với những học sinh giỏi. Đây là công việc đòi hỏi ở chúng ta nhiều công phu không kém gì công việc giải quyết các bài toán. Tuy nhiên, ở nước ta dường như công việc này còn chưa được quan tâm đúng mức. Đây đó, được ưa chuộng hơn cả vẫn là sử dụng các bài toán hay, mẫu mực đã có hoặc tận dụng các đề toán mới được công bố ở các nước khác. Cách làm này khá tiện lợi, hợp lý và hiệu quả nhưng thực tế có hai nguy cơ:
∙ Một là, nếu sử dụng các bài toán đã được công bố trong các kỳ thi, việc đánh giá sẽ thiếu công bằng và chính xác;
∙ Hai là, đáp án của nhiều bài toán do vô tình hay hữu ý, đã ít nhiều bị biến dạng. Điều này có thể làm cho cách trình bày trở nên ngắn gọn hơn nhưng đồng thời cũng đã làm mất đi những ý tưởng trong sáng và tự nhiên ban đầu khi những bài toán đó được xây dựng nên. Vì thế, nếu sử dụng lại các đáp án một cách máy móc, thiếu sự biên tập cần thiết thì rất có thể chúng sẽ có tác dụng tiêu cực đến việc rèn luyện tư duy của người học.

Với những suy nghĩ đó, tôi nghĩ chắc cũng đã đến lúc chúng ta cần tăng cường sự quan tâm và đầu tư nhiều công sức hơn nữa cho công việc “sáng tác” này. Một công việc không dễ dàng nhưng chắc chắn sẽ rất thú vị và bổ ích. Bây giờ, đã đến lúc đi thẳng vào chủ đề của bài này: Hình học tĩnh hay động? Nếu chỉ nhìn các bài toán mà chúng ta vẫn thường giải quyết hoặc tìm tòi thì hình học vừa tĩnh lại vừa động. Hình học tĩnh trong những bài toán mà ở đó, các yếu tố như điểm, đường thẳng, đường tròn,... đều không thay đổi và yêu cầu đặt ra ở đây thường là chứng minh các tính chất hình học hoặc tính toán các đại lượng nào đó trong hình vẽ đã cho. Còn hình học sẽ động trong những bài toán mà ở đó, bên cạnh các yếu tố cố định, không thay đổi có 1 vài yếu tố thay đổi và yêu cầu ở đây thường là tìm quĩ tích, tìm các điểm cố định hoặc tìm giá trị lớn nhất, nhỏ nhất của một đại lượng hình học. Tuy nhiên, đây chỉ là cái nhìn ban đầu. Trên quan điểm của những người mong muốn đi tìm lời giải cho các bài toán khó và cả trên quan điểm của những người mong muốn phát hiện ra những bài toán hình học mới, theo tôi, hình học luôn luôn cần vận động, vận động ngay cả trong những bài toán mà các yếu tố được cho đều cố định, không đổi. Bởi vì chính cách nhìn, cách tư duy trong các yếu tố của hình vẽ không ngừng biến động, tuơng tác, thậm chí toàn bộ cả hình vẽ đều không thay đổi sẽ giúp chúng ta tìm ra đúng những lời giải đẹp nhất và phản ánh trọn vẹn nhất bản chất hình học của một bài toán.




2. ĐỘNG TRONG BIẾN HÌNH

Một trong những công cụ quan trọng hàng đầu để thực hiện việc biến đổi các yếu tố trong một hình chính là phép biến hình. Không phải ngẫu nhiên mà hiện nay, những lời giải hay nhất của nhiều bài toán hình học cũng như rất nhiều phát hiện hình học thú vị thường nhận được trên cơ sở vận động ý tưởng và kỹ thuật của các phép biến hình.

Thế nhưng để có thể vận dụng chúng một cách hiệu quả, trước hết phải có được một nền tảng tương đối vững chắc về biến hình mà cụ thể là phải nắm bắt được một vài mệnh đề quan trọng và làm quen được với một số tình huống tiêu biểu cho việc thực hiện các động tác biến hình hợp lý.

Vậy đó là những mệnh đề nào, những tình huống nào? Trong khuôn khổ bài này, tôi chỉ xin phép trình bày những gì liên quan đến phép quay, một loại phép biến hình tuy đơn giản nhưng lại có mức độ áp dụng cao và mang lại rất nhiều kết quả phong phú. Tương tự, không khác biệt với phép quay bao nhiêu là phép vị tự quay. Thông thường, phép vị tự quay đem lại các kết quả tổng quát hơn và nâng cao độ phức tạp của bài toán mà vẫn giữ nguyên ý tưởng ban đầu của phép quay.

Nhưng trước khi phát biểu ra đây các mệnh đề, tình huống cần thiết được nhắc ở trên, xin phép được nói qua một chút cái gọi là “cảm hứng” thúc đẩy tôi viết ra những dòng này. “Cảm hứng” đó nảy sinh từ việc xem xét giáo trình Hình học nâng cao lớp 11 vừa được đưa vào giảng dạy từ vài năm học vừa qua, trong đó điểm đáng lưu ý nhất là phần các phép biến hình được trình bày đầy đủ hơn và đặc biệt là đã được phân bố ngay vào đầu năm học (trước đây, phần này chỉ được giảng dạy vào cuối năm lớp 10). Rõ ràng, với sự thay đổi này, hội đồng biên soạn sách giáo khoa cho thấy ý định rất nghiêm túc của mình là tăng cường hơn nữa sự chú ý cho phần các phép biến hình và đây thực sự là điều rất nên làm.

Các phép biến hình chính là mảng kiến thức mà ở đó, học sinh có thể làm được với những ý tưởng và những kỹ năng thích hợp nhất cho việc tiếp thu các kiến thức của toán học hiện đại. Những ý tưởng và những kỹ năng đó là gì? Đó là ý tưởng ánh xạ rất rõ nét trong cách trình bày và hệ thống các phép biến hình. Đó là ý tưởng phân loại và mô tả đầy đủ các lớp phép biến hình (mà tiêu biểu nhất là các phép dời hình). Và tất nhiên, quan trọng hơn cả là qua việc vận dụng các phép biến hình để giải toán, tư duy hình học của học sinh sẽ được nâng lên ở một cấp độ mới. Thay vì chỉ biết tính toán và so sánh các đại lượng hình học (góc, độ dài, diện tích,... ) để từ đó đi đến một chứng minh như trước đây, nay với việc sử dụng các phép biến hình, các em sẽ được tập quan sát những vận động, những tương tác giữa các yếu tố, những cấu trúc tiềm ẩn trong một hình vẽ để rồi từ đó rút ra được những chứng minh, những kết luận sâu sắc, nêu bật toàn diện bản chất của hình vẽ đó.

Những ý định như vậy là rất đúng đắn và chắc cũng đã được hội đồng biên soạn sách giáo khoa đem ra cân nhắc kỹ lưỡng trước khi quyết định việc phân bố lại chương trình sách giáo khoa nâng cao về hình học. Chỉ tiếc một điều, theo nhận xét chủ quan của tôi, là nội dung trình bày trong sách giáo khoa lớp 11 có lẽ vẫn còn chưa đủ để học sinh rèn luyện, nắm bắt và vận dụng công cụ biến hình ở mức độ cần thiết, ít ra là chưa cho phép các em làm quen được với ba ý tưởng quan trọng và bổ ích được kể ra ở trên.
Vậy nên cần bổ sung những điều gì? Xin điểm qua một vài điều tôi cho là quan trọng nhất và nhân tiện, đây cũng chính là trả lời cho câu hỏi đặt ra ở đầu phần này. Đó là phát biểu các mệnh đề, các tình huống chính mà bất cứ ai khi học các phép toán biến hình (cụ thể là phép quay) đều phải biết để có thể vận dụng thực sự tốt công cụ này.

2.1. Sự tồn tại của phép quay. Trước hết, để giúp cho học sinh hiểu rõ và tự tin hơn khi sử dụng các phép biến hình, nên trang bị cho các em các mệnh đề về tồn tại duy nhất của một phép biến hình trong những tình huống đơn giản và thông dụng nhất. Đối với phép quay, mệnh đề sau đáp ứng đủ các yêu cầu đó.

Mệnh đề 2.1. Cho hai đoạn thẳng $AB$ và $A'B'$ sao cho $AB = A'B'$ và $\overrightarrow{AB}\neq \overrightarrow{A'B'}$ Lúc đó, tồn tại duy nhất một phép quay $R$ biến tương ứng $AB$ thành $A'B'$.


phepquay1.png


Mệnh đề này cho phép ta chỉ cần quan sát thấy có hai đoạn thẳng bằng nhau là có thể liên tưởng ngay đến một phép quay và sẵn sàng vận dụng nó nếu có thêm các điều kiện thích hợp chứ không phải chờ đến khi có được hai tam giác, hai hình bằng nhau mới bắt đầu nghĩ đến phép quay. Ngoài ra, mệnh đề này còn là cơ sở để mô tả đầy đủ các phép dời hình (sẽ đề cập ở dưới). Tuy nhiên, nó chỉ có ý nghĩa giúp ta làm quen với tình huống. Muốn mang lại hiệu quả thực sự phải bổ sung thêm một ít về việc xác định phép quay tồn tại nói trên.

Mệnh đề 2.2 (Mệnh đề 1 bổ sung). Phép quay $R$ có góc quay $\alpha = (\overrightarrow{AB}, \overrightarrow{A'B'})$ và tâm $O$ đồng thời nằm trên các trung trực của $AA',BB'$ cũng như các cung tròn (đơn) chứa các điểm nhìn đoạn $AA',BB'$ dưới một góc có hướng bằng $\alpha$.


Bổ sung này cho ta một cái nhìn khá toàn diện về tình huống đang xét (xem hình vẽ); nhưng để có được sự quan sát đầy đặn và sâu sắc hơn nữa, cần trang bị thêm:

Mệnh đề 2.3. Ta giữ các giả thiết như trong các Mệnh đề 1 và 2.
(1) Giả sử các đường thằng $AB$ và $A'B'$ cắt nhau tại điểm $P$. Khi đó, các tứ giác
$AA'OP$ và $BB'OP$ nội tiếp;
(2) Giả sử các đường thẳng $AA'$ và $BB'$ cắt nhau tại điểm $Q$. Khi đó, các tứ giác
$ABOQ$ và $A'B'OQ$ nội tiếp.

Các mệnh đề này rõ ràng là chứng minh không khó (nên xin bỏ qua ở đây). Còn lợi ích mà chúng có thể mang lại thì lại khá phong phú. Xin bắt đầu bằng một bài tập khá quen thuộc trong đó việc vận dụng ý tưởng biến hình là rất tự nhiên và đơn giản.

Ví dụ 2.4. Cho tam giác $ABC$ cân tại $A$. Trên cạnh $AB$ và $AC$ lần lượt lấy các điểm
$M, N$ sao cho $AM = CN$. Chứng minh đường tròn ngoại tiếp tam giác $AMN$ luôn đi qua một điểm cố định khác $A$.


PHEPQUAY2.png


Lời giải. Để giải, ta xét phép quay $R$ biến đọan thẳng $AM$ tương ứng thành đoạn thẳng $CN$.
Tâm quay $O$ theo mệnh đề 2 là giao điểm của trung trực $AC$ và cung tròn quĩ tích những điểm $K$ sao cho:

$$\left (\overrightarrow{KA}, \overrightarrow{KC} \right ) = \left (\overrightarrow{AM}, \overrightarrow{CN} \right )$$

nên tâm quay $O$ cố định.
Cuối cùng, do $AM$ và $CN$ cắt nhau tại $A$ nên theo Mệnh đề 3, tứ giác $MNAO$ nội tiếp. Vậy đường tròn ngoại tiếp tam giác $AMN$ đi qua điểm $O$ cố định.

Bài tập này rất thích hợp cho việc làm quen với các ứng dụng của phép quay. Nó chỉ có một khiếm khuyết là nếu tam giác $ABC$ cân thì điểm $O$ cần tìm chính là tâm đường tròn ngoại tiếp tam giác $ABC$. Do đó, nhiều học sinh có thể mày mò, dự đoán và chứng minh kết quả trên mà không cần sử dụng phép quay. Thực ra, để khắc phục điều này, có thể xem tam giác $ABC$ không cân và còn tổng quát hơn là bài tập sau mà cách giải không có gì thay đổi.

Ví dụ 2.5. Trên 2 tia $Ox$ và $Oy$ của góc $Oxy$, cho 2 điểm $A,B$. Gọi $M,N$ là 2 điểm thay đổi trên $Ox, Oy$ sao cho $AM=BN$ ($M$ khác phía $O$ đối với điểm $A$, còn $N$ cùng phía $O$ đối với điểm $B$). Chứng minh rằng đường tròn ngoại tiếp tam giác $OMN$ luôn đi qua một điểm cố định khác $O$


PHEPQUAY3.png


Nếu bổ sung vào bài tập này thêm một vài yếu tố với những mối quan hệ tương tự (chẳng hạn lấy thêm các điểm $P,Q$ trên $Ox,Oy$ cũng với tính chất $AP=BQ$ để phép quay được xét cũng biến $P$ thành $Q$) và thay đổi chút ít cách phát biểu cũng như vận dụng tính chất còn lại (tính chất 2) của Mệnh đề 3. Ta nhận được:

Bài toán. Cho tứ giác $ABC$ có $AB=CD$ và các điểm $M,N$ trên $AB,CD$ sao cho $AM=DN$. Giả sử đường thẳng $MN$ cắt $AD$ và $BC$ lần lượt tại $P,Q$. Chứng minh rằng tồn tại một điểm $O$ có cùng phương tích với tất cả bốn đường tròn ngoại tiếp các tam giác $PAM,PDN,QBM,QCN$


phepquay4.png


Lời giải. Gọi $O$ là tâm của phép quay $R$ biến $AB$ tương ứng thành $CD$ và $M$ thành $N$. Theo mệnh đề 3 (tính chất 2), các tứ giác $AMOP,ANOP,BMOQ,CNOQ$ đều nội tiếp. Vậy $O$ nằm trên bốn đường tròn nội tiếp các tam giác $PAM,PDN,QBM,QCN$
nên $O$ có cùng phương tích đối với các đường tròn này.

2.2. Tích của hai phép quay. Điều cần bổ sung thứ hai liên quan đến bản chất ánh xạ của các phép biến hình. Một khi đã định nghĩa chúng như các ánh xạ thì lẽ tự nhiên cũng cần phải đề cập đến tích của hai phép biến hình. Vậy tích của hai phép quay là gì?

Mệnh đề 2.6. Cho hai phép quay $R(O_1; \alpha _1), R(O_2; \alpha _ 2)$. Nếu $\alpha _1 + \alpha _2 = 2k\pi$ thì tích $R = R_2.R_1$ cũng là một phép quay với góc quay $\alpha = \alpha _1 + \alpha _2$. Tâm $O$ của phép quay này được xác định từ điều kiện sau
$$\left (\overrightarrow{O_1O}, \overrightarrow{O_1O_2} \right ) = \frac{\alpha_1}{2};\left (\overrightarrow{O_2O_1}, \overrightarrow{O_2O} \right ) = \frac{\alpha_2}{2}$$


phepquay5.png


Chứng minh. Việc $R$ là một phép quay có thể suy ra ngay từ Mệnh đề 1. Còn tâm $O$ chính là điểm bất động duy nhất qua tích
$R = R_2.R_1$ nên nếu chọn $O$ như trên
và lấy $O’$ đối xứng với $O$ qua $O_1O_2$ thì ta
có $R_1(O)=O’$ và $R_2(O’) = O$. Suy ra
$R(O) = O$. Vậy điểm $O$ xác định với điều kiện trên chính là tâm quay.

Bài tập sau có thể xem là ứng dụng mẫu mực việc vận dụng tích 2 phép quay.

Ví dụ 2.7. Bên ngoài tam giác $ABC$ và trên các cạnh dựng các tam giác $BCA_1,CAB_1,ABC_1$ cân lần lượt tại $A_1,B_1,C_1$ với góc $BA_1C = 160^o$ và các góc $CB_1A=AC_1B= 100^o$. Tính góc $B_1A_1C_1$


phepquay6.png


Lời giải. Nhận xét rằng:
$$R(A_1;-160^o)=R(B_1;100^o).R(C_1;100^o)$$
Như vậy, theo tính chất tâm của tích hai phép quay thì góc $B_1A_1C_1 = 80^o$


Tất nhiên, với đề bài như trên, một số học sinh vẫn có thể đi “tính” góc $B_1A_1C_1$ với một khối lượng tính toán hết sức cồng kềnh và với kỹ thuật tính toán đáng nể. Nếu bây giờ biến tấu bài tập này đi một chút bằng cách cất đi điểm “mấu chốt” $A_1$ và gắn thêm tính di động cho các điểm $B_1,C_1$, ta nhận được phương án sau:


Bài toán. Cho tam giác $ABC$ nội tiếp đường tròn $(O)$ có $B,C$ cố định, còn $A$ thay đổi trên $(O)$. Bên ngoài tam giác, trên các cạnh $AB,AC$ dựng các tam giác $ABC_1,ACB_1$ với $\widehat{AC_1B}=\widehat{AB_1C}=100^o$. Chứng minh rằng trung trực của $B_1C_1$ luôn đi qua một điểm cố định.

Rõ ràng điểm cố định cần tìm chính là điểm $A_1$ trong bài tập trên nay đã được “giấu” đi. Và chính vị trí không dễ đoán của $A_1$ đã làm cho bài toán trở nên vô cùng khó khăn cho những ai chưa nắm được ý tưởng về tích của hai phép quay.

2.3. Về các phép dời hình khác. Để kết thúc phần này, xin nêu ra điều cần bổ sung cuối cùng để cho nội dung về phép biến hình được cân đối, hoàn chỉnh. Chúng ta biết rằng lớp các phép biến hình được trình bày đầy đủ nhất chính là lớp các phép dời hình. Chúng có thể được mô tả rất trọn vẹn thông qua các phép dời hình cơ sở là tịnh tiến, quay và đối xứng trục. Vậy nên chăng sau khi đã học xong các phép biến hình cụ thể này, chúng ta sẽ khái quát bằng khái niệm các phép dời hình và kết thúc bằng một mệnh đề mô tả đầy đủ lớp các phép dời hình để làm sáng tỏ bản chất khá đơn giản của chúng. Đây thường là sơ đồ mẫu mực khi trình bày về một lớp các phép biến đổi nào đó trong các lĩnh vực khác của toán học.
Mệnh đề mô tả các phép dời hình ở đây rất gọn, đơn giản và có thể suy ra

trực tiếp từ Mệnh đề 1 ở trên. Nhưng trước khi phát biểu nó, theo tôi nên phân loại các phép dời hình thành các phép dời hình thuận (là các phép dời hình bảo toàn định hướng) và các phép dời hình ngược (thay đổi định hướng). Điều này cũng gần giống như việc phân biệt hai tam giác bằng nhau thuận và bằng nhau nghịch mà học sinh đã rất quen thuộc. Việc phân loại các phép dời hình như vậy sẽ không gây ra khó khăn nào mà trái lại, nó còn có thể giúp học sinh hiểu và cảm nhận rõ ràng hơn về định hướng (cụ thể là chiều “quay” của một tam giác) trong các phép biến hình.
Đối với các phép dời hình thuận (quan trọng nhất và được xem xét kỹ lưỡng nhất) ta có sự mô tả đầy đủ sau:

Mệnh đề 2.8. Một phép dời hình thuận chỉ có thể là một phép tịnh tiến hoặc một phép quay.

Đối với các phép dời hình nghịch thì khó khăn hơn một chút:

Mệnh đề 2.9. Một phép dời hình nghịch có thể được biểu diễn như là tích một phép tịnh tiến với một phép đối xứng trục.


Trong phần bài tập của bộ sách giáo khoa Hình học nâng cao lớp 11, dạng tích này cũng được xét đến và được gọi là phép “đối xứng trượt”. Theo tôi, Mệnh đề 2.9 có thể không nhất thiết phải trình bày hoặc chỉ cần nhắc qua và đưa ra như một bài tập. Nhưng Mệnh đề 2.8 thì nên phát biểu như một lời đúc kết của phần các phép dời hình để sao cho khi học xong phần này, học sinh có cảm giác nắm bắt trọn vẹn, rõ ràng, không còn chút gì mơ hồ về các phép dời hình.

(còn nữa)




#340954 Chuyên đề Hệ phương trình

Đã gửi bởi Ban Biên Tập on 28-07-2012 - 00:00 trong Chuyên đề toán THPT

$ \bullet $ Mở rộng cách nhìn về hệ đối xứng kiểu II.

Trước hết, hãy xem xét cách giải hệ phương trình sau:

Ví dụ 23: $\left\{\begin{matrix} x^3+y^2=1 & \\y^3+x^2=1 & \end{matrix}\right.$

Bài giải: Trừ theo vế 2 phương trình của hệ ta thu được:
\[{x^3} - {y^3} + {y^2} - {x^2} = 0 \Leftrightarrow \left( {x - y} \right)\left[ {{x^2} + xy + {y^2} - \left( {x + y} \right)} \right] = 0\]
$$ \Leftrightarrow \left[ \begin{array}{l}x = y\\ {x^2} + xy + {y^2} = x + y \end{array} \right.$$


Trường hợp: $x = y$ thì thế và giải phương trình:
$$x^3+x^2-1=0\Leftrightarrow x=y=\frac{1}{3}\left [ \sqrt[3]{\frac{25}{2}-\frac{3\sqrt{69}}{2}}+\sqrt[3]{\frac{25}{2}+\frac{3\sqrt{69}}{2}}-1 \right ]$$
Cộng theo vế 2 phương trình và kết hợp với $x^2+xy+y^2=x+y$ ta được hệ đối xứng loại I:
$$\left\{\begin{matrix} x^2+xy+y^2=x+y & \\x^3+y^3+x^2+y^2=2 & \end{matrix}\right.$$
$$\Leftrightarrow \left\{\begin{matrix} (x+y)^2-xy=(x+y) & \\(x+y)^3-3xy(x+y)+(x+y)^2-2xy=2 & \end{matrix}\right.$$
Đặt $S=x+y$, $P=xy$, hệ trở thành:

\[\left\{ \begin{array}{l} {S^2} - P = S\\{S^3} - 3SP + {S^2} - 2P = 2\end{array} \right. \Leftrightarrow \left\{ \begin{array}{l}{S^3} + {S^2} - 2 - (3S + 2)({S^2} - S) = 0\\P = {S^2} - S\end{array} \right.\]

\[\left\{ \begin{array}{l} (S - 1)({S^2} + 1) = 0\\ P = {S^2} - S \end{array} \right. \Leftrightarrow \left\{ \begin{array}{l} S = 1\\ P = 0 \end{array} \right. \Leftrightarrow \left\{ \begin{array}{l} x = 1,y = 0\\ x = 0,y = 1 \end{array} \right.\]
Vậy hệ đã cho có 3 nghiệm như trên.

Hình đã gửi

$ \bullet $ Qua bài giải trên, hẳn chúng ta nhận rõ vai trò của việc kết hợp “cộng” và “trừ” để đưa đến hpt đối xứng kiểu I (đây là một hướng nhìn mới). Việc làm này hoàn toàn có cơ sở. Hãy xem lại câu nói: “Cũng như loại I, loại II cũng có “đối xứng” nhưng là đối xứng giữa 2 phương trình chứ không phải là đối xứng trong từng phương trình như kiểu I”. Như vậy, khi cộng theo vế sẽ luôn cho một trong hai phương trình đối xứng kiểu I. Và việc lấy đi nghiệm $(x-y)$ sau khi trừ cũng để lại cho ta 1 phương trình đối xứng nữa.

Ví dụ 24: Giải hệ phương trình: $\left\{\begin{matrix} x^3+y=2 & \\y^3+x=2 & \end{matrix}\right.$

$ \bullet $ Và đôi lúc việc cộng trừ cũng không đem lại cho ta kết quả khả quan:

Ví dụ 25: Giải hệ phương trình: $\left\{\begin{matrix} x+\frac{2xy}{\sqrt[3]{x^2-2x+9}}=x^2+y & \\y+\frac{2xy}{\sqrt[3]{y^2-2y+9}}=y^2+x & \end{matrix}\right.$

Nhận xét: Quả đúng, khi cộng hay trừ ta không thể làm gì với cái căn khủng khiếp kia. Tuy nhiên, một số điểm ta lại thấy rõ và đáng phải nghĩ là trong cái căn bậc 3 kia, có một đẳng thức: $(x-1)^2+8=x^2-2x+9$. Nhẩm nghiệm $x=y=1$ và $\sqrt[3]{8}=2$ (căn đẹp!). Phải liên kết và sử dụng chúng như thế nào?

Bài giải: Cộng theo vế 2 hệ của phương trình: $\frac{2xy}{\sqrt[3]{x^2-2x+9}}+\frac{2xy}{\sqrt[3]{y^2-2y+9}}=x^2+y^2$.

Sử dụng đánh giá:
$$\sqrt[3]{x^2-2x+9}=\sqrt[3]{(x-1)^2+8}\geq 2\Rightarrow \frac{2xy}{\sqrt[3]{x^2-2x+9}}\leq \frac{2xy}{2}=xy$$
Tương tự ta có:
$$\frac{2xy}{\sqrt[3]{y^2-2y+9}}\leq xy\Rightarrow \frac{2xy}{\sqrt[3]{x^2-2x+9}}+\frac{2xy}{\sqrt[3]{y^2-2y+9}}\leq 2xy=x^2+y^2-(x-y)^2\leq x^2+y^2$$
Vậy hệ có nghiệm khi $x=y=1$, thử lại thấy đúng, kết luận nghiệm.

$ \bullet $ 2 kiểu hệ đối xứng I và II là những dạng rất cơ bản. Tuy nhiên, qua “chế biến” của người ra đề thì không thể nói trước được điều gì. Vì vậy, cần có cái nhìn tổng quan, nhìn nhiều khía cạnh, không nên chỉ biết nhìn hình thức rồi rập khuôn lời giải của dạng. Một số ví dụ thêm:

Ví dụ 26: (Thi thử ĐH CĐ THPT Lê Văn Hưu, Thanh Hóa năm 2011)
Giải hệ phương trình: $\left\{\begin{matrix} 2x^2+x-\frac{1}{y}=2 & \\y-y^2x-2y^2=-2 & \end{matrix}\right.$

Ví dụ 27: Giải hệ phương trình: $\left\{\begin{matrix} x^3(2+3y)=1 & \\(y^3-2)x=3 & \end{matrix}\right.$

Ví dụ 28: (Tuyển sinh vào lớp 10 THPT Chuyên Phan Bội Châu 2009-2010)
Giải hệ phương trình: $\left\{\begin{matrix} x^3(2+3y)=8 & \\(y^3-2)x=6 & \end{matrix}\right.$

Ví dụ 29: (Thi ĐH - CĐ khối B 2003)
Giải hệ phương trình: $\left\{\begin{matrix} 3y=\frac{y^2+2}{x^2} & \\3x=\frac{x^2+2}{y^2} & \end{matrix}\right.$

Ví dụ 30: (Olympic 30-4-2010)
Giải hệ phương trình: $\left\{\begin{matrix} \sqrt{x^2+2x+22}-\sqrt{y}=y^2+2y+1 & \\\sqrt{y^2+2y+22}-\sqrt{x}=x^2+2x+1 & \end{matrix}\right.$

Ví dụ 31: (Thi thử ĐH năm 2011, THTT số 379, 2009)
Giải hệ phương trình: $\left\{\begin{matrix} x+\sqrt{x^2-2x+2}=3^{y-1}+1 & \\y+\sqrt{y^2-2y+2}=3^{x-1}+1 & \end{matrix}\right.$

Ví dụ 32: Giải hệ phương trình: $\left\{\begin{matrix} x+\sqrt{x^2-2x+2}=3^{y-1}+1 & \\y+\sqrt{y^2-2y+2}=3^{x-1}+1 & \end{matrix}\right.$

III. MỘT SỐ PHƯƠNG PHÁP MỚI

1. Phương pháp 01: Hằng số = $t$ = ẩn số:

Xem xét cách giải một số ví dụ sau:


Ví dụ 1: (Hệ phương trình TST Nghệ An 2009-2010)
Giải hệ phương trình: $\left\{\begin{matrix} x^2+y^2=\frac{1}{5} & \\4x^2+3x-\frac{57}{25}=-y(3x+1) & \end{matrix}\right.$

Bài giải: Nhân phương trình sau của hệ với $2$ rồi cộng theo vế với phương trình đầu ta được:
$$9{x^2} + {y^2} + 6xy + 6x + 2y = \frac{{119}}{{25}} \Leftrightarrow {(3x + y + 1)^2} = \frac{{144}}{{25}} \Leftrightarrow \left [ \begin{array}{l}3x + y + 1 = \frac{{12}}{5}\\3x + y + 1 = - \frac{{12}}{5}\end{array} \right.$$

Đến đây thì thế vào phương trình ban đầu ta giải phương trình bậc 2 nữa là xong.


Ví dụ 2: Giải hệ phương trình: $\left\{\begin{matrix} (x-y)^2+y=3 & \\x^2+2xy-5y^2-5x+13y=6 & \end{matrix}\right.$

Bài giải: Nhân 3 vào phương trình đầu rồi trừ theo vế với phương trình sau ta được:
$$2{x^2} + 8{y^2} - 8xy + 5x - 10y = 3 \Leftrightarrow 2{(x - 2y)^2} + 5(x - 2y) - 3 = 0 $$

$$\Leftrightarrow (x - 2y + 3)(2x - 4y - 1) = 0 \Leftrightarrow \left[ \begin{array}{l}x - 2y = - 3\\2x - 4y = 1\end{array} \right.$$



Đến đây, thế từng trường hợp rồi thay vào phương trình ban đầu là xong.

$ \bullet $ Nhận xét: Hai bài giải trên thật hay, đơn giản với công việc nhân thêm rồi cộng lại, sau đó phân tích thành nhân tử.


Nhưng! Điều chúng ta băn khoăn và thắc mắc ở đây chính là việc biết phải nhân với con số nào. Đây chính là cơ sở để chúng ta đi đến phương pháp ẩn số $= t =$ hằng số.

$ \bullet $ Như chúng ta đã biết, cái chưa biết chính là ẩn số. Đây cũng vậy, để biết cần nhân với bao nhiêu, ta đưa thêm ẩn $t$ vào. Do đó, hpt của chúng ta đã có đến tận 3 ẩn với chỉ 2 giả thuyết. Như vậy, phải có thêm một cái gì đó ràng buộc. Nó là gì? Quan sát lại 2 ví dụ trên một lần nữa.

$ \bullet $ Phương pháp: Hằng số $= t =$ ẩn số:

- Phạm vi ứng dụng: hệ phương trình 2 ẩn $x$, $y$ có bậc không quá 2.

- Cơ sở phương pháp: giải phương trình bậc 2.


Xét phương trình: $ax^2+bx+c=0$. Có: $\Delta =b^2-4ac$.

Nếu: $\Delta

Nếu $\Delta >0$ phương trình có 2 nghiệm phân biệt.

Đặc biệt: $\Delta =0$ phương trình có 1 nghiệm duy nhất, tức là khi đó phương trình tương đương với: $$a(x+\frac{b}{2a})^2=0$$. Đây chính là cơ sở cơ bản của phương pháp
.
(Bài viết sẽ không trình bày giải hệ phương trình tổng quát mà sẽ thực hiện giải chi tiết những ví dụ cụ thể nhằm tạo cho bạn những tu duy, suy nghĩ mới và tự hình thành cho mình những phương pháp và kĩ năng. Hơn nữa việc trình bày tổng quát khá phức tạp)

Hãy xem xét lại 2 ví dụ trên:

Thay vì nhân vào những con số $2$ như Ví dụ 1, con số $3$ như Ví dụ 2 mà có vẻ dường như ta đã biết, ta sẽ nhân vào đó con số $t$.


Ví dụ 1: Giải hệ phương trình: $\left\{\begin{matrix} x^2+y^2=\frac{1}{5} & \\4x^2+3x-\frac{57}{25}=-y(3x+1) & \end{matrix}\right.$

Nhân $t$ vào phương trình đầu rồi cộng theo vế với phương trình sau ta có:
$$ty^2+y(3x+1)+(t+4)x^2+3x-\frac{5t+57}{25}=0$$
Xem đây là phương trình bậc 2 ẩn $y$, xét:
\[{\Delta _y} = {(3x + 1)^2} - 4t\left[ {(t + 4){x^2} + 3x - \frac{{5t + 57}}{{25}}} \right]\]
\[ = (9 - 4{t^2} - 16t){x^2} + 6x(1 - 2t) + 1 + \frac{{4t(5t + 57)}}{{25}}\]
Để xuất hiện nhân tử như trên thì $\Delta _{y}=f^2(x)$ và như vậy thì:
$$(9-4t^2-16t)x^2+6x(1-2t)+1+\frac{4t(5t+57)}{25}=f^2(x)$$
$$\Leftrightarrow \Delta '_{x}=0\Leftrightarrow 9(1-2t)^2-4(9-16t-4t^2)\left [ 1+\frac{4t(5t+57)}{25} \right ]=0$$
$$\Leftrightarrow (1-2t)\left [ 1-2t-4(9+2t)\left [ 1+\frac{4t(5t+57)}{25} \right ] \right ]=0$$
Dễ thấy $t=\frac{1}{2}$ là giá trị thỏa mãn.

$ \bullet $ Để có lời giải gọn và đẹp thì khi trình bày bài giải, chúng ta nhân thêm $2$ vào phương trình sau thay vì nhân $\frac{1}{2}$ vào phương trình đầu. Từ đó ta có lời giải gọn và đẹp như trên.


Xem lại ví dụ 2: Giải hệ phương trình: $\left\{\begin{matrix} (x-y)^2+y=3 & \\x^2+2xy-5y^2-5x+13y=6 & \end{matrix}\right.$

Chúng ta cũng thực hiện công việc nhân $t$ như trên: Nhân $t$ vào phương trình đầu rồi cộng theo vế 2 phương trình ta được:
$$(t-5)y^2+y\left [ 2x(t-1)+t+13 \right ]+(t+1)x^2-5x-3(t+2)$$
$$\Delta _{y}=\left [ 2x(t-1)+t+13 \right ]^2-4(t-5)\left [ (t+1)x^2-5x-3(t+2) \right ]$$
$$=8(3+t)x^2-4x(t^2+7t+12)+9t^2+2t+249=f^2(x)$$
Khi xem xét phương trình này thì nhận thấy ngay $t=-3$ sẽ cho ta $f^2(x)=18^2$ vì để ý $t^2+7t+12=(t+3)(t+4)$. Từ đó ta có lời giải.

Một số ví dụ thêm:

Ví dụ 3: Giải hệ phương trình: $\left\{\begin{matrix} x^2+y^2=xy+x+y & \\x^2-y^2=3 & \end{matrix}\right.$

Ví dụ 4: Giải hệ phương trình: $\left\{\begin{matrix} x^2+2y^2=3x-2 & \\2(x+y-1)=2xy & \end{matrix}\right.$

Ví dụ 5: (THTT số 379, tháng 1 năm 2011)
Giải hệ phương trình: $\left\{\begin{matrix} y^2=(5x+4)(4-x) & \\y^2-5x^2-4xy+16xy-8y+16=0 & \end{matrix}\right.$

Ví dụ 6: (ĐH CĐ khối A, năm 2008)
Giải hệ phương trình: $\left\{\begin{matrix} x^2+y+x^3y+xy^2+xy=-\frac{5}{4} & \\x^4+y^2+xy(1+2x)=-\frac{5}{4} & \end{matrix}\right.$

Ví dụ 7: Giải hệ phương trình: $\left\{\begin{matrix} x^2-y^2=3 & \\x^2+y^2=xy+x+y & \end{matrix}\right.$

$ \bullet $ Một số lưu ý khi sử dụng phương pháp (Xem ở phần tản mạn)

Cần linh hoạt trong việc chọn lựa nhân $t$ ở phương trình nào để thuận lợi trong việc phân tích.
……


$ \bullet $ Mở rộng phương pháp:

$ \bullet $ Cở sở suy luận: bạn có nghĩ, liệu có bắt buộc bậc của $x$ và $y$ trong hệ phải là bậc 2 cả. Đúng, để luôn giải được thì nhất thiết phải yêu cầu là cả 2 đều bậc 2. Tuy nhiên, cái hay của Toán chính là đa dạng, muôn màu muôn vẻ, bắt buộc chúng ta phải luôn tinh tế, sáng tạo hơn nữa trong phương pháp và suy nghĩ. Biết 1 chưa chắc đã giải được 10. Trước hết, hãy xem cái hệ sau.

Ví dụ 8: (Thi thử ĐH CĐ năm 2011)
Giải hệ phương trình: $\left\{\begin{matrix} x^4+2x^3y+x^2y^2=2x+9 & \\x^2+2xy=6x+6 & \end{matrix}\right.$

Bậc cao nhất của $x$ là 4, nhưng bậc của $y$ lại là 2. Hơn thế nữa, nếu quan sát tinh ý hơn:
$$\left\{\begin{matrix} x^4+2x^2(xy)+(xy)^2=2x+9 & \\x^2+2xy=6x+6 & \end{matrix}\right.$$
(thì nên gom và xem $xy$ là ẩn)

Nhân thêm hằng số $t$ vào phương trình sau rồi cộng theo vế với phương trình đầu, ta được:
$$x^2y^2+2xy(t+x^2)+x^4+tx^2-2x(1+3t)-9-6t=0$$
$$\Delta '_{xy}=(t+x^2)^2-x^4-tx^2+2x(1+3t)+9+6t=tx^2+2x(1+3t)+(t+3)^2=f^2(x)$$
$$\Leftrightarrow \Delta '_{x}=(1+3t)^2-4t(t+3)^2=0$$
Dễ thấy ngay $t=1$ là một nghiệm của phương trình nên hệ số cần nhân chính là 1.

Việc trình bày lời giải còn lại xin dành cho bạn đọc.

$ \bullet $ Hệ này khá đặc biệt nhưng vì rút gọn ta thu được $\Delta _{xy}$ là một tam thức bậc 2. Qua đó cơ sở phương pháp của chúng ta vẫn áp dụng được. Nhưng. Ví dụ sau thì sao?

Ví dụ 9: Giải hệ phương trình: $\left\{\begin{matrix} 1+x^2y^2=19x^2 & \\xy^2+y=-6x^2 & \end{matrix}\right.$

$ \bullet $ Nhận xét: chú ý bậc cao nhất của $y$ như trên vẫn là bậc 2. Nhưng có vấn đề gì cần bàn ở đây?
Nháp: Nhân $t$ vào phương trình sau rồi cộng lại.


Còn tiếp ...

Mời bạn cùng thảo luận tại: http://diendantoanho...ệ-phương-trinh/



#340082 Vài bài toán hay về Bất đẳng thức lượng giác trong tam giác (phần 1)

Đã gửi bởi Ban Biên Tập on 25-07-2012 - 16:27 trong Chuyên đề toán THPT


VÀI BÀI TOÁN HAY VỀ ĐẲNG THỨC LƯỢNG TRONG TAM GIÁC

Đoàn Quốc Khánh, học sinh Lớp 11A1


trường THPT Ngọc Lặc, huyện Ngọc Lặc, tỉnh Thanh Hóa



I. Lời giới thiệu

Trong môn hình học ở trường phổ thông, hình học phẳng có khá nhiều phân môn, thể loại, và hình tam giác, có vai trò rất đặc biệt. Việc chứng minh nhiều định lý và giải rất nhiều bài toán hình học đòi hỏi phải vận dụng hợp lý nhiều kiến thức về hình tam giác(tam giác bằng nhau, tam giác đồng dạng, các đường thẳng đặc biệt trong tam giác, v.v…)
Hình tam giác đã được nhiều nhà toán học trên thế giới nghiên cứu từ hàng nghìn năm nay và mãi cho đến những năm gần đây, nhiều tính chất, định lý mới, hoặc nhiều cách chứng minh mới của các định lý đã biết lần lượt ra đời. Ở bài viết này, tác giả xin giới thiệu đến bạn đọc những định lý, những bài toán hay về đẳng thức lượng giác trong tam giác, bao gồm định lý Stewart, định lý Morley, định lý Steiner-Lenmus về tam giác cân, bài toán Napoleon … và những mở rộng, chú ý, cách chứng minh độc đáo của nhiều nhà toán học cũng được nêu ra trong bài viết này, chúng ta hãy cùng tìm hiểu.



Hình đã gửi


Hình đã gửi


II. Định lý STEWART


Bài toán: Cho$\Delta ABC$. $D$ là một điểm trên cạnh $BC$. Đặt $AD = d, BD = m, DC = n$. Khi đó ta có công thức sau: $a{d^2} = m{b^2} + n{c^2} - amn$
Lời giải.


Hình đã gửi



Kẻ đường cao $AH$. Xét hai tam giác $ABD$ và $ACD$ và theo định lý hàm số $cos$, ta có:
\[{c^2} = {d^2} + {m^2} - 2md\cos \widehat {ADB} = {d^2} + {m^2} - 2m.HD\,\,\,\,\,\,\,(1)\]
$${b^2} = {d^2} + {n^2} - 2dn\cos \left( {\pi - \widehat {ADB}} \right) = {d^2} + {n^2} + 2dn\cos \widehat {ADB} = $$
$$={d^2} + {n^2} + 2nHD\,\,\,\,\,\,(2)$$
Nhân từng vế $(1), (2)$ theo thứ tự với $n$ và $m$ rồi cộng lại, ta có:
$$n{c^2} + m{b^2} = {d^2}(n + m) + mn(m + n) \,\,\,\,\,\,\,(3)$$
Do $m + n = a$, nên từ $(3)$ ta có: $a{d^2} = m{b^2} + n{c^2} - amn$

Định lý Stewart chứng minh xong.
Chú ý:
$ \bullet $ Stewart (1717 – 1785) là nhà toán học và thiên văn học người Scotland.
$ \bullet $ Nếu trong hệ thức Stewart xét $AD$ là đường trung tuyến, thì từ hệ thức Stewart có:
\[am_a^2 = \frac{a}{2}{b^2} + \frac{a}{2}{c^2} - a\frac{a}{2}\frac{a}{2} \Leftrightarrow m_a^2 = \frac{{2{b^2} + 2{c^2} - {a^2}}}{4}\,\,\,\,\,\,\,\,\,\,(4)\]
Hệ thức trên chính là hệ thức xác định trung tuyến quen biết trong tam giác.
$ \bullet $ Nếu trong hệ thức Stewart xét $AD$ là phân giác. Khi đó theo tính chất đường phân giác trong ta có:
\[\frac{m}{c} = \frac{n}{b} \Leftrightarrow m = \frac{{ac}}{{b + c}}\,\,\,\,\,\,\text{và}\,\,\,\,\,\,n = \frac{{ab}}{{b + c}}\]
Từ hệ thức Stewart có:
\[al_a^2 = \frac{{ac}}{{b + c}}{b^2} + \frac{{ab}}{{b + c}}{c^2} - a\frac{{{a^2}bc}}{{{{\left( {b + c} \right)}^2}}} \Rightarrow l_a^2 = \frac{{bc\left[ {{{\left( {b + c} \right)}^2} - {a^2}} \right]}}{{{{\left( {b + c} \right)}^2}}}\,\,\,\,\,\,\,\,\,(5)\]
Chú ý rằng: $$2{\cos ^2}\frac{A}{2} = 1 + \cos A = 1 + \frac{{{b^2} + {c^2} - {a^2}}}{{2bc}} = \frac{{{{(b + c)}^2} - {a^2}}}{{2bc}}\,\,\,\,\,\,\,\,\,\,(6)$$
Từ $(5)$ và $(6)$ suy ra:
$$l_a^2 = \frac{{4{b^2}{{\cos }^2}\frac{A}{2}}}{{{{(b + c)}^2}}} \Rightarrow {l_a} = \frac{{2bc\cos \frac{A}{2}}}{{b + c}}$$
Hệ thức trên chính là hệ thức xác định đường phân giác.
Vậy hệ thức Stewart là tổng quát hoá của các hệ thức xác định đường trung tuyến và đường phân giác đã quen biết.
III. Định lý MORLEY
Bài toán: Cho $\Delta ABC$. Ở mỗi góc của tam giác vẽ hai đường chia góc đó ra làm ba phần bằng nhau. Các đường ấy cắt nhau tại $X, Y, Z$ (hình vẽ). Chứng minh rằng $\Delta XYZ$ đều.
Lời giải.
Hình đã gửi


Đặt $A = 3\alpha $, $B = 3\beta $,$C = 3\gamma $. Gọi các cạnh $BC, CA, AB$ và đường kính đường tròn ngoại tiếp $\Delta ABC$ là $a, b, c, d$. Theo định lý hàm số $sin$ trong$\Delta CYA$, ta có:
$$\frac{{CY}}{{\sin \alpha }} = \frac{b}{{\sin \left( {{{180}^o} - \alpha - \gamma } \right)}}\,\,\,\,\,\,\,\,\,\,\,(1)$$
Do $\alpha + \gamma = {60^o} - \beta $, vậy từ $(1)$ suy ra
\[{CY = b\frac{{\sin \alpha }}{{\sin \left( {{{120}^o} + \beta } \right)}} = d\sin 3\beta \frac{{\sin \alpha }}{{\sin \left( {{{60}^o} - \beta } \right)}}\,\,\,\,\,\,\,\,\,(2)}\]
Ta có: $$\sin 3\beta = 3\sin \beta - 4{\sin ^3}\beta = 4\sin \beta \left[ {{{\left( {\frac{{\sqrt 3 }}{2}} \right)}^2} - {{\sin }^2}\beta } \right] =$$
$$=4\sin \beta \left( {{{\sin }^2}{{60}^o} - {{\sin }^2}\beta } \right) = 4\sin \beta \sin \left( {{{60}^o} + \beta } \right)\sin \left( {{{60}^o} - \beta } \right)\,\,\,\,\,\,\,\,\,\,\,(3)$$
Thay $(3)$ vào $(2)$ có: $n = CY = 4d\sin \alpha \sin \beta \sin \left( {{{60}^o} + \beta } \right)$

Lí luận tương tự có: $m = CX = 4d\sin \alpha \sin \beta \sin \left( {{{60}^o} + \alpha } \right)$
Trong $\Delta CXY$, áp dụng định lý hàm số $cos$, ta có:
\[X{Y^2} = {m^2} + {n^2} - 2mn\cos \gamma = \]
\[ = 16{d^2}{\sin ^2}\alpha {\sin ^2}\beta \left[ {{{\sin }^2}\left( {{{60}^o} + \alpha } \right) + {{\sin }^2}\left( {{{60}^o} + \beta } \right) - 2\sin \left( {{{60}^o} + \alpha } \right)\sin \left( {{{60}^o} + \beta } \right)\cos \gamma } \right]\,\,\,\,\,\,\,\,(4)\]
Do $\left( {{{60}^o} + \alpha } \right) + \left( {{{60}^o} + \beta } \right) + \gamma = {180^0}$, nên xét $\Delta EFG\,\,\,\,\,\text{với}\,\,\,\,\widehat E = {60^o} + \alpha ,\widehat F = {60^o} + \beta ,\widehat G = \gamma .$
Gọi $d_1$ là đường kính đường tròn ngoại tiếp tam giác này. Theo định lý hàm số $cos$ trong tam giác này có:
\[{FG = e = {d_1}\sin \left( {{{60}^o} + \alpha } \right) \Rightarrow \sin \left( {{{60}^o} + \alpha } \right) = \frac{e}{{{d_1}}}}\]
\[{EG = f = {d_1}\sin \left( {{{60}^o} + \beta } \right) \Rightarrow \sin \left( {{{60}^o} + \beta } \right) = \frac{f}{{{d_1}}}}\]
\[{EF = g = {d_1}\sin \gamma \Rightarrow \sin \gamma = \frac{g}{{{d_1}}}}\]
Vậy thay vào $(4)$, ta có:
\[{X{Y^2} = 16{d^2}{{\sin }^2}\alpha {{\sin }^2}\beta \frac{{{e^2} + {f^2} - 2ef\cos \gamma }}{{d_1^2}}}\]
\[{ = 16{d^2}{{\sin }^2}\alpha {{\sin }^2}\beta \frac{{{g^2}}}{{d_1^2}} = 16{d^2}{{\sin }^2}\alpha {{\sin }^2}\beta {{\sin }^2}\gamma }\]
\[{ \Rightarrow XY = 4d\sin \alpha \sin \beta \sin \gamma }\]
Do vai trò bình đẳng, ta cũng có \[XZ = ZY = 4d\sin \alpha \sin \beta \sin \gamma \Rightarrow XY = YZ = ZX.\]
Vậy $\Delta XYZ$ là tam giác đều (đpcm).
Chú ý:
$ \bullet $ Frank Morley (1860 – 1937) sinh tại Anh, nhưng hầu như suốt đời sống ở Mĩ. Trong vài chục năm ông là giáo sư toán học ở trường đại học tổng hợp thuộc bang Baltimore. Bản thân học cách chứng minh của ông rất phức tạp. Cách chứng minh ở trên là của nhà toán học Ấn Độ Naranengar tìm ra vào năm 1909. Một nhà toán học Ấn Độ khác là Xachianarian cho cách giải "phi lượng giác" (chỉ dùng đến kiến thức hình học lớp 9)
$ \bullet $ Định lý về đường chia ba góc được Morley tìm ra từ 1899, nhưng mãi đến năm 1914 ông mới công bố cách chứng minh và mở rộng định lý với việc xét không chỉ các đường chia ba góc trong mà cả các đường chia ba góc ngoài của tam giác. Định lý Morley đã hấp dẫn nhiều người, trong đó có nhà toán học Pháp nổi tiếng Henri Lebesgue (1875 – 1941). Năm 1939, Lebesgue công bố chứng minh sơ cấp của định lý này. Ông xét các đường chia ba các góc trong và ngoài của tam giác (có tất cả 12 đường), và đã chứng minh được rằng trong các giao điểm của các đường đó có 27 bộ ba điểm là các đỉnh của tam giác đều.



Còn nữa ...




#335135 Chuyên đề Hệ phương trình

Đã gửi bởi Ban Biên Tập on 13-07-2012 - 09:20 trong Chuyên đề toán THPT

CHUYÊN ĐỀ HỆ PHƯƠNG TRÌNH

Phạm Hùng Vương

Học sinh lớp 12C1 trường THPT Phan Đăng Lưu, Nghệ An


I. Lời nói đầu.

Chuyên đề là kết quả thu được qua một thời gian học tập và nghiên cứu của bản thân về hệ phương trình. Tuy nhiên có thể nói rằng, đó là sự kết tinh qua nhiều thế hệ, là sự giúp đỡ, là sự học hỏi từ những người bạn của mình cũng như rất nhiều yếu tố khác.

Để đạt hiệu quả cao khi tham khảo chuyên đề này, xin được trích dẫn mấy lời của nhà giáo G.Polya:
" [...] Một số bài toán có nêu lời giải đầy đủ (tuy vắn tắt), đối với một số bài khác, chỉ vạch ra mấy bước giải đầu tiên, và đôi khi chỉ đưa ra kết quả cuối cùng.

Một số bài toán có kèm thêm chỉ dẫn để giúp người đọc giải được dễ dàng hơn. Chỉ dẫn cũng có thể nằm trong những bài toán khác ở gần bài toán đang xét. Nên đặc biệt lưu ý đến những nhận xét mở đầu trước từng bài tập hay cả một nhóm bài tập gặp thấy trong chương.

Nếu chịu khó, gắng sức giải một bài toán nào đó thì dù không giải nổi đi chăng nữa, bạn đọc cũng thu hoạch được nhiều điều bổ ích. Chẳng hạn, bạn đọc có thể giở ra xem (ở cuốn sách) phần đầu mỗi lời giải, đem đối chiếu với những suy nghĩ của bản thân mình, rồi gấp sách lại và thử gắng tự lực tìm ra phần còn lại của lời giải.

Có lẽ thời gian tốt nhất để suy nghĩ, nghiền ngẫm về phương pháp giải bài toán là lúc bạn vừa tự lực giải xong bài toán hay vừa đọc xong lời giải bài toán trong sách, hay đọc xong phần trình bày phương pháp giải trong sách. Khi vừa hoàn thành xong nhiệm vụ, và các ấn tượng hãy còn "nóng hổi", nhìn lại những nổ lực vừa qua của mình, bạn đọc có thể phân tích sâu sắc tính chất của những khó khăn đã vượt qua. Bạn đọc đọc có thể tự đặt cho mình nhiều câu hỏi bổ ích: "Khâu nào trong quá trình giải là quan trọng nhất? Khó khăn chủ yếu là ở chỗ nào? Ta có thể làm gì cho tốt hơn? Chi tiết ấy mình cũng đã liếc qua mà không chú ý đến - muốn "nhìn thấy" chi tiết này thì đầu óc phải có tư chất ra sao? Liệu ở đây có một cách gì đó đáng lưu ý để sau này gặp một tình huống tương tự, ta có thể áp dụng được không?" Tất cả những câu hỏi đó đều hay cả, và cũng còn nhiều câu hỏi bổ ích khác nữa, nhưng câu hỏi hay nhất chính là câu hỏi tự nhiên nảy ra trong óc, không cần ai gợi ý cả!"




(trích "Mấy lời khuyên và chỉ dẫn" -G.Polya trong "Sáng tạo toán học")


Do thời gian cũng như 1 số vấn đề khác như kiến thức, trình bày,.. mà chuyên đề này còn khá nhiều khiếm khuyết. Rất mong được các bạn quan tâm và chia sẻ đề hoàn thiện chuyên đề hơn. Hi vọng nó sẽ là tài liệu bổ ích giúp chúng ta vượt qua 1 chẳng nhỏ trong chặng đường chinh phục toán học.

II. MỘT SỐ PHƯƠNG PHÁP CŨ.

1. Hệ phương trình đối xứng kiểu I.

Nhận dạng: Hệ đối xứng kiểu I: gồm 2 phương trình ẩn x,y mà vai trò x,y trong mỗi phương trình là như nhau. Ví dụ: $\left\{\begin{matrix} a(x+y)+bxy=c\\x^2+y^2=c \end{matrix}\right.$. Và phương pháp giải là đặt ẩn phụ: $S = x + y, P = xy$. Giải tìm $S, P$ sau đó sử dụng định lí Vi-et, dễ thấy $x,y$ là nghiệm của phương trình: $X^2 - S.X + P = 0$
Cùng xem xét 1 vài ví dụ (cách giải và một số hướng giải quyết mới)

Ví dụ 1: (Đề thi HSG lớp 9 Tỉnh Bến Tre năm 2009-2010)
Giải hệ phương trình: $\left\{\begin{matrix}x^2+y^2-2x-2y=6 \\x+y-xy=5 \end{matrix}\right.$

Bài giải: Đặt $S = x + y, P = xy$, ta thu được hệ mới tương đương:
$$\left\{\begin{matrix} S^2-2P-2S=6\\S-P=5 \end{matrix}\right. \Leftrightarrow \left\{\begin{matrix} S^2-4S+4=0\\ P=S-2 \end{matrix}\right.\Leftrightarrow \left\{\begin{matrix} S=2\\P=-3 \end{matrix}\right.$$
Như vậy, theo định lí Vi-ét, $x,y$ là nghiệm của phương trình:
$${X^2} - 2X - 3 = 0 \Leftrightarrow (X - 3)(X + 1) = 0 \Rightarrow \left[ \begin{array}{l}
x = 3,y = - 1\\
x = - 1,y = 3
\end{array} \right.$$
Vậy hệ có 2 nghiệm $(x;y)$ thỏa mãn là: $(-1;3)$ và $(3;-1)$.

Những bài như thế này và bài giải như vậy đã trở nên quen thuộc, không còn mới lạ. Tuy nhiện, cũng có 1 số bài hệ, dù biết là đối xứng kiểu I, nhưng lại phải làm gì để sử dụng được? Hãy xem ví dụ:

Ví dụ 2: (ĐH-CĐ Khối A năm 2006)
Giải hệ phương trình: $\left\{ \begin{matrix} x+y-\sqrt{xy}=3\\\sqrt{x+1}+\sqrt{y+1}=4 \end{matrix}\right. $

Bài giải:

Ý tưởng 1: Thử đặt như cũ: $S=x+y, P=xy$, hệ khi đó trở thành:
$$\left\{ \begin{matrix} x+y-\sqrt{xy}=3\\ x+y+2+\sqrt{xy+x+y+1}=16 \end{matrix}\right. \Leftrightarrow \left\{ \begin{matrix} S-\sqrt{P} =3\\ S+2\sqrt{P+S+1}=14 \end{matrix}\right.$$
$$\Leftrightarrow \left\{ \begin{matrix} S=\sqrt{P}+3\\2\sqrt{P+\sqrt{P}+4}=11-\sqrt{P} \end{matrix}\right. \Leftrightarrow \left\{ \begin{matrix} S=\sqrt{P}+3\\3P-26\sqrt{P}-105=0 \\ 0 \leq P \leq 121 \end{matrix}\right. $$
Đến đây, giải tìm $P$, sau đó quay lại giải tìm ra nghiệm $x,y$. ( chú ý điều kiện)

Hơn nữa, luôn nhớ: $S^2 \geq 4P$ để loại bớt nghiệm.

Ý tưởng 2: Đặt ẩn $a =\sqrt{x +1},b =\sqrt{y+1}$ nhằm làm đơn giản 1 phương trình của hệ. (kĩ thuật đặt ẩn làm gọn này rát có ý nghĩa, đặc biệt trong bất đẳng thức (BĐT) có giả thiết rườm rà, với phương trình hay hệ cũng vậy). Khi đó:
$$HPT \Leftrightarrow \left\{ \begin{matrix} a+b=4\\a^2+b^2-2-\sqrt{(a^2-1)(b^2-1)}=3 \end{matrix}\right. \Leftrightarrow \left\{ \begin{matrix} S=4\\S^2-2P-2-\sqrt{P^2-S^2+2P+1}=3 \end{matrix}\right. \Leftrightarrow \left\{ \begin{matrix} S=4\\\sqrt{P^2+2P-15}=11-2P \end{matrix}\right. $$
Trong đó $S=a+b, P=ab$. Đến đây, ta cũng có thể giải tương tự.

Ví dụ 3: (Thi thử ĐH-CĐ, THPT chuyên Nguyễn Huệ 2011)
Giải hệ phương trình: $\left\{ \begin{matrix} \sqrt{x+1}+\sqrt{y-1}=4\\\sqrt{x+6}+\sqrt{y+4}=6 \end{matrix}\right.$

Ví dụ 4:(Đề thi HSG lớp 9 tỉnh Nghệ An năm 2009-2010)
Giải hệ phương trình: $\left\{ \begin{matrix} \frac{1}{x}+\frac{1}{y}+\frac{1}{z}=2\\ \frac{2}{xy}-\frac{1}{z^2}=4 \end{matrix}\right.$

Ví dụ 5: Giải hệ phương trình: $\left\{ \begin{matrix} (x+y)(1+xy)=4xy\\(x^2+y^2)(1+x^2y^2)=4x^2y^2 \end{matrix}\right.$

Thực ra, dạng hệ đối xứng kiểu I có hướng giải khá đơn giản, rõ ràng với việc đặt ẩn và sử dụng định lí Vi-ét. Chính vì vậy mà hệ đối xứng kiểu I thường gắn với việc giải và biện luận, một sở trường của phương pháp này! Chúng ta cùng xét một số ví dụ sau.

Ví dụ 6: (Đề thi HSG lớp 9 tỉnh Hà Nội năm 2009-2010)
Tìm $a$ để hệ phương trình sau có nghiệm duy nhất: $\left\{ \begin{matrix} ay+x+y=a+1\\x^2y+xy^2=a \end{matrix}\right.$

Bài giải: Đặt : $S = x + y, P = xy$, ta có hệ mới: $\left\{ \begin{matrix} S+P=a+1\\SP=a \end{matrix}\right.$

Theo Vi-ét, $S$ và $P$ là nghiệm của phương trình: $X^2-(a +1)X + a = 0 (1)$

Hơn nữa, cũng theo Vi-ét $x,y$ lại là nghiệm của phương trình: $X^2 -S.X + P = 0 (2).$

Do đó, để hệ có 1 nghiệm duy nhất thì $(2)$ có nghiệm duy nhất, tức $\Delta _{(2)} = 0 \Leftrightarrow S^2 = 4P \Leftrightarrow x = y$ Hoặc có thể dùng nhận xét: do vai trò $x, y$ trong mỗi phương trình của hệ là như nhau nên nếu hệ có nghiệm $(m; n)$ thì nó cũng có nghiệm $(n; m)$. Như vậy để hệ có nghiệm duy nhất thì cần có $x = y$. Thế vào được:
$$\left\{ \begin{matrix} x^2+2x=a+1\\2x^3=a \end{matrix}\right. \Leftrightarrow \left\{ \begin{matrix} x^2+2x-(a+1)=0 \,\,\,\,\,\,(*)\\x=\sqrt[3]{\frac{a}{2}} \end{matrix}\right. $$
Để hệ có nghiệm duy nhất thì $(*)$ có duy nhất 1 nghiệm $x=\frac{-2}{2.1}=-1\Rightarrow \sqrt[3]{\frac{a}{2}}=-1\Leftrightarrow a=-2.$

Thử lại thấy thỏa mãn. Kết luân giá trị cần tìm là $a=-2.$

Ví dụ 7: (Đề thi HSG lớp 9 Tỉnh Hưng Yên năm 2009-2010)
Tìm $m$ để hệ phương trình sau có nghiệm: $\left\{ \begin{matrix} \sqrt{x}+\sqrt{y}=m\\x+y-\sqrt{xy}=m \end{matrix}\right.$

Ví dụ 8: (Thi thử ĐH-CĐ THPT Lương Ngọc Quyến, Thái Nguyên 2011)

Tìm $a$ để hệ phương trình sau có nghiệm: $\left\{ \begin{matrix} \sqrt{x+1}+\sqrt{y+1}=a\\x+y=2a+1 \end{matrix}\right.$

Nếu đơn thuần chỉ là hệ đối xứng kiểu I thì chắc chắn nó sẽ nhanh chóng được chúng ta giải quyết. Chính vì vậy, mà sau đây sẽ các ví dụ cần dùng các kĩ thuật nhỏ chuyển về hệ đối xứng kiểu I. (Phần kĩ năng sẽ trình bày rõ hơn ở mục sau).

Ví dụ 9: Giải hệ phương trình: $\left\{ \begin{matrix} (x-1)^2+6(x-1)y+4y^2=20\\x^2+(2y+1)^2=2 \end{matrix}\right.$

Nhận xét: Quan sát thì thấy ngay không thể là hệ đối xứng kiểu I. Nhưng! Hãy xem hướng giải sau:

Bài giải: Đặt $a = x -1, b = 2y$ thì hệ trở thành: $\left\{ \begin{matrix} a^2+3ab+b^2=20\\(a+1)^2+(b+1)^2=2 \end{matrix}\right.$. Đúng là hệ đối xứng kiểu I!

Bây giờ, thì có thể đi tiếp theo phương pháp được rồi.

Có thể nói rằng, vấn đề đặt gọn luôn ẩn hiện 1 điều gì đó rất thú vị nếu ta tinh ý trong các biểu thức nhìn có vẻ có vấn đề. Như ví dụ trên chẳng hạn, phải biết nghi ngờ $(x -1)$, khi nó được đặt trong ngoặc.

Hãy tiếp tục với hệ sau:

Ví dụ 10: Giải hệ phương trình: $\left\{ \begin{matrix} x^2+6xy+4y^2=19+2y+6y \,\,\,\,\,\,(1) \\ x^2+4y^2=1-4y \,\,\,\,\,\,\,\, (2) \end{matrix}\right.$

Có thể thấy, cả 2 ví dụ 10 và ví dụ 11 đều chỉ là một. Nhưng nếu nghiệm theo cách đặt ẩn gọn thì đặt cái nào.

Nếu đặt $a = x-1,\,\,b = 2y$ như trên thì tại sao lại biết mà đặt như vậy. Đây chính là vấn đề cần bàn. Nếu đi theo phân tích phương trình $(1)$ thì sẽ có khá nhiều phương án: chẳng hạn nghĩ đến hằng đẳng thức: $(1) \Leftrightarrow (x + 3y)^2 -5y2 = 19 + 2(x + 3y), ..v.v$. Có khá nhiều đẳng thức có thể nghĩ tới để đặt.

Nhưng với phương trình $(2)$ thì lại khác: nó chỉ có một đằng thức cần chú ý: $(2) \Leftrightarrow x^2 + (2y +1)^2 = 2.$ Như vậy, ý tưởng đặt làm gọn $(2)$ mở ra: $a = 2 y +1$, hơn nữa có thể thấy ở phương trình $(1)$ hệ số của $y$ luôn chẵn, khi thế có thể thế $2y = a -1$ (đây không phải là một trùng hợp ngẫu nhiên. Hãy nghĩ vậy).

Việc làm còn lại thì khá rõ rồi, ta cũng thu được một hệ đối xứng kiểu I và tiếp tục giải.

Hãy thử với các ví dụ:

Ví dụ 11: Giải hệ phương trình: $\left\{ \begin{matrix} x^4-4x^2+(y-3)^2=0\\x^2y+x^2+2y-22=0 \end{matrix}\right.$

Ví dụ 12: Giải hệ phương trình: $\left\{ \begin{matrix} (x-y)^2=1-x^2y^2\\x(xy+y+1)=y(xy+1)+1 \end{matrix}\right.$

Thậm chí còn có những bài có những cách đặt đưa về hệ đối xứng rất thú vị, khó mà thấy được nếu không qua chút biến đổi. Vì vậy, hãy cố gắng quan sát và đặt gọn phù hợp.

Ví dụ 13: Giải hệ phương trình: $\left\{ \begin{matrix} x^2+y^2=5+4x-4y\\3x+xy-y=15 \end{matrix}\right.$

Ví dụ 14: (THTT số 379 năm 2009)
Giải hệ phương trình: $\left\{ \begin{matrix} xy-3x-2y=16\\x^2+y^2-2x-4y=33 \end{matrix}\right.$

Ví dụ 15: Giải hệ phương trình: $\left\{ \begin{array}{l}
{x^2} + {y^2} = 2\\
2{x^2} + 3xy - 2{y^2} + 3x + y = 7
\end{array} \right.$ (xem giải ở mục II. phương pháp 02)

Hơn nữa, dạng hệ đối xứng kiểu I này rất hay vận dụng một hằng đẳng thức (đang có xu hương lớn trong các đề thi thử): $\frac{1}{x^2}+y^2=(\frac{1}{x}+y)^2-2 \frac{y}{x}.$. Tiếp tục với các ví dụ sau, bạn sẽ thấy rõ.

Ví dụ 16: Giải hệ phương trình: $\left\{ \begin{matrix} \frac{y}{x^2}+\frac{y^2}{x}=6\\\frac{1}{x^2}+y^2=5 \end{matrix}\right.$

Ví dụ 17: Giải hệ phương trình: $\left\{ \begin{matrix} xy+y^2+x-7y=0\\xy+x^2-12y=0 \end{matrix}\right.$

Ví dụ 18: Giải hệ phương trình: $\left\{ \begin{matrix} (x^2+y^2)(1+\frac{1}{xy})^2=9\\(x^3+y^3)(1+\frac{1}{xy})^3 =4 \end{matrix}\right.$

Ví dụ 19: Giải hệ phương trình: $\left\{ \begin{matrix} xy(2x+y-6)+2x+y=0\\(x^2+y^2)(1+\frac{1}{xy})^2=8 \end{matrix}\right.$

Và cả một dạng (ở phần cuối chuyên đề)

2. Hệ phương trình đối xứng kiểu II.

Nhận dạng: Cũng như loại I, loại II cũng “đối xứng” nhưng là đối xứng giữa 2 phương trình chứ không không phải là đối xứng trong từng phương trình như kiểu I.

Một cách nhận dạng khác nữa là cho $x = y$ thì 2 phương trình của hệ như nhau. Hay nói cách khác $x = y$ chính là nghiệm của hệ. Đây chính là đặc điểm khai thác của hệ này.

Phương pháp: Thông thường, ta trừ theo vế ta thu được nghiệm $x = y$, và 1 số nghiệm khác. Sau đó thay lại tìm ra nghiệm $(x; y)$.

Cùng xem xét một số ví dụ đơn giản.
Ví dụ 20: Giải hệ phương trình: $\left\{ \begin{matrix} x^2+y=5x+3\\y^2+x=5y+3 \end{matrix}\right.$

Bài giải: Trừ theo của hệ thu đươc: $x^2 -y^2 = 6(x-y) \Leftrightarrow (x-y)(x + y-6) = 0$. Do đó, hệ phương trình đã cho tương đương với:

\[\left[ \begin{array}{l}
\left\{ \begin{array}{l}
x = y\\
{x^2} - 4x + 3 = 0
\end{array} \right.\\
\left\{ \begin{array}{l}
x + y = 6\\
{x^2} + x - 5\left( {6 - x} \right) = - 3
\end{array} \right.
\end{array} \right. \Leftrightarrow \left[ \begin{array}{l}
\left[ \begin{array}{l}
x = y = 1\\
x = y = 3
\end{array} \right.\\
\left[ \begin{array}{l}
x = 3,y = 3\\
x = - 9,y = 15
\end{array} \right.
\end{array} \right. \Leftrightarrow \left[ \begin{array}{l}
x = y = 1\\
x = y = 3\\
x = - 9,y = 15
\end{array} \right.\]
Vậy hệ phương trình đã cho có nghiệm là $\left( {x;y} \right) = \left\{ {\left( { - 9;15} \right),\left( {1;1} \right),\left( {3;3} \right)} \right\}$

Ví dụ 21: Giải hệ phương trình: $\left\{ \begin{matrix} x^3=2y+1\\y^3=2x+1 \end{matrix}\right.$

Bài giải: Trừ theo vế của hệ ta thu được: $x^3 - y^3 = 2 (y - x) \Leftrightarrow (x - y)(x^2 + xy + y^2 + 2) = 0 \Leftrightarrow x = y$

Vì $x^2 + xy + y^2 + 2 = (x+\frac{y}{2})^2+\frac{3}{4}y^2+2>0$. Như vậy thế $x = y$ vào hệ, ta chỉ cần giải phương trình:

\[{x^3} - 2x + 1 = 0 \Leftrightarrow \left( {x - 1} \right)\left( {{x^2} + x - 1} \right) = 0 \Leftrightarrow \left[ \begin{array}{l}
x = 1\\
{x^2} + x - 1 = 0
\end{array} \right. \Leftrightarrow \left[ \begin{array}{l}
x = 1\\
x = \frac{{ - 1 \pm \sqrt 5 }}{2}
\end{array} \right.\]

Vậy hệ phương trình đã cho có nghiệm là $x = \left\{ {\frac{{ - 1 \pm \sqrt 5 }}{2};1} \right\}$

$ \bullet $ Chú ý: Khi trừ theo vế, ta thu đươc: $x^3 + 2 x = y^3 + 2 y$. Nếu không dùng phân tích trên, ta có thể tính đạo hàm: $f (t) = t^3 + 2t$ có $f'(t) = 3t^2 + 2 > 0$ nên suy ra: $x = y .$

$ \bullet $ Nhận xét: Đơn giản chỉ là trừ vế theo vế, nhưng với những bài khác nhau lại cần thêm những kĩ thuật khai thác khác nhau để là xuất hiện $(x – y)$. Hãy xem:

Ví dụ 22: (Thử sức trước kì thi, THTT số 407, 2011)
Giải hệ phương trình: $\left\{\begin{matrix} \sqrt{x+10}+\sqrt{y-1}=11 & \\\sqrt{y+10}+\sqrt{x-1}=11 & \end{matrix}\right.$

Bài giải: Điều kiện các phân thức có nghĩa: $x,y\geq 1$. Chú ý $x=y=1$ không là nghiệm của hệ nên trừ theo vế 2 phương trình của hệ và nhân lượng liên hợp ta có:
\[\sqrt {x + 10} - \sqrt {y + 10} + \sqrt {y - 1} - \sqrt {x - 1} = 0\]
\[ \Leftrightarrow \left( {x - y} \right)\left( {\frac{1}{{\sqrt {x + 10} + \sqrt {y + 10} }} - \frac{1}{{\sqrt {x - 1} }} + \frac{1}{{\sqrt {y - 1} }}} \right) = 0 \Leftrightarrow x = y\]
(Vì do $\sqrt{x+10}+\sqrt{y+10}>\sqrt{x-1}+\sqrt{y-1}$ nên biểu thức còn lại vô nghiệm).

Thế $x = y$ vào ta dễ dàng giải phương trình của hệ.

Còn tiếp...




#332038 Toán học trong bầu cử

Đã gửi bởi Ban Biên Tập on 05-07-2012 - 07:52 trong Toán học lý thú

BBT không đăng lại phần 3 vì phần này tác giả trình bày quan điểm chính trị của cá nhân.
Hệ thống bầu cử 1 lần có chuyển phiếu (single transferable vote, viết tắt là STV) để bầu cử một ai hay một cái gì đó, khi mà có ít nhất 3 ứng cử viên, như sau:
* Mỗi lá phiếu ghi tên (hay đánh dấu) các ứng cử viên theo thứ tự lựa chọn của cử tri: dòng đầu tiên là người mà cử tri thích nhất, dòng thứ hai là người mà cử tri muốn bầu nếu người mà mình thích nhất không được bầu, và cứ thế.(Không nhất thiết phải ghi tên toàn bộ các ứng cử viên, nếu những người nào mà cử tri hoàn toàn không thích bầu thì không cần cho vào danh sách).
Hình đã gửi
* Thuật toán bầu như sau:
- Đầu tiên đếm số phiếu của các ứng cử viên theo dòng thứ nhất (số cử tri đặt ứng cử viên lên hàng đầu). Nếu có ai đạt trên 50% số phiếu thì thắng, và việc bầu cử kết thúc. Nếu không ai đạt 50%, thì loại đi người đạt ít phiếu nhất trong lần đếm đầu tiên này, và chuyển sang lần đếm thứ hai.
- Ở lần đếm thứ hai, gạch tên ứng cử viên đã bị loại ra khỏi các lá phiếu. Ví dụ, nếu ứng cử viên bị loại đứng hàng đầu ở một lá phiếu nào đó, thì bây giờ ứng cử viên đứng hàng thứ 2 ở lá phiếu đó được chuyển lên thành hàng đầu , ứng cử viên đứng hàng thứ 3 được chuyển lên thành hàng 2, và cứ thế. Sau khi gạch tên ứng cử viên đã bị loại như vậy, thì lặp lại quá trình đếm: nếu có ứng cử viên nào đứng ở hàng đầu ở trên 50% số phiếu thì được bầu, còn nếu không thì loại đi ứng cử viên có số phiếu hàng đầu ít nhất, rồi tiếp tục như trên.


Hình đã gửi
Hệ thống bầu cử 1 lần có chuyển phiếu trên, và các dạng tương tự của nó, xuất hiện từ thế kỷ 19, và ngày nay nó được dùng trong các cuộc bầu cử ở khá nhiều nơi trên thế giới, trong đó có Úc, Anh, Ấn Độ, Hồng Kông, v.v. Giải Oscar về điện anh cũng được bầu theo STV. Hệ thống STV còn được biết đến với các tên gọi khác như: Instant runoff voting, alternative vote.
Hệ thống STV có nhiều điểm ưu việt rõ rệt so với hệ thống “simple plurality” (không cần quá bán mà chỉ cần số phiếu nhiều hơn các ứng cử viên khác để được bầu ngay vòng đếm phiếu đầu tiên) hiện còn được dùng ở nhiều nơi, và hệ thống bầu cử 2 vòng ở Pháp. Hệ thống “simple plurality” quá là rởm rít trong trường hơp có nhiều ứng cử viên, không chấp làm gì. So với hệ thống bầu hai vòng ở Pháp, thì hệ thống STV có các ưu điểm sau:
* Cử tri chỉ cần đi bầu 1 vòng, thay vì 2 vòng. Tổ chức bầu 2 vòng tốn kém về thời gian (có khi mất thêm cả tháng) và tiền bạc (tính theo đơn vị trăm triệu USD) so với là chỉ 1 vòng.
* Trong trường hợp có nhiều ứng cử viên, thì việc chỉ chọn 2 người vào vòng 2 nhiều khi cũng éo le chẳng kém gì việc để người được nhất vòng 1 thắng ngay. Đây là điều đã xảy ra trong bầu cử tổng thống Pháp năm 2002. Nếu như Pháp dùng hệ thống STV đã không xảy ra sự éo le đó.
* Hệ thống STV khiến cho người ta bầu thật sự theo suy nghĩ của mình hơn là hệ thống 2 vòng. Ở Pháp, người ta phải kêu gọi “voter utile” vòng 1 (tức là không bầu cho người mình thực sự thích nhất, mà bầu cho người mình không thích lắm nhưng có nhiều khả năng trúng cử nhất trong số các ứng cử viên còn lại mà mình thấy tạm được) để tránh khỏi các tình huống éo le khi có nhiều ứng cử viên. Nhưng kiểu “voter utile” đó là một thứ phản dân chủ, khi các cử tri (hay các đảng phái) bỏ phiếu ngược lại ý nguyện thực sự của mình hòng thao túng kết quả bầu cử.
Hệ thống STV chưa phải là “hoàn hảo”. Nó không thỏa mãn một số tính chất quan trọng, trong đó có tính chất đơn điệu (monotonicity) sau: nếu 1 cử tri tăng thứ tự lựa chọn 1 ứng cử viên nào đó lên trong lá phiếu bầu của mình, thì điều đó không thể làm hại ứng cử viên đó, mà chỉ có thể hoặc không ảnh hưởng gì hoặc làm tốt lên cho ứng cử viên đó. Ví dụ đơn giản sau cho thấy, trong một số trường hợp nào đó dùng STV , có thể làm hại một ứng cử viên bằng cách nâng anh ta lên:
100 người bầu cho 3 ứng cử viên A,B,C, với kết quả các là phiếu như sau:

36 ABC


(tức là 36 người chọn A hàng đầu, sau đó đến chọn B, và xếp C là phương án tồi nhất)



34 BCA
30 CAB


Trong lần đếm phiếu đầu tiên thì C bị loại (chỉ có 30 phiếu, ít nhất). Lần đếm thứ hai thì A thắng (được 66 phiếu, trong khi B vẫn chỉ được 34 phiếu)
Nay giả sử có 5 người thay vì chọn BCA lại chọn thành ABC, tức là nâng A từ thứ ba lên thứ nhất trong các lá phiếu của họ, kết quả sẽ thành


41 ABC
29 BCA
30 CAB


Nếu có 5 người nâng A lên như vậy, thì B bị loại trong lần đếm đầu, và A thua trong lần đếm thứ 2, và C thắng chứ không phải A thắng !
Theo định lý Gibbard-Satterthwaite thì thực ra không có một hệ thống bầu cử dân chủ nào là có thể hoàn toàn tránh khỏi lũng đoạn, nên ví dụ trên có lẽ không đáng ngạc nhiên lắm. Tuy nhiên, có các công trình cho thấy, lũng đoạn bầu cử trong hệ thống STV là vấn đề “NP-hard”, tức là trên thực tế thì không đáng sợ lắm chuyện người ta không thật lòng khi bầu cử theo hệ thống STV, xem: http://www.cs.duke.e...09/stv_hard.pdf. Bởi vậy, hệ thống STV có thể coi là khá tốt để chống lại các trò “strategic voting”.
Trong trường hợp mà cuộc bầu cử có nhiều người chứ không chỉ 1 người được bầu (ví dụ như là bầu vào quốc hội), thì các thuật toán của các hệ thống bầu cử STV không những cho phép chuyển phiếu từ các ứng cử viên đã bị loại sang các ứng cử viên mà cử tri chọn lựa tiếp theo, mà nó còn có thể cho phép chuyển bớt phiếu từ các ứng cử viên đã được bầu mà thừa phiếu để được bầu sang các ứng cử viên “cùng phe” khác. Thuật toán chuyển phiếu thừa này cho phép hệ thống bầu cử STV gần đạt tính chất tỷ lệ thuận (proportional, tức là nếu đảng phái nào hay nhóm nào có tỷ lệ bao nhiêu % cử tri ủng hộ, thì cũng có tỷ lệ gần như vậy người được bầu) hơn hẳn so với hệ thống bầu cử quốc hội 2 vòng ở Pháp hiện tại.



#331702 Laurent Schwartz - Người bạn lớn của VN .

Đã gửi bởi Ban Biên Tập on 04-07-2012 - 00:42 trong Các nhà Toán học

BBT: Đây là bài viết của GS Nguyễn Duy Tiến gửi cho Diễn đàn toán học Tưởng niệm 10 năm ngày mất GS LAURENT SCHWARTZ (4/7/2002-4/7/2012)

NGƯỜI VIỆT NAM CHÚNG TÔI MÃI MÃI BIẾT ƠN ÔNG, GIÁO SƯ LAURENT SCHWARTZ
(Tưởng niệm 10 năm ngày mất GS LAURENT SCHWARTZ
4/7/2002-4/7/2012)

Hình đã gửi

GS LAURENT SCHWARTZ (05/03/1915 - 04/07/2002)



Trong hồi ký của giáo sư Laurent Schwartz có câu “Les Vietnamiens ne m’ou- blient pas” (Người Việt Nam không quên tôi). Đúng thế, những người Việt Nam chúng tôi không bao giờ quên Ông, Giáo sư Laurent Schartz nhà toán học vĩ đại, người đã tới thăm và giảng bài Việt Nam 3 lần vào những năm tháng khó khăn nhất của dân tộc ta.

Lần thứ nhất vào cuối năm 1968 với tư cách là thành viên của Tòa án Betrand Russell "xử tội ác đế quốc Mỹ trong cuộc chiến tranh với Việt Nam". Ông được Thủ Tướng Phạm Văn Đồngtiếp 3 tiếng, và được nói chuyện với Hồ Chủ Tịch 1 tiếng (một trường hợp hiếm có). Vì thời gian có hạn, nên Ông chỉ có một buổi làm việc với giới toán học Việt Nam. Hôm đó Ông nêu một nhận xét trong phương trình đạo hàm riêng. Tôi nhớ là Ông nói "nhận xét đơn giản này mở ra cho một hưóng mới của toán học hiện đại". Tôi không hiểu gì.

Sau đó, Ông đã ủng hộ 10.000 quan cho Việt Nam khi bệnh viện Bạch Mai bị máy bay Mỹ ném bom.

Lần thứ hai vào mùa xuân 1976, Ông giảng (trong 3 tuần liền (1), giáo sư Phan Đức Chính dịch) về ánh xạ Radon hóa (Radonifying maps) và một ít về Hình Học các không gian Banach (Geometry of Banach Spaces). Tôi thực sự may mắn, vì

1) Sau khi bảo vệ tiến sĩ (26/04/1974), trước thời hạn 18 tháng, với luận án "Một số vấn đã về xác suất trong không gian Banach", tôi được tiêp tục ở lại Tbilisi làm việc với nhóm nghiên cứu của giáo sư N. N. Vakhania (thầy giáo hưng dẫn luận án tiến sĩ của tôi). Có thể nói, Tbilisi là trung tâm toán học nổi tiếng toàn thế giới về hàm phức và cơ học với những nhà toán học lừng danh như Mushkhelisvily, Vekua (thầy của giáo sư Ngô Văn Lực), Bissadze (thầy của giáo sư Nguyễn Thừa Hợp). Nhưng vào những năm 60, ở Tbilisi xác suất và thống kê thì còn yếu. Nhóm nghiên cứu của giáo sư Vakhania là nhóm mới nổi lên nhờ có quan hệ tốt với trường phái xác suất của Mockva, đc biệt là được sự hỗ trợ của Yu. V. Prokhorov, V.V Sazonov. Lúc tôi tới Tbilisi (5/10/1971), Vakhania mới bảo vệ tiến sĩ khoa học được 2 năm, và S. Chobanyan mới bảo vệ tiến sĩ được 1 năm (dưi sự hưng dẫn của Vakhania). Seminar của chúng tôi (gồm có Vakhania, Chobanyan, V. Tarieladze, Z. Gorgadze, V. Kvaratskhelia, tôi, và sau đã có thêm A. Weron từ Ba Lan sang thực tập) có tên rất hấp dẫn là "Giải tích hàm và lý thuyết xác suất" hoạt động rất tích cực. Có thể nói, tôi đã gặp may khi được làm việc theo nhóm như thế, đặc biệt là làm việc với ChobanyanTarieladze (cả hai sau này đều là những chuyên gia hàng đầu về xác suất trong không gian tuyến tính). Nhưng chúng tôi không ai biết tiếng Pháp, vì thế tôi quyết định học tiếng Pháp để đọc tài liệu, đặc biệt là những bài đăng trong Comptes Rendues của B. Maurey, G. Pisier, Fernique.

Tôi đang nghiên cứu về Hình học các không Banach và thu được một số kết quả theo hướng này, thì đất nước Việt Nam chiến thắng (30/04/1975) và thống nhất. Nửa năm sau (5/10/1975), tôi có mặt tại Hà Nội. Thế rồi, Dacuna Casten (giáo sư Toán học, người Pháp, chuyên ngành về xác suất và thống kê) sang Hà Nội giảng bài (giáo sư Nguyễn Hữu Anh dịch) và đã cập tới không gian lồi đều và trơn đều. Tôi rất hiểu bài giảng của ông và hỏi ông nhiều câu hỏi liên quan đến hình học Banach. Lần đầu tiên tôi được ông cho tôi biết kết quả của Pisier (tuyệt vời) về hình học Banach và ứng dụng vào lý thuyết Martingales Tháng 3/1976, giáo sư Laurent Schwartz đến Hà Nội, thì tôi đã có đã kiến thức nghe và hiễu các bài giảng của ông. Nội dung chính của các bài giảng này là trinh bày các kết quả của seminar Maurey-Schwartz. Ý tưởng của seminar này băt nguồn từ những kết quả của S. Kwapien: ứng dụng lý thuyết toán tử khả tổng tuyệt đại (absolutly summing operators) của Pietsch vào nghiên cứu xác suất. Cụ thể như sau: cho $X,Y$ là hai không gian Banach, và $\mu $ là độ đo xác suất trụ trong $X$ . Giả sử $T:X\rightarrow Y$ là toán tử tuyến tính, liên tục. Khi nào độ đo ảnh $T(\mu)$ là độ đo Radon trên $Y$ ?
Chẳng hạn, khi $X = Y = H$ là không gian Hilbert và $\mu =\gamma$ là độ đo trụ Gauss chuẩn tắc trên $H$, tức là, phiếm hàm đặc trưng có dạng:
$$\widehat{\gamma }=\exp\left (-\frac{\left \| h \right \|^2}{2} \right ),h\in H$$
thì định lý Muorier-Sazonov khẳng định rằng điều kiện cần và đủ để $T(\mu)$ là độ đo Radon là $T$ là tóan tử Hilbert-Schmidt hay tương đương $T$ là toán tử khả tổng tuyệt đại (tức là T biến một dãy khả tổng tuyệt đại yếu thành một dãy khả tổng tuyệt đại mạnh).

Tôi còn nhớ, dù trời nóng, nhưng Ông (lúc đã đã 61 tuổi) giảng bài rất say sưa, rất rõ ràng, khúc triết tại giảng đường C1 của đại học Bách Khoa Hà Nội có tới 100 người nghe. Sau này các bài giảng của Ông được giáo sư Nguyễn Đình Trí và giáo sư Phan Đức Chính ghi chép lại và được in thành sách (bằng tiếng Pháp): Radonifying maps. Thầy Nguyễn Bác Văn và tôi được trực tiếp làm việc với Ông nhiều buổi (lúc thì ở hội trường Bách Khoa, lúc thì ở số 9 Hai Bà Trưng, và cả nơi vợ chồng Ông Bà nghỉ tại khách sạn Metropol Hà Nội. Tôi không nói được tiếng Anh và tiếng Pháp, thành thử thầy Văn đã phiên dịch (rất chính xác) cho tôi. Trong khi tôi trình bày kết quả mới của tôi về Phiễm hàm tuyến tính đo được trong không gian Banach với độ đo Gauss, Ông hỏi tôi đến từng chi tiết nhỏ nhất. Mắt Ông sáng ngời và rất thích kết quả này và Ông nói như khuyên nhủ tôi: "Anh nên học tiếng Pháp rồi sang Pháp làm việc trong seminar của tôi. Anh sẽ được học bổng với số tiền khiêm tốn là 2.500 quan". Tôi còn đặt ra 16 vấn đã về độ đo trong không gian vector topo. Ông chăm chú nghe và cùng tôi thử giải quyết một số vấn đề. Ông làm việc say sưa quên cả giờ giấc, quên cả ăn tối. Lúc bấy giờ tôi hiểu ra rằng "toán học đầy ắp trong Ông", và khi có vấn đã thì từ bộ não của người được giải Fields (lúc 35 tuổi) các ý tưởng toán học tuôn chảy ra mãnh liệt và liên tục. Thỉnh thoảng ông lại hỏi "Anh có hiểu không?"

2) Tôi đã tìm được người hợp tác khoa học, đó là Đặng Hùng Thắng. Có thể nói rằng các bài giảng của giáo sư Laurent Schwartz là cầu nối tôi với sinh viên xuất sắc Đặng Hùng Thắng. Một hôm anh Thắng gặp tôi và trình bày với tôi một khái niệm mới: giá của độ đo xác suất trụ và đặt câu hỏi: giá trụ và giá của độ đo có khi nào trùng nhau không? Tôi thấy câu hỏi rất thú vị. Sau vài ngày suy nghĩ tôi đã tìm được câu trả lời, trong trường hợp giá là tập lồi thì giá trụ và giá của độ đo trùng nhau vì giá trụ là giá của độ đo trong topo yếu. Đặc biệt, điều này Đúng đối với đo đo Gauss, vì giá của độ đo Gauss đối xứng là không gian con. Sau đã tôi đã cho anh Thắng đọc chứng minh của tôi về kết quả này, anh nhận xét rằng chứng minh ấy mở rộng được cho trường hợp độ đo $p$-ổn định với $1<p\leq 2$. Từ đấy tôi và anh Thắng làm việc với nhau và thu được một số kết quả hay về độ đo ổn định.

Lần thứ ba vào đầu năm 1979, Ông giảng về Lý thuyết Martingales và tích phân ngẫu nhiên (giáo sư Nguyễn Đình Ngọc dịch). Lần này Ông trình bày các kết quả chính của Meyer. Lần đầu tiên tôi được biết khái niệm CADLAG từ các bài giảng của Ông và hiểu được tầm quan trọng của khái niệm này. Nhưng ấn tượng hơn cả là tài phiên dịch của giáo sư Nguyễn Đình Ngọc (có lẽ, theo tôi, giáo sư Ngọc là người giỏi nhất về tiếng Pháp và tiếng Việt). Cứ mỗi lần giáo sư Laurent Schwartz vừa giảng hết một ý, thì giáo sư Ngọc đã dịch ngay sang tiếng Việt rất chuẩn mực, tới mức, giáo sư Laurent Schwartz phải thốt lên "Tôi thực sự kinh ngạc về khả năng tiếng Pháp của anh. Tôi chưa bao giờ gặp một người vừa lau bảng vừa dịch rất trôi chảy các điều khóvề toán như ông Ngọc."

Tóm lại, giáo sư Laurent Schwartz có 3 hưóng nghiên cứu chính thì Ông đã giảng cả 3 cho người Việt Nam. Đó là:

1) Lý thuyết về phân phối (hàm suy rộng). Chính nhờ kết quả cơ bản này, Ông đã được giải thưởng Fields năm 1950. Cần chú, ý rằng Sobolev, nhà toán học vĩ đại người Nga, cũng là tác giả của lý thuyết hàm suy rộng, là tác giả của Đnh lý nhúng nổi tiếng trong các không gian Sobolev, được dùng thưòng xuyên trong lý thuyết phương trình đạo hàm riêng.

2) ánh xạ Radon và Hình học các không gian Banach. Thực ra, Ông không phải là người khởi nguồn lý thuyết này, nhưng Ông có ảnh hưởng lớn tới sự phát triển nhanh chóng của lý thuyết này cho tới tận ngày nay nhờ seminar do Ông và Maurey dẫn dắt từ 1969-1981. Đặc biệt, các khái niệm không gian loại $p$ , đối loại $q$; không gian $p$-trơn đều, $q$-lồi đều do Maurey đưa ra (1973) đã được chính MaureyPisier áp dụng rất thành công vào việc nghiên cứu 3 luật cơ bản của lý thuyết xác suất: Luật số Lớn, Định lý giới hạn trung tâm và luật loga lặp.

Cần lưu ý đã có tới 2 giải thưởng Fields trao cho các nhà toán học đạt được những kết quả xuất sắc trong lĩnh vực Hình học các không gian Banach. Đó là Jain Bourgain (1994, Bỉ) và Timothy Gowers (1998, Anh).

3) Tính toán ngẫu nhiên trên đa tạp.

Cuối cùng, một điều rất quan trọng là sở thích của giáo sư Laurent Schwartz: sưu tập tượng và bắt bướm. Bộ sưu tập về loài bướm nhiệt đới của Ông có tới hơn 25.000 loại (là một trong những bộ sưu tập cá nhân lớn nhất), và một số loại mang tên Ông. Khi đến thăm Việt Nam Ông cũng bắt được một số bướm quí hiếm.

Thầy tôi, giáo sư Vakhania (là đệ tử của Sobolev) hết sức kính trọng Sobolev, có lần nói với chúng tôi: "Lịch sử thường bị lãng quên, nhưng lịch sử không bao giờ quên những đóng góp của N. Sobolev". Tôi muốn mượn câu nói này đã viết: "Cho dù lịch sử có phức tạp như thế nào, thì chúng tôi luôn ghi nhớ công lao to lớn của Ông, giáo sư Laurent Schwatsz, đối với sự phát triển toán học Việt Nam".
Gíao sư Nguyễn Đình Trí (đại học Bách Khoa Hà Nội) là người có quan hệ mật thiết với giáo sư Laurent Schwartz từ lần đầu tiên Ông sang thăm Việt Nam và hai lần sau đã Ông đều giảng bài ở đại học Bách Khoa Hà Nội. Trong một thư (message) giáo sư Trí gửi cho tôi (ngày 20/06/2012) có đoạn cảm động sau:

"Toi doc bai ong viet ve GS L. Schwartz va rat hoan nghenh bai viet nay. Toi la nguoi tiep xuc voi GS Scwartz ngay tu lan Gs sang VN lan dau, sau do gap GS o ICM Moscow 1966 va cung co nhung hoat dong rat hay tai do voi nhung nha toan hoc co cam tinh coi VN, do GS Schwartz goi y va chu tri. Ong con sang lan cuoi cung do loi moi cua Bo truong Tran Hong Quan de tham van ve viec danh gia cac truong dai hoc VN, vi luc do o Phap moi thanh lap Comite National d'Evaluation ma GS Schwartz lam chu tich dau tien. Toi cung da viet ve nhung van de do ngay sau hki GS Schwartz mat. Toi thay anh viet nhu the la du. Toi cung con duoc ve tham mo GS Schwartz o lang Eden ma anh Khoai da viet. O day co mot mo chung cua GS cung voi con trai ong".

Gíao sư Đặng Hùng Thắng gửi 2 thư (message) cho tôi viết:
Trong thư thứ nhất (ngày 17/06/2012)
1) Lần thứ nhất đến VN mùa hè năm 1968: Lúc đó em đang là học sinh lớp 8 chuyên Toán ĐHSP Hà Nội (tức lớp 10 bây giờ). Em đang ở chỗ sơ tán thì được Nhà trường cử về HN tham dự buổi nói chuyện của của L.Sha tại Giảng đường lớn ĐHTH (nay là hội trường Lê Văn Thiêm) với các học sinh giỏi Toán tiêu biểu của Thủ đô. Hôm đó có sự hiện diện của Thủ tướng Phạm Văn Đồng và Bộ trường Tạ Quang Bửu. Em còn nhớ GS đến bắt tay em và một số bạn học sinh và nói “Làm toán là một công việc rất thú vị

2) Lần thứ hai: Thời gian là từ 8/3/1976 đến 26/3 1976 tại Hội trường trường ĐHBK Hà Nội (không phải là tháng 8/1976 như trong bài) “Người VN không quên tôi, Việt nam ghi dấu ấn trong cuộc đời tôi”. Em (và anh) chính là một trong những người VN mãi mãi không quên ông. Bài giảng tuyệt diệu của GS đã thực sự ảnh hưng rất lớn với em, khi đã vừa tốt nghiệp ĐH, đã ghi dấu ấn,tạo bước ngoặt trong cuộc đì làm toán của em: Những bài giảng của ông về Xác suất Radon, xác suất trụ, thác triển độ đo trụ thành độ đo Radon, ,tính chất hình học (loại và đối loại) của không gian Banach, mối quan hệ giũa tính chất hình học của không gian Banach và tính chất xác suất, ánh xạ $p$-tổng hóa và ánh xạ Radon hóa, áp dụng vảo chuyển động Brown… đã cuốn hút em, làm em thực sự thú vị và làm nền tảng cho kiến thức của em về Xác suất, Độ đo, tích phân, Giải tích hàm mãi cho đến bây giờ.

3) Lần thứ ba : Cuối năm 1979 (không phaỉ là tháng 8/1976 như trong bài) Em và anh đã cho ông đọc bài báo trình bày những kết quả mới của mình về thác triển độ đo trụ $p$- ổn định thành xác suất Radon. Từ Pháp GS đã đc gửi thư trả lời (qua địa chỉ Bộ trưởng Nguyễn Đình Tứ ) nêu ra những góp ý comment của ông. Không những vậy ông còn nhờ G.Pisier đc và gửi kèm theo các góp ý và comment của G.Pisier về các kết quả này.

Trong thư thứ hai (ngày 23/06/2012)

Anh Tiến thân kính
Em tìm lại các vở ghi chép cũ thì xác định được chính xác thời điểm đến VN giảng bài lần thứ ba: Từ ngày 11/5/1979 đn ngày 15/5/1979 Nội dung về Lý thuyết Martingale , tích phân ngẫu nhiên và ứng dụng trong giải tích, PT đạo hàm riêng bài toán biên... Lần này GS trình bày tổng quan, không phải bài giảng chi tiết như lần thứ hai (từ 8/3/1976 đn 26/3/1976).


Em cũng muốn bổ sung thêm về các giai đoạn nghiên cứu của GS: Giai đoạn thứ nhất (1944-1954): Lý thuyết phân bố, hàm suy rộng. Giai đoạn thứ hai (từ 1954-1966): Giải tích và Phương trình đạo hàm riêng (PDE). Giai đoạn thứ ba ( từ 1967-1988): Lý thuyết xác suất và hình học không gian Banach.

Ngoài ra GS còn nhiều công trình về lịch sử Toán học và giảng dạy Toán học. Cuốn hồi ký khá dày của GS, em đã được đc bản dich ra tiếng Anh ở một thư viện nước ngoài, có nhiều đoạn cảm động GS viết về VN, thể hiện tình cảm yêu mến của ông đối với đất nước và con người VN. Ước gì cuốn hồi ký này được dịch toàn bộ hoặc một phần ra tiếng Việt thì đây là một việc làm rất có ý nghĩa tưởng nhớ tới GS



Hà Nội, hè 2012.

Hình đã gửi


Như vậy, giáo sư Laurent Schwartz có 16 "học trò" và 2278 "môn đệ".

(GS Nguyễn Duy Tiến)



#330730 Toán học trong bầu cử

Đã gửi bởi Ban Biên Tập on 01-07-2012 - 09:42 trong Toán học lý thú

Trong một chuyện cổ tích, có một cô gái có 3 chàng trai tài ba đến cầu hôn, mà cô gái loay hoay mãi vẫn không biết nên chọn tràng nào. Cô thích chàng B hơn chàng A, vì chàng B hào hoa phong nhã hơn. Nhưng cô lại thích chàng C hơn chàng B, vì chàng C thông minh sắc sảo hơn, nhưng cô lại thích chàng A hơn chàng C, vì chàng A vạm vỡ chắc chắn hơn. Mãi không quyết định được, nên cô đành ở vậy.

Cô gái trong chuyện trên gặp phải tình huống éo le, mà theo thuật ngữ toán học thì gọi là các lựa chọn không xếp được theo thứ tự có tính bắc cầu (tức là nếu thích A hơn B, thích B hơn C, thì cũng thích A hơn C), mà việc xếp thứ tự bị mắc phải lỗi quay vòng (cyclicity). Theo thuật ngữ xã hội, thì lỗi quay vòng của các lựa chọn của cô gái này được gọi là phi lý trí.

Từng người một có thể phi lý trí, thì xã hội cũng có thể phi lý trí. Tệ hơn nữa, là kể cả khi từng người một trong xã hội đều có lý trí (tức là biết mình thích lựa chọn nào hơn lựa chọn nào trong số các lựa chọn được đưa ra), thì cả xã hội cũng vẫn có thể phi lý trí. Nghịch lý về lựa chọn xã hội này đã được Condorcet phát hiện từ thế kỷ 18, và hay được gọi là nghịch lý Condorcet.

Lấy một ví dụ đơn giản sau: giả sử có 5 người $A,B, C,D,E$ cần chọn ra một giải pháp trong số 3 giải pháp $x,y,z$. Kết quả chọn lựa như sau:

$A: x,y,z$ $($tức là $A$ thích $x$ nhất, sau đó đến $y$, và cuối cùng mới đến $z$$)$
$B: y,z,x$
$C: z,x,y$
$D: y,z,x$
$E: x,z,y$

Có thể coi là xã hội 5 thành viên $A, B, C, D, E$ thích lựa chọn $x$ hơn là $y$, vì có 3 người thích $x$ hơn $y$ trong khi chỉ có 2 người thích $y$ hơn $x$. Nhưng nếu so giữa $y$ và $z$ thì $y$ thắng vì có 3 người thích $y$ hơn $z$, và nếu so giữa $z$ và $x$ thì $z$ lại thắng. Như vậy là cái xã hội này đã rơi vào vòng phi ý trí, như là cô gái phía trên.

Đến quãng những năm 1950-1960, vấn đề phi lý trí này đã được Kenneth Arrow nghiên cứu và đưa ra thành định lý về tính "không thể có lý trí" của các hệ thống bầu cử (chọn lựa) trong xã hội. Một phần nhờ định lý này mà Arrow được giải Nobel về kinh tế năm 1972.

Phát biểu toán học của định lý không thể (impossibility theorem) của Arrow như sau:
Gọi A là tập hợp các lựa chọn (các ứng cử viên), và $L(A)$ là tập hợp tất cả các xắp xếp thứ tự tuyến tính đầy đủ của $A$. Một xắp xếp thứ tự tuyến tính đầy đủ là khi với 2 phần tử $x,y$ khác nhau bất kỳ của $A$ thì hoặc $x > y$ ($x$ được thích hơn $y$) hoặc $y > x$ nhưng không thể cả hai, và có tính bắc cầu. Một hàm lựa chọn xã hội chặt chẽ là một hàm $F:L{\left( A \right)^N} \to L\left( A \right)$, trong đó $N$ là số cử tri (mỗi cử tri xắp xếp các lựa chọn theo ý mình, và hàm $F$ là hàm bầu cử đưa ra lựa chọn chung của toàn xã hội dựa trên các lựa chọn riêng của từng cử tri). Định lý Arrow nói rằng, nếu như tập $A$ có ít nhất 3 phần tử (3 ứng cử viên), thì không thể có được hàm $F$ nào thỏa mãn cả 3 điều kiện sau:

1) Tính nhất quán, hay còn gọi là điều kiện hiệu quả Pareto) (unanimity, Pareto efficiency): Nếu mọi cử tri đều cho $x$ nằm trên $y$, thì trong lựa chon chung của xã hội $x$ cũng nằm trên $y$.

2) Không có độc tài (no dictator): Có độc tài là khi tồn tại một chỉ số $i \in \left\{ {1,2,...,N} \right\}$ sao cho luôn có $F\left( {{R_1},{R_2},...,{R_n}} \right) = {R_i}$ với mọi $\left( {{R_1},{R_2},...,{R_n}} \right) \in L{\left( A \right)^N}$ (độc tài quyết định thay cho toàn xã hội, bất kể sở thích của những người khác ra sao)

3) Sự không phụ thuộc vào các lựa chọn không liên quan (independence of irrelevant alternatives, viết tắt là IIA). Điều đó có nghĩa là, nếu như các cử tri chuyển vị trí xếp hạng của một lựa chọn $z$ nào đó trong các bảng xếp hạng của mình nhưng không làm thay đổi thứ tự tương đối của các lựa chọn khác, thì điều đó cũng không làm thay đổi thứ tự tượng đối của các lựa chọn khác trong bảng xếp hạng chung của toàn xã hội. ($z$ ở đây được coi là lựa chọn không liên quan đến bảng xếp hạng tương đối giữa các lựa chọn khác). Tính chất này có thể hình dung qua ví dụ sau: Một người được chọn giữa 1 xe BMW và 1 xe Mercedes và quyết định chọn BMW, sau khi tham khảo nhiều ý kiến. Khi người đó nghe nói "Audi bền lắm", thì điều đó không làm thay đổi quyết định chọn BMW thay vì Mercedes.

Hàm $F$ phía trên là luật bầu cử (lựa chọn xã hội). Như vậy, theo định lý Arrow, thì không có một hệ thống bầu cử nào thỏa mãn cả 3 tính chất rất có lý trên, mà lại luôn xắp xếp được thứ tự các lựa chọn xã hội, mà không rơi vào tình trạng quay vòng phi lý trí dù cho các cử tri chọn lựa ra sao. Chứng minh định lý Arrow không khó: nó chỉ dài khoảng 1 trang, và có thể làm bài tập cho những ai tò mò. Còn ai sốt ruột thì có thể đọc chứng minh của nó ở trang web wikipedia về định lý Arrow đã có link phía trên. Tuy không khó, nhưng định lý Arrow được coi là một trong các cột mốc quan trọng nhất của lý thuyết hiện đại về lựa chọn của xã hội. Nó cũng hay bị suy diễn và hiểu sai. Chẳng hạn, người ta suy diễn từ nó ra rằng các hệ thống dân chủ đều tồi, không có một hệ thống bầu cử dân chủ nào là tốt cả. Thực ra, định lý Arrow không phải vậy. Nó chỉ nói rằng, với bất cứ một hệ thống bầu cử nào theo kiểu sắp xếp thứ tự dựa trên sắp xếp thứ tự của các cử tri, thì cũng tồn tại những tình huống "éo le" không cho ra kết quả thỏa mãn các tính chất "có lý trí". Nhưng các tình huống “éo le” này ít xảy ra trên thực tế, và nếu nó éo le với hệ thống bầu cử này, thì có thể là không "éo le" với hệ thống khác, nên việc chọn lựa hệ thống bầu cử thích hợp cũng là quan trọng.

Một "định lý không thể" thú vị khác trong kinh tế học là định lý Holstrom, được phát biểu như sau: không tồn tại "incentive system" nào thỏa mãn cả 3 tính chất:

1) Cân bằng ngân sách (income = outflow)

2) Có điểm ổn định Nash

3) Thỏa mãn điều kiện hiệu quả Pareto.



#329852 Tổng kết Kì thi thử Đại học năm 2012 của Diễn đàn Toán học (VMF)

Đã gửi bởi Ban Biên Tập on 27-06-2012 - 23:56 trong Năm 2012

logo8namvmf.jpg



1. Giới thiệu

Kì thi thử Đại học VMF - 2012 diễn ra từ thứ 7 ngày 12/11/2011 và kết thúc vào thứ 5 ngày 31/05/2012. Đây là lần đầu tiên Diễn Đàn Toán Học (VMF) tổ chức một cuộc thi hướng vào đối tượng là các học sinh phổ thông lớp 12 chuẩn bị thi Đại học, với mong muốn chuẩn bị cho các sĩ tử một hành trang kiến thức vững chắc trước khi bước vào "trận đánh thật" sẽ diễn ra vào tháng 7 tới. Xuyên suốt hơn 5 tháng, diễn đàn đã soạn thảo bộ 7 đề thi môn toán theo chương trình của Bộ Giáo Dục nhằm giúp các bạn thí sinh ôn tập. Với mỗi đề thi, các thành viên Diễn Đàn Toán Học sẽ có 1 tuần kể từ khi đề thi được công bố để giải quyết hết khả năng của mình trước khi nộp bài làm cho Ban Giám Khảo (BGK). Người tham gia thi thử chỉ được nộp bài một lần duy nhất. Hết hạn nộp bài đề thi sẽ được thảo luận rộng rãi trên diễn đàn, tất cả các thành viên sẽ được xem và nhận xét bài làm của các thí sinh cũng như chất lượng đề thi. BGK gồm 6 người, là các thầy giáo toán, sinh viên, nghiên cứu sinh đang hoạt động tích cực trên diễn đàn : WWW, hxthanh, E. Galois, batigoal, ongtroi, T*genie*. BGK có trách nhiệm ra đề thi, chấm bài và giải đáp thắc mắc để đảm bảo kì thi được diễn ra tốt đẹp. Ngoài đề thi số 1, 6 đề thi còn lại đều ra đúng cấu trúc như một đề thi thật. Tuy còn nhiều thiếu sót, số lượng tham gia mỗi đề thi chưa đông nhưng kì thi thử Đại học phần nào cũng tạo được tiếng vang và là một hoạt động tương đối thành công của ban điều hành VMF kể từ khi diễn đàn quay trở lại với một giao diện mới. Hi vọng rằng kì thi thử sẽ là một bước đệm thu hút các bạn yêu toán, các thầy giáo toán đến với ngôi nhà chung VMF.

2. Một vài số liệu

7 đề thi thử của VMF đã nhận được sự quan tâm lớn của độc giả với tổng cộng 6633 lượt tải đề tính đến hết ngày 21/06/2012 (trung bình gần 1000 lượt tải mỗi đề thi), trong đó đề số 1 có số lượng tải nhiều nhất (1449 lượt) và đề số 7 ít nhất (464 lượt). Đề số 5 có số lượng thí sinh tham gia đông nhất (8 thí sinh) và thấp nhất là đề số 7 chỉ có 1 thí sinh tham gia gửi bài. Thí sinh tham gia tích cực nhất là bạn Đoàn Quốc Khánh (khanh3570883). Khánh tham gia 6/7 đề thi, đặc biệt em mới chỉ là học sinh lớp 11.

$$\begin{array}{|c|c|c|}
\hline
\textbf{Đề thi thử số} & \textbf{Lượt tải về} & \textbf{Số thí sinh tham gia} \\
\hline
1 & 1449 & 3\\
\hline
2 & 1145 & 7 \\
\hline
3 & 1149 & 5 \\
\hline
4 & 700 & 7 \\
\hline
5 & 852 & 8 \\
\hline
6 & 874 & 3 \\
\hline
7 & 464 & 1 \\
\hline
\end{array}$$


Bảng tổng kết thi thử VMF - 2012

vmf1.jpg

Đồ thị cột biểu diễn số lượt tải về qua mỗi đề thi


vmf2.jpg

Đồ thị cột biểu diễn số thí sinh tham gia qua mỗi đề thi



3. Một vài link trao đổi

Các bạn có thể trao đổi về các đề thi, tham khảo đáp án của BGK cũng như đáp án của các thí sinh tham gia tại các link sau :


$\bullet$ Đề thi thử số 1 : http://diendantoanho...showtopic=64684

$\bullet$ Đề thi thử số 2 : http://diendantoanho...showtopic=65834

$\bullet$ Đề thi thử số 3 : http://diendantoanho...showtopic=67158

$\bullet$ Đề thi thử số 4 : http://diendantoanho...showtopic=68420

$\bullet$ Đề thi thử số 5 : http://diendantoanho...showtopic=69292

$\bullet$ Đề thi thử số 6 : http://diendantoanho...showtopic=70913

$\bullet$ Đề thi thử số 7 : http://diendantoanho...showtopic=72545


4. Giải thưởng

Ngoài việc trao thưởng cho thí sinh có tổng điểm cao nhất qua 7 đề thi thử, Diễn Đàn Toán Học nhân kỉ niệm tròn 8 năm tuổi cũng trao giải thưởng là 3 chiếc áo đồng phục VMF cho 3 bạn giải quyết xuất sắc nhất đề thi thử số 3.

4.1. Danh sách các bạn được giải ở đề thi thử số 3

$$\begin{array}{|c|c|c|c|}
\hline
\color{Red}{\textbf{Tên}} & \color{Red}{\textbf{Nick trên diễn đàn}} & \color{Red}{\textbf{Điểm đề thi thử số 3/10}} & \color{Red}{\textbf{Hạng}} \\
\hline
\textbf{Phạm Hùng Vương} & \text{h.vuong_pdl} & 8.75 & 1 \\
\hline
\textbf{Thái Nguyễn Hưng} & \text{Nguyễn Hưng} & 8.5 & 2 \\
\hline
\textbf{Đoàn Quốc Khánh}^{\star} & \text{khanh3570883} & 8.5 & 3 \\
\hline
\end{array}$$

($\star$) Khánh có cùng số điểm với Hưng nhưng xếp sau do xét hệ số phụ (nộp bài muộn hơn).

4.2. Danh sách các bạn được giải chung cuộc

$$\begin{array}{|c|c|c|c|c|}
\hline
\color{Red}{\textbf{Tên}} & \color{Red}{\textbf{Nick trên diễn đàn}} & \color{Red}{\textbf{Số đề tham gia giải/7}} & \color{Red}{\textbf{Tổng điểm/70}} & \color{Red}{\textbf{Hạng}} \\
\hline
\textbf{Đoàn Quốc Khánh} & \text{khanh3570883} & 6 & 48.75 & 1 \\
\hline
\textbf{Vũ Đình Việt} & \text{vietfrog} & 5 & 42.25 & 2 \\
\hline
\textbf{Phạm Hùng Vương} & \text{h.vuong_pdl} & 5 & 41.5 & 3 \\
\hline
\end{array}$$

5. Lời cảm ơn

BGK thi thử Đại học VMF - 2012 xin gửi lời cảm ơn đến bạn PSW đã gửi bài cộng tác cho chúng tôi trong đề thi thử số 3 và admin Nesbit đã duy trì những vấn đề kĩ thuật cho BGK trong suốt đợt thi thử.


6. Lời kết


Graduation.jpg


Thành công là hành trình chứ không chỉ là điểm đến...


Như vậy là kì thi thử Đại học môn toán năm 2012 của VMF đã thành công tốt đẹp. Do lần đầu tổ chức vẫn còn tồn đọng nhiều hạn chế như chất lượng đề thi, công tác chấm bài... nhưng chúng ta cũng cần ghi nhận những nỗ lực của tập thể BGK, BQT diễn đàn cũng như tất cả các thí sinh tham gia. Diễn Đàn Toán Học chúc tất cả các thí sinh VMF nói riêng và tất cả các thí sinh lớp 12 chuẩn bị bước vào kì thi đại học nói chung một mùa thi thành công và gặp nhiều may mắn. Hi vọng 7 đề thi thử của chúng tôi sẽ góp một phần nhỏ hành trang kiến thức giúp các bạn vượt ải vũ môn đầy cam go vào tháng 7 tới. Sự cổ vũ động viên của các bạn là nguồn cổ vũ tinh thần to lớn để chúng tôi tiếp thục hoàn thành các bộ đề thi thử sau được tốt hơn, đóng góp nhiều hơn nữa vào kho tài liệu chung cho các bạn yêu toán phổ thông.


Trân trọng.


Thay mặt BGK thi thử Đại học 2012,

T*genie*




#329285 Toán học trong bầu cử

Đã gửi bởi Ban Biên Tập on 26-06-2012 - 10:11 trong Toán học lý thú

Nicolas de Condorcet (1743-1794) là một nhà triết học, toán học và chính trị học có ảnh hưởng rất lớn ở Pháp thời phục hưng. Có một khái niệm trong chính trị học (lý thuyết về lựa chọn trong xã hội), gọi là "Condorcet winner", mang tên ông ta.

Giả sử có một cuộc bỏ phiếu (bầu cử) để chọn ra một trong $(>= 2)$ các lựa chọn (ứng cử viên) khác nhau: $A,B,C, … , Z$. Lựa chọn $Z$ sẽ được gọi là "Condorcet winner" (kẻ thắng theo nghĩa Condorcet) nếu như khi "đấu tay đôi" với bất kỳ lựa chọn khác nào thì $Z$ đều thắng: giữa $Z$ và $A$ (hay $B, C$, …) thì có hơn $\dfrac{1}{2}$ số người bỏ phiếu thích bầu cho $Z$ hơn là cho $A$ (hay $B, C$, …). "Condorcet loser" được định nghĩa một cách tương tự.

Giả sử có 1 luật bầu cử, áp dụng cho 1 cuộc bầu cử để chọn ra đúng 1 người (hay 1 lựa chọn) thắng. Luật đó sẽ được gọi là thỏa mãn tính Condorcet, nếu như trong mọi trường hợp mà có 1 Condorcet winner thì Condorcet winner đó thắng cử, và ngược lại mọi Condorcet loser đều không thắng cử. (Dễ thấy là không thể có quá 1 Condorcet winner, và có nhiều tình huống không có Condorcet winner nào cả).

Nhiều hệ thống bầu cử được dùng trên thế giới hiện tại không thỏa mãn tính Cordorcet. Trong đó "tai tiếng nhất" là hệ thống "pure plurality": chỉ bầu 1 vòng, và ai có nhiều số phiếu nhất trong số các ứng cử viên (không nhất thiết phải đạt trên $50\%$ số phiếu) thì thắng cuộc. Một ví dụ là cuộc thi trao giải âm nhạc Grammy Award (giải danh giá nhất ở Mỹ) năm 1985: album tương đối mờ nhạt "Can’t Slow Down" của Lionel Richie được giải năm đó, trong khi hai album ứng cử viên xuất chúng năm đó là "Born in the USA" của Bruce Springsteen và "Purple Rain" của Prince được mọi người ưa chuộng hơn thì lại "xâu xé nhau" để cuối cùng cả hai đều được ít phiếu hơn Richie. Theo bình luận viên, thì nếu bầu theo đôi một, chắc chắn Richie sẽ thua cả hai album kia. Ngay hệ thống bầu cử tổng thống 2 vòng của Pháp cũng không tránh khỏi "bi kịch" này: vào năm 2002, ông Jospin, người được đánh giá là được ưa chuộng nhất trong số các ứng cử viên lúc đó, đã bị loại ngay vòng đầu vì phiếu bị phân tán sang những ứng cử viên "gần phe" với ông ta, để cho ứng cử viên đảng cực hữu lọt vào vòng hai.

Tuy nhiên, có một câu hỏi "lý thuyết" là: liệu việc thỏa mãn tính Condorcet có chắc là tốt đối với một hệ thống bầu cử không? Và câu trả lời là KHÔNG! Nói cách khác, lựa chọn là Condorcet winner cũng có những khi lại là lựa chọn rất tồi đối với xã hội, và ngược lại Condorcet loser có khi lại là lựa chọn tốt cho xã hội.

Thử lấy một ví dụ sau: $3$ người "cannibal" $A, B, C$ có $4$ lựa chọn $a, b, c, d$. Lựa chọn $a$ là $B$ và $C$ ăn thịt $A$, lựa chọn $b$ là $A$ và $C$ ăn thịt $b$, lựa chọn $c$ là $A$ và $B$ ăn thịt $C$, và lựa chọn $d$ là không ăn thịt nhau mà tiến hóa lên tìm cái khác để ăn. Những người cannibal này thích ăn thịt người khác và không thích mình bị ăn thịt. Như vậy lựa chọn $d$ là Condorcet loser ở đây, nhưng có khi lại là lựa chọn tốt nhất cho nhóm người $A, B, C$. Nếu chẳng hạn $A, B, C$ tính điểm cho mỗi lựa chọn theo sự sung sướng hay đau khổ mà nó mang lại cho mình: ví dụ như $A$ cho điểm $-10$ cho lựa chọn $a$ (quá đau khổ khi bị ăn thịt), và $+1$ cho mỗi lựa chọn $b$ và $c$, và $0$ cho lựa chọn $d$, và $B$ và $C$ cũng làm tương tự, rồi cộng các điểm vào với nhau, thì lựa chọn $d$ sẽ là lựa chọn có tổng số điểm cao nhất $(= 0)$ trong khi các lựa chọn khác có tổng số điểm là âm.

(Ví dụ Grammy Award phía trên là lấy từ bài viết của Paul E. Johnson: Voting systems, 2005)

Theo zung.zetamu.net




#326543 Tìm số hạng tổng quát của dãy số bằng hàm Hypebolic

Đã gửi bởi Ban Biên Tập on 18-06-2012 - 08:08 trong Chuyên đề toán THPT

Ví dụ 4: Tìm số hạng tổng quát của dãy số $({u_n})$ định bởi: $\left\{ \begin{array}{l} {u_1} = \frac{5}{4}\\ {u_{n + 1}} = 2{u_n} + \sqrt {3u_n^2 - 3} \,,\,\,\forall n \in \mathbb{N^*} \end{array} \right.$


Giải: Đặt $\left\{ \begin{array}{l} ch\beta = 2\\ sh\beta = \sqrt 3 \end{array} \right.$. Giải hệ ta được ${e^\beta } = 2 + \sqrt 3 $

Đặt $\left\{ \begin{array}{l} ch\alpha = \frac{5}{4}\\ sh\alpha = \sqrt {{{\left( {\frac{5}{4}} \right)}^2} - 1} = \frac{3}{4} \end{array} \right.$. Giải hệ ta được ${e^\alpha } = 2$

Ta có: ${u_1} = ch \alpha$. Giả sử ${u_n} = ch\left[ {\alpha + \left( {n - 1} \right)\beta } \right]$. Khi đó: $$ch\beta .ch\left[ {\alpha + \left( {n - 1} \right)\beta } \right] + sh\alpha .sh\left[ {\alpha + \left( {n - 1} \right)\beta } \right] = ch\left( {\alpha + n\beta } \right)$$
Vậy: $${u_n} = \frac{{{e^{\alpha + \left( {n - 1} \right)\beta }} + {e^{ - \,\alpha - \,\left( {n - 1} \right)\beta }}}}{2} = {\left( {2 + \sqrt 3 } \right)^{n - 1}} + \frac{1}{4}{\left( {2 - \sqrt 3 } \right)^{n - 1}}$$

Hình đã gửi


Ví dụ 5: Tìm số hạng tổng quát của dãy số $({u_n})$ định bởi: $\left\{ \begin{array}{l} {u_1} = 3\\ {u_{n + 1}} = \frac{1}{2}{u_n}\sqrt {16 + u_n^2} \,,\,\,\forall n \in \mathbb{N^*} \end{array} \right.$

Giải: Biến đổi giả thiết: $\frac{{{u_{n + 1}}}}{4} = 2\left( {\frac{{{u_n}}}{4}} \right)\sqrt {1 + {{\left( {\frac{{{u_n}}}{4}} \right)}^2}} $

Đặt ${x_n} = \frac{{{u_n}}}{4}$ ta được dãy $({x_n})$ định bởi: $\left\{ \begin{array}{l} {x_1} = \frac{3}{4}\\ {x_{n + 1}} = 2{x_n}\sqrt {1 + x_n^2} \,,\,\,\forall n \in \mathbb{N^*} \end{array} \right.$

Đặt $sh\alpha = \frac{3}{4} \Leftrightarrow \frac{{{e^\alpha } - {e^{ - \,\alpha }}}}{2} = \frac{3}{4}$. Giải phương trình ta được ${e^\alpha } = 2$

Ta có: ${x_1} = sh\alpha $. Giả sử ${x_n} = sh\left( {{2^{n - 1}}\alpha } \right)$. Khi đó: $${x_{n + 1}} = 2sh\left( {{2^{n - 1}}\alpha } \right)\sqrt {1 + s{h^2}\left( {{2^{n - 1}}\alpha } \right)} = 2sh\left( {{2^{n - 1}}\alpha } \right)ch\left( {{2^{n - 1}}\alpha } \right) = sh\left( {{2^n}\alpha } \right)$$
Vậy: $${x_n} = sh\left( {{2^{n - 1}}\alpha } \right) = \frac{{{{\left( {{e^\alpha }} \right)}^{{2^{n - 1}}}} - {{\left( {{e^{ - \,\alpha }}} \right)}^{{2^{n - 1}}}}}}{2}\,\,\,\text{ suy ra}\,\,\, {u_n} = 2\left( {{2^{{2^{n - 1}}}} - {{\left( {\frac{1}{2}} \right)}^{{2^{n - 1}}}}} \right)$$
Ví dụ 6: Tìm số hạng tổng quát của dãy số $({u_n})$ định bởi: $\left\{ \begin{array}{l} {u_1} = \frac{3}{{\sqrt 6 }}\\ {u_{n + 1}} = 24u_n^3 - 12\sqrt 6 u_n^2 + 15{u_n} - \sqrt 6 \,,\,\,\forall n \in \mathbb{N^*} \end{array} \right.$

Nhận xét: Đặt ${u_n} = a{x_n} + b$, thay vào giả thiết và rút gọn ta được: $$a{x_{n + 1}} + b = 24{a^3}x_n^3 + 12\left( {6{a^2}b - \sqrt 6 {a^2}} \right)x_n^2 + 3\left( {24a{b^2} - 8\sqrt 6 ab + 5a} \right){x_n} + 24{b^3} - 12\sqrt 6 {b^2} + 15b - \sqrt 6 $$
Đầu tiên ta chọn $b$ sao cho: $\left\{ \begin{array}{l} 6{a^2}b - \sqrt 6 {a^2} = 0\\ 24{b^3} - 12\sqrt 6 {b^2} + 15b - \sqrt 6 = b \end{array} \right. \Rightarrow b = \frac{1}{{\sqrt 6 }}$

Khi đó: $a{x_{n + 1}} = 24{a^3}x_n^3 + 3a{x_n}\,\,\text{hay}\,\,{x_{n + 1}} = 24{a^2}x_n^3 + 3{x_n}$

Giải: Đặt ${u_n} = \frac{1}{{\sqrt 6 }}{x_n} + \frac{1}{{\sqrt 6 }}$ ta được dãy số $({x_n})$ định bởi: $\left\{ \begin{array}{l} {x_1} = 2\\ {x_{n + 1}} = 4x_n^3 + 3{x_n} \end{array} \right.$

Đặt $sh\alpha = 2 \Leftrightarrow \frac{{{e^\alpha } - {e^{ - \,\alpha }}}}{2} = 2$. Giải phương trình ta chọn nghiệm ${e^\alpha } = 2 + \sqrt 5 $

Áp dụng công thức $sh\left( {3\alpha } \right) = 4s{h^3}\alpha + 3sh\alpha $, rồi chứng minh quy nạp ta được:
$${x_n} = sh\left( {{3^{n - 1}}\alpha } \right) = \frac{{{{\left( {{e^\alpha }} \right)}^{{3^{n - 1}}}} - {{\left( {{e^{ - \,\alpha }}} \right)}^{{3^{n - 1}}}}}}{2} = \frac{{{{\left( {2 + \sqrt 5 } \right)}^{{3^{n - 1}}}} - {{\left( {\dfrac{1}{{2 + \sqrt 5 }}} \right)}^{{3^{n - 1}}}}}}{2}$$
Vậy: ${u_n} = \frac{1}{{2\sqrt 6 }}\left[ {{{\left( {2 + \sqrt 5 } \right)}^{{3^{n - 1}}}} - {{\left( {\frac{1}{{2 + \sqrt 5 }}} \right)}^{{3^{n - 1}}}}} \right] + \frac{1}{{\sqrt 6 }}$

Ví dụ 7: Tìm số hạng tổng quát của dãy số $({u_n})$ định bởi: $\left\{ \begin{array}{l} {u_1} = 1\\ {u_{n + 1}} = \frac{{ - \,u_n^2}}{{u_n^2 + 2{u_n} + 2}}\,,\,\,\forall n \in \mathbb{N^*} \end{array} \right.$

Giải: Biến đổi giả thiết: ${u_{n + 1}} + 1 = \frac{{2\left( {{u_{n + 1}} + 1} \right)}}{{1 + {{\left( {{u_{n + 1}} + 1} \right)}^2}}}$

Đặt ${x_n} = {u_n} + 1$ ta được dãy số $({x_n})$ định bởi: $\left\{ \begin{array}{l} {x_1} = 2\\ {x_{n + 1}} = \frac{{2{x_n}}}{{1 + x_n^2}}\,,\,\,\forall n \in \mathbb{N^*} \end{array} \right.$

Đặt $\coth \alpha = 2 \Leftrightarrow \frac{{{e^\alpha } + {e^{ - \,\alpha }}}}{{{e^\alpha } - {e^{ - \,\alpha }}}} = 2$. Giải phương trình ta được: ${e^\alpha } = \sqrt 3 $

Ta có: ${x_2} = \frac{{2\coth \alpha }}{{1 + {{\coth }^2}\alpha }} = \frac{{2\frac{1}{{th\alpha }}}}{{1 + \frac{1}{{t{h^2}\alpha }}}} = \frac{{2th\alpha }}{{1 + t{h^2}\alpha }} = th\left( {2\alpha } \right)$

Giả sử ${x_n} = th\left( {{2^{n - 1}}\alpha } \right)$. Khi đó: ${x_{n + 1}} = \frac{{2th\left( {{2^{n - 1}}\alpha } \right)}}{{1 + t{h^2}\left( {{2^{n - 1}}\alpha } \right)}} = th\left( {{2^n}\alpha } \right)$

Vậy: ${x_n} = th\left( {{2^{n - 1}}\alpha } \right) = \frac{{{{\left( {{e^{2\alpha }}} \right)}^{{2^{n - 1}}}} - 1}}{{{{\left( {{e^{2\alpha }}} \right)}^{{2^{n - 1}}}} + 1}} = \frac{{{3^{{2^{n - 1}}}} - 1}}{{{3^{{2^{n - 1}}}} + 1}}\,\,\,\text {suy ra}\,\,\,\,{u_n} = \frac{{ - \,2}}{{{3^{{2^{n - 1}}}} + 1}},\,\,\,\forall n \ge 2$

Ví dụ 8: Tìm số hạng tổng quát của dãy số $({u_n})$ định bởi: $\left\{ \begin{array}{l} {u_1} = \frac{5}{4}\\ {u_{n + 1}} = \frac{{u_n^3 + 3{u_n}}}{{1 + 3u_n^2}}\,,\,\,\forall n \in \mathbb{N^*} \end{array} \right.$

Giải: Đặt $\coth \alpha = {u_1} \Leftrightarrow \frac{{{e^\alpha } + {e^{ - \,\alpha }}}}{{{e^\alpha } - {e^{ - \,\alpha }}}} = \frac{5}{4}$. Giải phương trình ta được: ${e^\alpha } = 3$

Giả sử ${u_n} = \coth \left( {{3^{n - 1}}\alpha } \right)$. Khi đó: ${u_{n + 1}} = \frac{{{{\coth }^3}\left( {{3^{n - 1}}\alpha } \right) + 3\coth \left( {{3^{n - 1}}\alpha } \right)}}{{1 + 3{{\coth }^2}\left( {{3^{n - 1}}\alpha } \right)}} = \dfrac{{\dfrac{1}{{t{h^3}\left( {{3^{n - 1}}\alpha } \right)}} + \dfrac{3}{{th\left( {{3^{n - 1}}\alpha } \right)}}}}{{1 + \dfrac{3}{{t{h^2}\left( {{3^{n - 1}}\alpha } \right)}}}}$

Vậy: ${u_n} = \coth \left( {{3^{n - 1}}\alpha } \right) = \frac{{{9^{{3^{n - 1}}}} + 1}}{{{9^{{3^{n - 1}}}} - 1}}$

Ví dụ 9: Cho dãy $({x_n})$ định bởi: $\left\{ \begin{array}{l} {x_1} = 4\\ {x_{n + 1}} = x_n^2 - 2\,,\,\,\forall n \in \mathbb{N^*} \end{array} \right.$ Tính $\lim \left( {\frac{{{x_{n + 1}}}}{{{x_1}{x_2}...{x_n}}}} \right)$

Giải: Đặt ${x_n} = 2{u_n}$ ta được dãy số $({u_n})$ định bởi: $\left\{ \begin{array}{l} {u_n} = 2\\ {u_{n + 1}} = 2u_n^2 - 1\,,\,\,\forall n \in \mathbb{N^*} \end{array} \right.$

Đặt $ch\alpha = 2 \Leftrightarrow \frac{{{e^\alpha } + {e^{ - \,\alpha }}}}{2} = 2$, giải phương trình ta chọn nghiệm ${e^\alpha } = 2 + \sqrt 3 $

Chứng minh quy nạp ta được: ${u_n} = ch\left( {{2^{n - 1}}\alpha } \right)$ suy ra ${x_n} = 2ch\left( {{2^{n - 1}}\alpha } \right)$

Ta có: $${x_{n + 1}} = 2ch\left( {{2^n}\alpha } \right) = {\left( {{e^\alpha }} \right)^{{2^n}}} + {\left( {{e^{ - \,\alpha }}} \right)^{{2^n}}} = {\left( {2 + \sqrt 3 } \right)^{{2^n}}} + {\left( {2 - \sqrt 3 } \right)^{{2^n}}}$$
$${x_1}{x_2}...{x_n} = {2^n}ch\left( \alpha \right).ch\left( {2\alpha } \right)....ch\left( {{2^{n - 1}}\alpha } \right) = \frac{{{2^n}}}{{sh\alpha }}sh\left( \alpha \right).ch\left( \alpha \right).ch\left( {2\alpha } \right)....ch\left( {{2^{n - 1}}\alpha } \right)$$
\[ = \frac{{sh\left( {{2^n}\alpha } \right)}}{{sh\alpha }} = \frac{{{{\left( {{e^\alpha }} \right)}^{{2^n}}} - {{\left( {{e^{ - {\kern 1pt} \alpha }}} \right)}^{{2^n}}}}}{{{e^\alpha } - {e^{ - {\kern 1pt} \alpha }}}} = \frac{{{\rm{ }}{{\left( {2 + \sqrt 3 } \right)}^{{2^n}}} - {{\left( {2 - \sqrt 3 } \right)}^{{2^n}}}}}{{2\sqrt 3 }}\]
Từ đó dễ dàng tính được $\lim \left( {\frac{{{x_{n + 1}}}}{{{x_1}{x_2}...{x_n}}}} \right) = 2\sqrt 3 $

Ví dụ 10: Tìm số hạng tổng quát của dãy số $({u_n})$ định bởi: $\left\{ \begin{array}{l} {u_1} = 3\\ {u_{n + 1}} = \left( {8u_n^3 + 4{u_n}} \right)\sqrt {1 + u_n^2} \,,\,\,\forall n \in \mathbb{N^*} \end{array} \right.$

Giải: Đặt $sh\alpha = 3 \Leftrightarrow \frac{{{e^\alpha } - {e^{ - \,\alpha }}}}{2} = 3$. Giải phương trình ta được: ${e^\alpha } = 3 + \sqrt {10} $

Giả sử ${u_n} = sh\left( {{4^{n - 1}}\alpha } \right)$.

Khi đó: ${u_{n + 1}} = \left[ {8s{h^3}\left( {{4^{n - 1}}\alpha } \right) + 4sh\left( {{4^{n - 1}}\alpha } \right)} \right]\sqrt {1 + s{h^2}\left( {{4^{n - 1}}\alpha } \right)} = sh\left( {{4^n}\alpha } \right)$

Vậy: ${u_n} = sh\left( {{4^{n - 1}}\alpha } \right) = \frac{1}{2}\left[ {{{\left( {{e^\alpha }} \right)}^{{4^{n - 1}}}} - {{\left( {\frac{1}{{{e^\alpha }}}} \right)}^{{4^{n - 1}}}}} \right] = \frac{1}{2}\left[ {{{\left( {\sqrt {10} + 3} \right)}^{{4^{n - 1}}}} - {{\left( {\sqrt {10} - 3} \right)}^{{4^{n - 1}}}}} \right]$

Ví dụ 11: (Đề thi Olympic đề nghị của trường Lê Hồng Phong TPHCM năm 2009)

Tìm số hạng tổng quát của dãy số $({u_n})$ định bởi: $\left\{ \begin{array}{l} {u_1} = m\\ {u_{n + 1}} = u_n^4 - 12u_n^3 + 50u_n^2 - 84{u_n} + 50\,,\,\,\forall n \in \mathbb{N^*} \end{array} \right.$

Giải: Đặt ${u_n} = {x_n} + 3$ ta được: $\left\{ \begin{array}{l} {x_1} = m - 3\\ {x_{n + 1}} = x_n^4 - 4x_n^2 + 2\,,\,\,\forall n \in \mathbb{N^*} \end{array} \right.$

Đặt ${x_n} = 2{v_n}$ ta được: $\left\{ \begin{array}{l} {v_1} = \frac{{m - 3}}{2}\\ {v_{n + 1}} = 8v_n^4 - 8v_n^2 + 1\,,\,\,\forall n \in \mathbb{N^*} \end{array} \right.$

* Nếu $m = 5$ ta được ${v_n} = 1$ suy ra ${u_n} = 5,\,\, \forall n \in \mathbb{N^*}$

* Nếu $m = 1$ ta được ${u_1} = 1,\, {u_n} = 5, \,\forall n \ge 2$

* Nếu $\left| {\frac{{m - 3}}{2}} \right| < 1 \Leftrightarrow 1 < m < 5$. Đặt ${v_1} = \cos \alpha $

Áp dụng công thức $\cos 4\alpha = 8{\cos ^4}\alpha - 8{\cos ^2}\alpha + 1$ ta được ${v_2} = \cos \left( {4\alpha } \right)$

Chứng minh quy nạp ta được ${v_n} = \cos \left( {{4^{n - 1}}\alpha } \right)$ suy ra ${u_n} = 2\cos \left( {{4^{n - 1}}\alpha } \right) + 3\,,\,\,\forall n \in \mathbb{N^*}$

* Nếu $\frac{{m - 3}}{2} > 1 \Leftrightarrow m > 5$. Đặt $ch\alpha = \frac{{m - 3}}{2} \Leftrightarrow \frac{{{e^\alpha } + {e^{ - \,\alpha }}}}{2} = \frac{{m - 3}}{2} \Leftrightarrow {e^{2\alpha }} - \left( {m - 3} \right){e^\alpha } + 1 = 0$

Giải phương trình ta lấy nghiệm ${e^\alpha } = \frac{{m - 3 + \sqrt {{m^2} - 6m + 5} }}{2}$

Chứng minh quy nạp ta được: ${v_n} = \frac{1}{2}\left( {{{\left( {{e^\alpha }} \right)}^{{4^{n - 1}}}} + {{\left( {{e^{ - \,\alpha }}} \right)}^{{4^{n - 1}}}}} \right)$

Suy ra: ${u_n} = {\left( {{e^\alpha }} \right)^{{4^{n - 1}}}} + {\left( {{e^{ - \,\alpha }}} \right)^{{4^{n - 1}}}} + 3\,,\,\,\forall n \in \mathbb{N^*}$

* Nếu $\frac{{m - 3}}{2} < - 1 \Leftrightarrow m < 1$. Đặt $ch\alpha = \frac{{3 - m}}{2} \Leftrightarrow \frac{{{e^\alpha } + {e^{ - \,\alpha }}}}{2} = \frac{{3 - m}}{2} \Leftrightarrow {e^{2\alpha }} + \left( {m - 3} \right){e^\alpha } + 1 = 0$

Giải phương trình ta lấy nghiệm ${e^\alpha } = \frac{{3 - m + \sqrt {{m^2} - 6m + 5} }}{2}$

Chứng minh quy nạp ta được: ${v_n} = \frac{1}{2}\left( {{{\left( {{e^\alpha }} \right)}^{{4^{n - 1}}}} + {{\left( {{e^{ - \,\alpha }}} \right)}^{{4^{n - 1}}}}} \right)\,,\,\,\forall n \ge 2$

Suy ra: ${u_n} = {\left( {{e^\alpha }} \right)^{{4^{n - 1}}}} + {\left( {{e^{ - \,\alpha }}} \right)^{{4^{n - 1}}}} + 3\,,\,\,\forall n \ge 2$

Ví dụ 12: (Trích đề thi chọn đội tuyển TPHCM _2012)

Tìm số hạng tổng quát của dãy số $({x_n})$ định bởi: $\left\{ \begin{array}{l} {x_1} = \frac{4}{5}\\ {x_{n + 1}} = \frac{{x_n^4}}{{x_n^4 - 8x_n^2 + 8}}\,,\,\forall n \in \mathbb{N^*} \end{array} \right.$

Giải: Từ giả thiết ta được ${x_n} > 0,\,\,\forall n \in \mathbb{N^*}$ và $\frac{1}{{{x_{n + 1}}}} = 8{\left( {\frac{1}{{{x_n}}}} \right)^4} - 8{\left( {\frac{1}{{{x_n}}}} \right)^2} + 1$

Đặt ${u_n} = \frac{1}{{{x_n}}}$ ta được dãy số $({u_n})$ định bởi: $\left\{ \begin{array}{l} {u_1} = \frac{5}{4}\\ {u_{n + 1}} = 8u_n^4 - 8u_n^2 + 1,\,\,\forall n \in \mathbb{N^*} \end{array} \right.$

Đặt $ch\alpha = \frac{5}{2} \Leftrightarrow \frac{{{e^\alpha } + {e^{ - \,\alpha }}}}{2} = \frac{5}{4}$, giải phương trình ta chọn nghiệm ${e^\alpha } = 2$

Chứng minh quy nạp ta được: $${u_n} = ch\left( {{4^{n - 1}}\alpha } \right) = \frac{{{{\left( {{e^\alpha }} \right)}^{{4^{n - 1}}}} + {{\left( {{e^{ - \,\alpha }}} \right)}^{{4^{n - 1}}}}}}{2} = \frac{{{2^{^{{4^{n - 1}}}}} + {{\left( {\dfrac{1}{2}} \right)}^{{4^{n - 1}}}}}}{2}$$
(Áp dụng công thức: $ch(4x) = 8ch^4x – 8ch^2x + 1$)

Vậy: ${x_n} = \frac{2}{{{2^{{4^{n - 1}}}} + {{\left( {\dfrac{1}{2}} \right)}^{{4^{n - 1}}}}}}$

Ví dụ 13: Cho hai dãy số $({x_n})$ và $({y_n})$ xác định như sau: ${x_1}=a>0,\,{y_1}=b>0,\,\,\,{x_{n + 1}} = \frac{{{x_n} + {y_n}}}{2}$, ${y_{n + 1}} = \sqrt {{x_{n + 1}}.{y_n}} \,\,,\,\,\forall n \in \mathbb{N^*}$. Tìm $\lim {x_n}\,\,\text{và}\,\,\lim {y _n}$

Giải: Ta xét các trường hợp sau:

* Trường hợp 1: Nếu $a = b$ thì ${x_n} = {y_n} = a, \,\,\forall n \in \mathbb{N^*}$ nên $lim{x_n} = lim{y_n} = 1$

* Trường hợp 2: Nếu $a < b$ thì đặt $\cos \alpha = \frac{a}{b}\,\,\,,\,\,\alpha \in \left( {0\,,\,\frac{\pi }{2}} \right)$. Khi đó ta có:
$${x_2} = \frac{{b\left( {1 + \dfrac{a}{b}} \right)}}{2} = \frac{{b\left( {1 + \cos \alpha } \right)}}{2} = b{\cos ^2}\frac{\alpha }{2}\,\,\,\text{và}\,\,\, {y_2} = \sqrt {{x_2}{y_1}} = \sqrt {{b^2}{{\cos }^2}\frac{\alpha }{2}} = b\cos \frac{\alpha }{2}$$
$${x_3} = \frac{{{x_2} + {y_2}}}{2} = \dfrac{{b\cos \dfrac{\alpha }{2}\left( {1 + \cos \dfrac{\alpha }{2}} \right)}}{2} = b\cos \frac{\alpha }{2}\cos \frac{\alpha }{{{2^2}}}\,\,\,\text{và}\,\,\, {y_3} = \sqrt {{x_3}{y_2}} = b\cos \frac{\alpha }{2}\cos \frac{\alpha }{{{2^2}}}$$
Chứng minh quy nạp ta được:
$${x_n} = b\left( {\cos \frac{\alpha }{2}.\cos \frac{\alpha }{{{2^2}}}....\cos \frac{\alpha }{{{2^{n - 1}}}}} \right){\cos ^2}\frac{\alpha }{{{2^n}}}\,\,\,\text{và}\,\,\,{y_n} = b\cos \frac{\alpha }{2}\cos \frac{\alpha }{{{2^2}}}...\cos \frac{\alpha }{{{2^n}}},\,\forall n \ge 2$$
Áp dụng công thức: $\cos x = \frac{{{\mathop{\rm s}\nolimits} {\rm{in2}}x}}{{2\sin x}}$ ta rút gọn được ${y_n} = b\frac{{\sin \alpha }}{{{2^n}\sin \dfrac{\alpha }{{{2^n}}}}}$

Vì $\lim \left( {{2^n}\sin \frac{\alpha }{{{2^n}}}} \right) = \lim \left( {\alpha \frac{{\sin \dfrac{\alpha }{{{2^n}}}}}{{\dfrac{\alpha }{{{2^n}}}}}} \right) = \alpha $ nên $\lim {y_n} = \frac{{b\sin \alpha }}{\alpha }$

Từ ${x_n} = {y_n}\cos \frac{\alpha }{{{2^n}}} \Rightarrow \lim {x_n} = \lim \left( {{y_n}\cos \frac{\alpha }{{{2^n}}}} \right) = \left( {\lim {y_n}} \right)\left( {\lim \cos \frac{\alpha }{{{2^n}}}} \right) = \frac{{b\sin \alpha }}{\alpha }$

* Trường hợp 3: Nếu $a > b$, chọn số $\alpha$ sao cho ${\mathop{\rm ch}\nolimits} \alpha = \frac{a}{b}$

$${x_2} = \frac{{b\left( {1 + \dfrac{a}{b}} \right)}}{2} = \frac{{b\left( {1 + {\mathop{\rm ch}\nolimits} \alpha } \right)}}{2} = b{{\mathop{\rm ch}\nolimits} ^2}\frac{\alpha }{2}\,\,\,\text{và}\,\,\, {y_2} = \sqrt {{x_2}{y_1}} = \sqrt {{b^2}{{{\mathop{\rm ch}\nolimits} }^2}\frac{\alpha }{2}} = b{\mathop{\rm ch}\nolimits} \frac{\alpha }{2}$$
$${x_3} = \frac{{{x_2} + {y_2}}}{2} = \frac{{b{\mathop{\rm ch}\nolimits} \dfrac{\alpha }{2}\left( {1 + {\mathop{\rm ch}\nolimits} \dfrac{\alpha }{2}} \right)}}{2} = b{\mathop{\rm ch}\nolimits} \frac{\alpha }{2}{\mathop{\rm ch}\nolimits} \frac{\alpha }{{{2^2}}}\,\,\,\text{và}\,\,\, {y_3} = \sqrt {{x_3}{y_2}} = b{\mathop{\rm ch}\nolimits} \frac{\alpha }{2}{\mathop{\rm ch}\nolimits} \frac{\alpha }{{{2^2}}}$$
Chứng minh quy nạp ta được:
$${x_n} = b\left( {{\mathop{\rm ch}\nolimits} \frac{\alpha }{2}.{\mathop{\rm ch}\nolimits} \frac{\alpha }{{{2^2}}}....{\mathop{\rm ch}\nolimits} \frac{\alpha }{{{2^{n - 1}}}}} \right){{\mathop{\rm ch}\nolimits} ^2}\frac{\alpha }{{{2^n}}}\,\,\,\text{và}\,\,\,{y_n} = b{\mathop{\rm ch}\nolimits} \frac{\alpha }{2}{\mathop{\rm ch}\nolimits} \frac{\alpha }{{{2^2}}}...{\mathop{\rm ch}\nolimits} \frac{\alpha }{{{2^n}}},\,\,\forall n \ge 2$$
Áp dụng công thức: ${\mathop{\rm ch}\nolimits} x = \frac{{sh\,{\rm{2}}x}}{{2shx}}$ ta rút gọn được ${y_n} = b\frac{{{\mathop{\rm sh}\nolimits} \alpha }}{{{2^n}.{\mathop{\rm sh}\nolimits} \dfrac{\alpha }{{{2^n}}}}}$

Đặt $x = \frac{\alpha }{{{2^n}}}$, khi $n\, \to + \infty $ thì $x \to 0$
$$\lim \left( {{2^n}.{\mathop{\rm sh}\nolimits} \frac{\alpha }{{{2^n}}}} \right) = \mathop {\lim }\limits_{x \to 0} \frac{{\alpha \left( {{e^x} - {e^{ - x}}} \right)}}{{2x}} = \mathop {\lim }\limits_{x \to 0} \left( {\frac{\alpha }{{{e^x}}}.\frac{{{e^{2x}} - 1}}{{2x}}} \right) = \alpha$$
$$\text{nên}\,\,\,\lim {y_n} = \frac{{b{\mathop{\rm sh}\nolimits} \alpha }}{\alpha },\,\,\,\lim \left( {{\mathop{\rm ch}\nolimits} \frac{\alpha }{{{2^n}}}} \right) = \mathop {\lim }\limits_{x\, \to \,0} \frac{{{e^x} + {e^{ - x}}}}{2} = 1$$
$$\text{và}\,\,\,{x_n} = {y_n}.\cosh \frac{\alpha }{{{2^n}}} \Rightarrow \lim {x_n} = \lim \left( {{y_n}.{\mathop{\rm ch}\nolimits} \frac{\alpha }{{{2^n}}}} \right) = \left( {\lim {y_n}} \right)\left( {\lim {\mathop{\rm ch}\nolimits} \frac{\alpha }{{{2^n}}}} \right) = \frac{{b{\mathop{\rm sh}\nolimits} \alpha }}{\alpha }.$$

II. Bài tập áp dụng

Bài 1: Tìm công thức tính số hạng tổng quát của các dãy số sau:
a) $\left\{ \begin{array}{l} {u_1} = 1\\ {u_{n + 1}} = 5u_n^2 - \frac{2}{5}\,,\,\,\forall n \in \mathbb{N^*} \end{array} \right.$
b) $\left\{ \begin{array}{l}{u_1} = 1\\ {u_{n + 1}} = 5u_n^2 - \frac{2}{5}\,,\,\,\forall n \in \mathbb{N^*} \end{array} \right.$
c)
$\left\{ \begin{array}{l}u_1 = \frac{1}{3}\\u_{n + 1} = 6u_n^2 + 8u_n + \frac{5}{3}\,\,\,\forall n \in \mathbb{N^*}\end{array} \right.$
d) $\left\{ \begin{array}{l}{u_1} = 20\\{u_{n + 1}} = u_n^2 - {2.5.2^n}\,,\,\,\forall n \in \mathbb{N^*}\end{array} \right.$
e) $\left\{ \begin{array}{l}{u_1} = 5\\{u_{n + 1}} = {2.3^{{2^n}}}u_n^2 - {3^{\left( {n + 1} \right){2^n}}}\,,\,\,\forall n \in \mathbb{N^*}\end{array} \right.$
f) $\left\{ \begin{array}{l}{u_1} = 0\\{u_{n + 1}} = \frac{{\sqrt {3{u_n} + 8} }}{3} - 2\,,\,\,\forall n \in \mathbb{N^*}\end{array} \right.$

Bài 2: Tìm công thức tính số hạng tổng quát của các dãy số sau:
a) $\left\{ \begin{array}{l}{u_1} = - 3\\{u_{n + 1}} = 2{u_n}\sqrt {1 + u_n^2} \,,\,\,\forall n \in \mathbb{N^*}\end{array} \right.$
b) $\left\{\begin{array}{l}{u_1} = - \,2\\{u_{n + 1}} = \frac{1}{2}{u_n} + \frac{1}{{{u_n}}}\,,\,\,\forall n \in\mathbb{N^*}\end{array} \right.$
c) $\left\{ \begin{array}{l}{u_1} = 1\\{u_{n + 1}} = 2{u_n}\sqrt {u_n^2 - 4{u_n} + 5} - 4\sqrt {u_n^2 - 4{u_n} + 5} + 2\,,\,\,\forall n \in \mathbb{N^*}\end{array} \right.$
d)
$\left\{ \begin{array}{l}u_1 = 3\\u_{n + 1} = \dfrac{u_n^2 - 4}{u_n^2 + 2u_n + 2}\,\,\,\forall n \in \mathbb{N^*} \end{array} \right.$
e) $\left\{ \begin{array}{l}u_1 = 5\\u_{n + 1} = 3u_n + 2\sqrt {2u_n^2 - 2} \,\,\,\forall n \in \mathbb{N^*} \end{array} \right.$
f) $\left\{ \begin{array}{l}{u_1} = - \,2\\{u_{n + 1}} = 2\sqrt {u_n^2 + 1} + \sqrt 5 {u_n}\,,\,\,\forall n \in \mathbb{N^*}\end{array} \right.$

Bài 3: Tìm công thức tính số hạng tổng quát của các dãy số sau:
a)
$\left\{ \begin{array}{l}u_1 = \dfrac{{\sqrt {2 + \sqrt 2 } }}{2} - 1\\u_{n + 1} = 4u_n^3 + 12u_n^2 + 9u_n\,\,\,\forall n \in \mathbb{N^*}\end{array} \right.$
b) $\left\{ \begin{array}{l} {u_1} = \frac{3}{2}\\ {u_{n + 1}} = 4u_n^3 - 6u_n^2 + 6{u_n} - \frac{3}{2}\,,\,\,\forall n \in \mathbb{N^*} \end{array} \right.$
c) $\left\{ \begin{array}{l}u_1 = 1\\u_{n + 1} = \dfrac{u_n^3 - 3u_n - 6}{3u_n^2 + 6u_n +7}\,\,\,\forall n \in \mathbb{N^*}\end{array} \right.$

Bài 4: Tìm công thức tính số hạng tổng quát của các dãy số sau:
a)
$\left\{ \begin{array}{l} u_1 = 3\\ u_{n + 1} = 8u_n^4 - 64u_n^3 + 184u_n^2 - 224u_n + 99\,\,\,\forall n \in \mathbb{N^*} \end{array} \right.$
b) $\left\{ \begin{array}{l} {u_1} = 1\\ {u_{n + 1}} = \dfrac{{u_n^4}}{{u_n^4 + 8u_n^2 + 8}}\,\,\,\forall n \in \mathbb{N^*} \end{array} \right.$
c) $\left\{ \begin{array}{l} u_1 = 5\\ u_{n + 1} = \dfrac{u_n^4}{\left( 8 + 4u_n \right)\sqrt {1 + u_n^2}} \end{array} \right.$
d) $\left\{ \begin{array}{l} u_1 = 2\\ u_{n + 1} = \dfrac{1 + 3u_n^2}{u_n^3 + 3u_n}\,\,\,\forall n \in \mathbb{N^*} \end{array} \right.$
e) $\left\{ \begin{array}{l} u_1 = 1\\ u_{n + 1} = \frac{u_n^3 + 6u_n^2 + 15u_n + 14}{3u_n^2 + 12u_n + 13}\,\,\,\forall n \in \mathbb{N^*} \end{array} \right.$
Mời các bạn cùng thảo luận tại: http://diendantoanho...showtopic=73743



#325346 Diễn đàn Heidelberg của những người được Giải thưởng

Đã gửi bởi Ban Biên Tập on 15-06-2012 - 09:09 trong Tin tức - Vấn đề - Sự kiện

LIÊN ĐOÀN TOÁN HỌC THẾ GIỚI (IMU)

Hình đã gửi
Ingrid Daubechies (Chủ tịch), Martin Grotschel (Tổng thư kí)
Ngày 22 tháng 6 năm 2012
Thông cáo báo chí




Diễn đàn Heidelberg của những người được giải.
Những người đoạt các giải thưởng danh giá: Abel, Fields và Turing sẽ gặp các nhà khoa học trẻ đầy hoài bão. Đó là nội dung của Diễn đàn được thành lập bởi Quỹ Klaus Tschira.

Quỹ Klaus Tschira sẽ thành lập Diễn đàn Heidelberg của những người được giải, dưới hình thức một hội nghị thường niên, để những nhà khoa học đạt các giải thưởng cao quý của ngành Toán (Abel và Fields) và Khoa học máy tính (Turing) gặp gỡ với những nhà nghiên cứu trẻ tài năng. Diễn đàn này được đề xướng bởi Học viện Nghiên cứu lý thuyết Heidelberg (HITS), một viện nghiên cứu của Quỹ Klaus Tschira ở Đức nhằm thúc đẩy các ngành Khoa học tự nhiên, Toán học và Khoa học máy tính. Diễn đàn Heidelberg của những người dược giải (gọi tắt Diễn đàn Heidelberg) được xây dựng theo mô hình Hội nghị thường niên của Các cuộc hội nghộ thường niên Lindau của những người được giải thưởng Nobel Laureate - tiến hành hơn 60 năm qua với mục đích tìm kiếm những ý tưởng mới. Klaus Tschira, người sáng lập và đồng quản lý Quỹ nói: “Gặp gỡ với các nhà khoa học hàng đầu của Toán học và Khoa học máy tính sẽ truyền cảm hứng và tiếp thêm động lực mạnh mẽ cho các nhà khoa học trẻ tuổi.”


Hình đã gửi



Thỏa thuận hợp tác về Diễn đàn Heidelberg giữa những người tổ chức và các tổ chức trao giải (Viện Hàn lâm Khoa học và Văn học Nauy, Liên đoàn Toán học thế giới và Hiệp hội Máy tính ) đã được ký kết vào ngày 22 tháng 5 năm 2012 tại Oslo nhân kỷ niệm 10 năm giải thưởng Abel.

Việc nghiên cứu trong tất cả các ngành đều cần đến các phương pháp toán học và công cụ máy tính, và thành quả của chúng được ghi nhận trong mọi khía cạnh cuộc sống hàng ngày của chúng ta. Toán học và Khoa học Máy tính là nền tảng không thể thiếu của thế giới công nghệ. Giải thưởng Turing từ lâu đã được ghi nhận là giải thưởng khoa học cao quý nhất thế giới cho ngành Khoa học Máy tính, cũng như giải thưởng Abel và Fields đối với ngành Toán. Tuy nhiên, trong khi các nhà nghiên cứu trẻ của ngành Vật lý, Hóa học, Dược, và Kinh tế có cơ hội giao lưu gần gũi với những người đoạt giải Nobel tại Lindau hàng năm thì giới nghiên cứu trẻ của Toán học và Khoa học Máy tính lại chưa có những cơ hội như vậy.

Bắt đầu từ tháng 9/2013, Diễn đàn Heidelberg sẽ tạo cơ hội để những người đoạt giải thưởng Abel, Fields và Turing có thể gặp gỡ những nhà khoa học trẻ từ khắp nơi trên thế giới. Cuộc gặp mặt sẽ được tổ chức tại thành phố Heidelberg, nơi có viện nghiên cứu của Quỹ Klaus Tschira là Học viện Nghiên cứu lý thuyết Heidelberg (HITS). Diễn đàn được tổ chức với sự cộng tác của Hiệp hội máy tính (ACM; Giải thưởng Turing), Liên đoàn Toán học thế giới (Giải thưởng Fields) và Viện Hàn lâm Khoa học và Văn học Nauy (Giải thưởng Abel). Cuộc gặp đầu tiên của Diễn đàn Heidelberg sẽ diễn ra trong các ngày 23-27 tháng 9 năm 2013.

Đại diện cho Liên đoàn Toán học thế giới, Chủ tịch Ingrid Daubechies, thể hiện sự vui mừng trước sáng kiến của Quỹ Klaus Tschira, và hi vọng diễn đàn mới này sẽ góp phần đáng kể trong việc thúc đẩy nhiệt huyết toán học cho thế hệ sau.Việc gặp gỡ những người “anh hùng khoa học” đối với các nhà nghiên cứu mới vào nghề là điều rất quan trọng bởi nó có thể cho họ thấy những những nhà khoa học lẫy lừng này không phải là những thần tượng không thể vươn tới được. Chẳng hạn như Toán học, cùng với tất cả những gì trừu tượng của nó, vẫn được chinh phục bởi con người, trong đó mọi khía cạnh của sự trao đổi giữa người với người cũng đóng một phần quan trọng. Chúng ta cần thu hút và giữ vững tinh thần nghiên cứu toán học ở những trí tuệ trẻ sáng lạn có ham muốn làm việc và tích cực đẩy mạnh sự phát triển của ngành Toán học.

Quỹ Klaus Tschira Stiftung là một quỹ của Đức thúc đẩy Khoa học tự nhiên, Toán học và Khoa học máy tính. Học viện Nghiên cứu lý thuyết Heidelberg là một viện nghiên cứu của Quỹ.

Bắt đầu từ năm 2003, Viện Hàn lâm Khoa học và Văn học Nauy trao Giải thường Abel hàng năm cho các nghiên cứu khoa học xuất sắc của ngành Toán. Giải thưởng trị giái 800,000 Euro. Thành lập năm 1857, Viện Hàn lâm Khoa học và Văn học Nauy là một cơ quan phi chính phủ, liên quốc gia và liên ngành quan tâm tới tất cả các lĩnh vực kiến thức. Viện có 895 thành viên, trong và ngoài Nauy.

Liên đoàn Toán học thế giới bao gồm hơn 70 quốc gia thành viên. Liên đoàn thúc đẩy hợp tác quốc tế về Toán học, tổ chức các Đại hội Toán học thế giới, khuyến khích và ủng hộ các hoạt động Toán học trên khắp thế giới nhằm đóng góp vào sự phát triển của Toán học ở bất kỳ phương diện nào: toán học đơn thuần, toán học ứng dụng hay giáo dục Toán học.

Hiệp hội Máy tính là cộng đồng giáo dục và khoa học về máy tính lớn nhất thế giới, quy tụ các nhà giáo dục, nghiên cứu và chuyên gia về máy tính để truyền cảm hứng vào các cuộc đối thoại, chia sẻ các nguồn nghiên cứu và những thách thức trong ngành.

Để biết thêm thông tin về sự kiện quan trọng này, vui lòng xem thêm các tin tức liên quan tại website: http://www.mathunion.org/general/news và các thông cáo báo chí đính kèm



Xin vui lòng thông báo những thông tin này đến cộng đồng của bạn
Thân mến,
Martin Grotschel
Tổng thư kí Liên đoàn Toán học thế giới
Theo http://viasm.edu.vn/?p=4750
Mời bạn cùng thảo luận tại:



#325345 Gs Vũ Hà Văn

Đã gửi bởi Ban Biên Tập on 15-06-2012 - 08:53 trong Các nhà Toán học

BBT: Đây là bài viết của Giáo sư Nguyễn Duy Tiến viết và gửi cho Diễn đàn toán học nhân ngày sinh nhật GS Vũ Hà Văn 12/06


Viết một bài Toán lạ thành quen
Viết trăm bài Văn quen thành lạ

Hình đã gửi


Chúc mừng sinh nhật GS Vũ Hà Văn

Vũ Hà Văn, con người Tài Hoa
Có thể nói rằng Vũ Hà Văn là nhà toán học Việt Nam xuất sắc nhất vể Toán Học Rời Rạc gồm lý thuyết Tổ Hợp, Xác Suất và Khoa Học Máy Tính, và là nhà toán học hàng đầu của thế giới trong lĩnh vực nói trên. Thật vậy, tính tới tháng 8/2010 (xem Lý lịch Khoa học dưới đây) GS. Vũ Hà Văn đã công bố 104 công trình trên các tạp chí uy tín nhất của Toán học (như Ann. Math; Adv. Math.), hoặc trên các tạp chí chuyên ngành (như Ann. Probab.), trong số đó có tới 74 bài ISI, và chỉ số H của anh là 13. Thêm vào đó, anh được trao tặng nhiều giải thưởng danh giá (trong đó có Giải thưởng Polya); được mời báo cáo ở rất nhiều hội nghị quốc tế; anh làm việc ở nhiều đại học và trung tâm khoa học lớn (3 lần làm việc ở Princeton institute for advanced studies; tham gia tổ chức nhiều hội nghị quốc tế. Năm 2006 GS. Vũ Hà Văn cùng với Terencer Tao (giải thưởng Fields) xuất bản cuốn sách nổi tiếng Additive Combinatorics (Tổ Hợp Cộng Tính), một đóng góp mới cho Toán Học Rời Rạc. Ngoài ra, anh còn đào tạo được nhiều học trò giỏi trong lĩnh vực này.

Hình đã gửi



Nếu Ngô Bảo Châu là bom tấn (đánh điểm) thì Vũ Hà Văn là bom rải thảm (đánh diện). Đó là hai nhà toán học (quốc tịch) Việt Nam mở đầu cho thời kỳ Thăng hoa của Toán Học nước ta.
Tôi là người nghiên cứu Xác Suất trong nhiều năm (gần 40 năm), khi đọc Lý lịch Khoa học của GS, Vũ Hà Văn, tự thấy cần rút lui để thế hệ trẻ (tài năng) tiến bước. Và giờ đây tôi xin đóng vai người dẫn chuyện (MC) thì có lẽ phù hợp hơn.

Tóm tắt tiểu sử khoa học của GS Vũ Hà Văn
Ngày sinh: 12/06/1970 (tuổi Canh Tuất).
Nơi sinh: Hà Nội.
Quê quán: Nam Đinh.
Họ và tên bố: Vũ Quần Phương (nhà thơ).
Họ và tên mẹ: Đào Thị Hường (dược sĩ).


Vũ Hà Văn là cựu học sinh chuyên toán trường trung học danh tiếng Chu Văn An, tốt nghiệp phổ thông năm 1986. Anh sinh ra trong một gia đình có truyền thống hiếu học (bố anh tên thật là Vũ Ngọc Chúc, sinh năm 1940, tốt nghiệp đại học Y, làm bác sĩ 2 năm, rồi trở thành nhà thơ Vũ Quần Phưng, với bài thơ nổi tiếng: Đợi, 100 bài thơ hay của thế kỉ XX, NXBGD, 2007), thi đỗ đại học Bách Khoa Hà Nội với số điểm rất cao (á thủ khoa). Sau đó anh dược nhà nước Việt Nam gửi sang Hungary học.

Vũ Hà Văn tốt nghiệp cử nhân tại Đại học Etvos, Budapest, Hungary năm 1994. Trong thư gửi cho tôi (ngày 12/07/2011) anh tâm sự với tôi: "Thật ra tiểu sử khoa học của Văn có một điểm khác, so với phẩn lớn những người làm toán khác. Đó là Văn khi mới vào đại học chưa theo học ngành toán, mà là hoe điện tử tại Đại Học Bách Khoa Budapest (chứ không phải Đại Học Etvos là trường Tổng Hợp). Học ở đó 1,5 năm. Văn vẫn thích học toán và làm nghiên cứu với bà vợ của GS. Lovasi, rồi GS Lovasi khuyên nên chuyển sang học Toán Lý thuyết. Văn cũng đi gặp anh Trần Văn Nhung hỏi ý kiến và anh Nhung cũng khuyên đây là một cơ hội tốt. Sau đó Văn mới chuyển sang trường Etvos. Thành ra việc trở thành người làm toán cũng một phẩn là do say mê, một phẩn như có số mệnh sắp đặt vậy, chứ con đường không được thẳng băng như mọt số người làm toán khác''.

Bảo vệ luận án tiến sĩ tại Đại học Yale, Mỹ, năm 1998 dưới sự hướng dẫn của GS Laszlo Lovasz.
Là người được tặng Giải thưởng Polya năm 2008.
Sau thời gian làm hậu tiến sĩ tại Viện Nghiên cứu cấp cao (IAS) Princeton và tại Ban Nghiên cứu của Microsoft, từ năm 2001 đến 2005, anh làm việc tại Đại học Caliíbmia ở San Diego, với tư cách trợ lý giáo sư, phó giáo sư và giáo sư (full professor). Từ mùa thu năm 2005, anh trở thành giáo sư Khoa Toán Đại học Rutgers, và hiện tại anh là GS. Đại Học Yale (nơi anh bảo vệ tiến sĩ, năm 1998).
Anh là giáo sư thỉnh giảng của Đại học Paris 6 năm 2006.


Lĩnh vực nghiên cứu của Vũ Hà Văn gồm: toán học tổ hợp, xác suất rời rạc, lý thuyết sô cộng tính và Khoa học Máy tính Lý thuyết.
Anh đã hai lần nhận được Giải thưởng Sloan dành cho các tài năng trẻ ở Mỹ khi viết luận án tiến sĩ (1997), và khi làm nghiên cứu viên (2002), rồi Giải thưởng NSF Career (2003).

Anh là thành viên Viện Nghiên cứu cấp cao Princeton trong những năm 1998-1999, 2005-2006, và 2007 (năm 2007 là người lãnh đạo nhóm dự án Số học tổ hợp tại viện này).
Tính đến nay, số nhà toán học được tặng Giải thưởng Polya vẫn còn rất ít, và họ
đều là những nhà toán học xuất sắc.

Có lần tôi hỏi Vũ Hà Văn: "Trong những giải thưởng anh đã nhận, theo anh giải thưởng nào danh giá hơn cả?"
Anh trả lời ngay: "Giải thưởng Polya năm 2008."

Giải thưởng Polya.
Đây là giải thưởng do Hội Toán công nghiệp và ứng dụng Society for Industrial and Applied Mathematics/ SIAM) của Mỹ lập ra từ năm 1969, trao 2 năm một lẩn, lẩn lượt cho những ứng dụng nổi bật về lý thuyết tổ hợp hoặc những đóng góp nổi bật trong các lĩnh vực khác mà George Polya từng yêu thích như: lý thuyết xấp xỉ, giải tích phức, lý thuyết số, đa thức trực giao, lý thuyết xác suất, v.v. Giải thưởng chủ yếu dành cho những công trình mới, hiếm khi cho thành tựu trong quá khứ.

SIAM được thành lập năm 1952, đặt trụ sở chính tại Philadelphia (Mỹ), có 12.000 thành viên cá nhân và 500 thành viên tập thể (gồm các trường đại học, viện nghiên cứu, xí nghiệp công nghiệp, công ty dịch vụ, tư vấn dân sự và quân sự khắp thế giới).
Quá trình xét chọn người trúng giải được tiến hành nghiêm ngặt. Uỷ ban Giải thưởng được lập ra ít nhất 18 tháng trước ngày tặng giải; phải tham khảo rộng khi xét chọn; có nhận xét bằng văn bản trước 10 tháng, v.v.
Người trúng giải được tặng một tấm huy chương và 20.000 USD. Lần trao Giải thưởng Polya năm 2008 dành cho những ứng dụng của lý thuyết tổ hợp. Người duy nhất được tặng giải là nhà toán học Việt Nam Van H. Vu (tức Vũ Hà Văn).


Đây là danh sách những người được giải Polya (tính đến 2010):
* 1971 R. L. Graham, K. Leeb, B. L. Rothschild, A. w. Hales, and R. I. Jewett
* 1975 R. p. Stanley, E. Sz emeredi, and R. M. Wilson
* 1979 L. Lovasz (thầy hướng dẫn luận án tiến sĩ của Vũ Hà Văn)
* 1983 A. Bjomer and p. Seymour
* 1987 A. c. Yao
* 1992 G. Kalai and s. Shelah
* 1994 Gregory Chudnovsky and Harry Kesten
* 1996 Jeffry Ned Kahn and David Reimer
* 1998 Percy Deift, Xin Zhou, and Peter Samak
* 2000 Noga Alon (cùng với Spencer cuốn sách nổi tiếng: Probabilistic Method)
* 2002 Craig A. Tracy and Harold Widom
* 2004 Neil Robertson and Paul Seymour
* 2006 Gregory Lawler, Oded Schramm, and Wendelin Wemer
* 2008 Van H. Vu
* 2010 Emmanuel Candốs and Terence Tao.

Như vậy, Vũ Hà Văn được giải Polya trước Terence Tao. Năm 2012 sẽ công bố người nhận giải Polya tiếo theo.
Từ ngày 16 đến 22/12/2009, tại Seoul, diễn ra cuộc gặp làm việc giữa các nhà toán học Mỹ và Hàn Quốc. Terence Tao, Van H. Vu (tức Vũ Hà Văn), James T. McKeman, Frank MorganHee Oh là những người được Hội Toán học Mỹ cử sang Seoul giới thiệu những công trình mới. Theo dõi qua Internet, tôi thấy giới toán học Hàn Quốc đón tiếp rất trọng thị đoàn đại biểu giới toán học Mỹ mà Vũ Hà Văn là một thành viên.


Năm 2009, Nhà nước ta đã công nhận Vũ Hà Văn là giáo sư kiêm chức tại Viện Toán học Việt Nam, khi anh 39 tuổi. Ngô Bảo ChâuVũ Hà Văn là hai giáo sư trẻ nhất Việt Nam. Dù sống và làm việc ở nước ngoài nhiều năm, cả hai anh vẫn giữ quốc tịch CHXHCN Việt Nam.

Mới đây nhất, GS Vũ Hà Văn đã phát biểu cảm tưởng về “sự kiện Ngô Bảo Châu”:
GS Châu là bạn tôi. Tôi rất phấn khởi khi nghe tin này. Mặc dù “tin đồn ” đã có từ ỉ âu trong giới toán học, nhưng khi biết chắc chắn vẫn vui hơn. Cũng như anh Châu, tôi và vợ con về Việt Nam gần như thường xuyên, và mỗi lẩn về, tôi đều kết hợp giảng dạy và nghiên cứu tại Viện Toán học và Trường đại học Khoa học tự nhiên (thuộc Đại học Quốc gia Hà Nội).


Hướng Nghiên cứu Toán học của Vũ Hà Văn
Tôi đã nghe GS. Vũ Hà Văn giảng bài hai lần. Lần đầu tiên ở xeminar Xác Suất trường đại học KHTN (DHQGHN) anh nói về Phương Pháp Xác Suất ứng dụng trong Lý Thuyết Số. Đây là lĩnh vực do trường phái Toán Học Hungary (với những nhà khoa học lừng danh như Renyi, Erdos) khai sinh từ những năm 60 của thế kỷ trước và hiện tại đang là mốt của Xác Suất rời rạc vì có nhiều ứng dụng trong lý thuyết số, đồ thị ngẫu nhiên và khoa học máy tính, Trong bài giảng, điều gây ấn tưạng nhất đối với tôi là anh ứng dụng bất đẳng thức Talagrand (một nhà toán học hàng đầu của nước Pháp trong lĩnh vực Xác Suất trên các không gian Banach) vào công trình của anh (cùng với Kim).

Quãng 5 năm sau (2009), tôi cùng với sinh viên nghe anh giảng về Matrận Ngẫu Nhiên. Lần này anh đã truyền đạt một trong những vấn đề về Tổ Hợp Cộng Tính hết sức sâu sắc và đang được cả thế giới toán, vật lý quan tâm. Anh giảng bài mà không cần một giáo án nào, tất cả kiến thức của anh rất chắc chắn, lần lượt tuôn ra từ bộ não sáng sủa của anh (không nhầm lẫn). Nghe anh giảng về toán giống như nghe bố anh, nhà thơ Vũ Quần Phương bình thơ vậy, tôi hết sức thích những ý tưởng hình học của anh và những ví dụ cụ thể. Anh là người có năng khiếu sư phạm bẩm sinh.

Bây giờ ta hãy tìm hiểu sơ qua những vấn đề mà GS. Vũ Hà Văn quan tâm.
1) Phương Pháp xác suất.
Từ lâu người ta đã dùng các kết quả của Xác Suất để chứng minh một số kết quả của Giải Tích, Đại Số hoặc Lý Thuyết số (ví dụ như dùng luật số lớn chứng minh Đinh lý Weierstras Xấp xỉ hàm liên tục bằng đa thức; Bất đẳng thức Khinchin v.v...). Để giới thiệu về phưang pháp quan trọng và mới này, tôi dẫn ra đây lời mở đầu của cuốn sách "The Probabilistic Method" của hai nhà toán học Noga Alon, Joel H. Spencer (2008, xuất bản lần thứ 3, của John Wiley & Sons. INC): Phương pháp xác suất là một phương pháp hữu hiệu được ứng dụng rộng rãi trong tổ hợp. Một trong những nguyên nhân chính giải thích cho sự phát triển mau lẹ của phương pháp này là tẩm quan trọng của tính ngẫu nhiên trong lý thuyết khoa học máy tính và vật lý thống kê. Tương tác qua lại giữa toán học rời rạc và khoa học máy tính đã gợi ý cho một cái nhìn thuật toán trong việc nghiên cứu phương pháp xác suất trong tổ hợp. Đây cũng là cách tiếp cận được sử dụng trong cuốn sách này. Cũng vì thế mà trong sách sẽ bao gồm những thảo luận về các kỹ thuật thuật toán cùng với sự nghiên cứu các phương pháp cổ điển cũng như các công cụ hiện đại được ứng dụng trong đó. Phẩn thứ nhất của cuốn sách đưa ra các công cụ được ứng dụng trong các lập luận xác suất, bao gồm các kỹ thuật cơ bản sử dụng kỳ vọng và phương sai, cũng như các ứng dụng gần đây của martingale và bất đẳng thức tương quan ịcorrelation inequaỉity). Phần thứ hai nghiên cứu một lớp rộng rãi các chủ đề trong đó các kỹ thuật xác suất đã được ứng dụng thành công. Phẩn này bao gồm một số chương về đồ thị ngẫu nhiên và đổ thị rời rạc (discrepancy graph), cũng như một vài lĩnh vực trong lý thuyết khoa học máy tính: mạch phức hợp ịcircuit complexity), hình học tính toán, tái ngẫu nhiên hóa các thuật toán ngẫu nhiên. Giữa các chương là các các đoạn ngắn được đặt dưới tiêu đề chung Lăng kính xác suất. Đây là những lời giải đẹp, chúng không nhất thiết có liên quan tới các chương trước đó và có thể đọc một cách độc lập. Cơ sở của phương pháp xác suất có thề mô tả như sau: đề chứng minh sự tồn tại của một cấu trúc tổ hợp với các tính chất xác định, chúng ta xây dựng một không gian xác suất thích hợp và chỉ ra rằng một phần tử được chọn một cách ngẫu nhiên trong không gian này có tính chất mong muốn với một xác suất dương. Phương pháp này được khởi đầu bởi Paul Erdos, người đã đóng góp rất nhiều cho sự phát triển của nó trong suốt hơn 50 năm, phương pháp này vì thê có thề được gọi là "Phương pháp Erdos". Đóng góp của Erdos có thể được đánh giá không chỉ bởi rất nhiều kết quả sâu sắc của ông, mà còn dựa trên nhiều bài toán và giả thuyết mang tính dẫn đường mà từ đó đã kích thích nghiên cứu rộng lớn trong lĩnh vực này. Dường như là không thể có một cuốn từ điển về phương pháp xác suất; có quá nhiều kết quả thú vị gần đây có sử dụng những lập luận xác suất, và chúng tơ sẽ không cố gắng đề cập tới tất cả những kết quả đó.

2)Đồ thị ngẫu nhiên.

Để hiểu lý thuyết này, ta xét ví dụ sau.Số Ramsey $R(k, l)$ là số nguyên dương bé nhất sao cho mỗi đồ thi đầy đủ với $n$ đỉnh $K_n$ và các cung được tô hai màu xanh và đỏ tồn tại hoặc đồ thi con đầy đủ với $k$ đỉnh $K_k$ với tất cả các cung màu đỏ, hoặc $K_l$ màu xanh. Ramsey (1929) đã chứng minh rằng $R(k, l)$ là hữu hạn với hai số nguyên bất kỳ $k$ và $l$. Ta sẽ nhận được cận dưới của các số đường chéo Ramsey $R(k, l).$

Mệnh đề: Nếu
$$\binom{n}{k}2^{1-\binom{k}{2}}<1$$
thì $R(k,k)>n$. Do đó $R(k,k) > \begin{bmatrix} 2^{k/2} \end{bmatrix}$ đối với tất cả $k \geq 3.$

Chứng minh. Xét một đồ thi ngẫu nhiên hai màu có các cung của $K_n$ nhận được bằng cách tô màu mỗi cung màu xanh hay màu đỏ độc lập với xác suất như nhau. Với tập cố đinh $R$ của $k$ đỉnh, ký hiệu $A_R$ là biến cố: đồ thi con cảm sinh của $K_n$ trên $R$ là đơn màu (tức là tất cả các cung của nó là đỏ, hoặc tất cả xanh). Rõ ràng là,
$$Pr\begin{bmatrix} A_R \end{bmatrix}=2^{1-\binom{k}{2}}.$$
Vì có $\binom{k}{2}$ khả năng chọn $R$, nên xác suất để có ít nhất một trong các biến cố $A_R$ xảy ra nhiều nhất bảng
$$\binom{n}{k} 2^{1-\binom{k}{2}}<1$$
với xác xuất dương không có biến cố $A_R$ nào xảy ra và tồn tại một đồ thị hai màu của $K_n$ mà không đơn màu $K_k$ nào; tức là, $R(k,k)>n$. Chú ý rằng nếu $k \geq 3$ và ta lấy $n=\begin{bmatrix} 2^{k/2}\end{bmatrix}$ với tất cả $k \geq 3$

3) Ma trận ngẫu nhiên
Ma trân ngẫu nhiên là ma trận
$$A=(a_{ij})$$
Trong đó $a_{ij}$ là các biến ngẫu nhiên.
Định lý Terence Tao, Van Vu (2006). Nếu $a_{ij}$ là các biến Bernoulli, tức là,
$$P_{(a_{ij}=1)}=P_{(a_{ij}=1)}=1/2$$
độc lập, thì
$$P_(det(A)=0)<(3/4+o(1))^n.$$


Một trong những kết quả hay nhất (theo tôi) của Vũ Hà Văn

Định lý giới hạn trung tâm đối với đa diện ngẫu nhiên
Đầu những năm 60, RenyiSulanke nghiên cứu mô hình sau. Cho $K$ là một hình lồi trong $R^d$ ($d$ cố định) có thể tích 1. Lấy mẫu $n$ điểm trong $K$ một cách ngẫu nhiên và gọi $K_n$ là bao lồi của những điểm đó. Lý thuyết do $R-S$ đề ra là nghiên cứu các biến ngẫu nhiên được xác định bởi $K_n$, chẳng hạn số đỉnh của $K_n$ hoặc thể tích $K_n$.

Một bài toán mở cơ bản của lý thuyết là:Các biến ngẫu nhiên này thỏa mãn đinh lý giới hạn trung tâm.

Cho$K$là một tập lồi trơn với thể tích 1 trong $R^d$. Chọn ngẫu nhiên $n$ điểm một cách độc lập theo phân bố đều. Ký hiệu $K_n$ là bao lồi của các điểm này và gọi nó là đa diện ngẫu nhiên. Việc nghiên cứu các phiếm hàm then chốt (như thể tích, số đỉnh,...) của $K_n$, khởi đầu từ Efron, R.LenyiSulanke là một hướng cổ điển của hình học lồi. Ký hiệu $Vol(K_n)$ là thể tích của $K_n$. Một giả thuyết nổi tiếng trong lĩnh vực này là biến ngẫu nhiên này thỏa mãn định lý giới hạn trung tâm khi n tiến tới vô cùng.
Hình đã gửi
Giả thuyết.
Tồn tại hàm $\varepsilon (n)$ tiến đến 0 khi $n$ tiến đến vô cùng sao cho với mọi $x \in R$ ta có:
$$\begin{vmatrix} P\left(\dfrac{Vol(K_n)-E(Vol(K_n))}{\sqrt{Var(Vol(Kn))}}<n\right)-\Phi(x)\end{vmatrix}<\varepsilon(n)$$
Trong đó $\Phi$ là hàm phân phối chuẩn. Giả thuyết này mới chỉ được xét trong trường hợp $K$ là hình cầu trong $R^2$. Trong việc nghiên cứ đa diện ngẫu nhiên trong thể lồi $K$, mặt của $K$ đống vai trò cốt lõi. Một phần có ý nghĩa của tài liệu hiện có tập trung hai trường hợp sau:

1) $K$ trơn, nghĩa là biên của $K$ hai lần khả vi với độ cong dương hữu hạn.
2) $K$ là một đa diện.

Trong bài báo rất hay của Vũ Hà Văn đăng trong tạp chí hàng đầu Advances of Math, Văn đã chứng minh giả thiết cho trường hợp $K$ là tập lồi trơn

Định lý 1 của Vũ Hà Văn 2006 Cho $K$ là thể lồi trơn với thể tích 1 trong $R^d$. Tồn tại hàm $\varepsilon (n)$ tiến đến $0$ khi $n$ tiến đến vô cùng sao cho với mọi $x \in R$ ta có:
$$\begin{vmatrix} P\left(\dfrac{Vol(K_n)-E(Vol(K_n))}{\sqrt{Var(Vol(Kn))}}<n\right)-\Phi(x)\end{vmatrix}<\varepsilon(n)$$


Kí hiệu $f_i(K_n)$ là số các mặt bên $i$-chiều của $K_n,0 \leq i \leq d-1$ $(f_0(K_n)$ là số đỉnh của $K_n)$ Văn đã có kết quả sau:

Định lý 2 của Vũ Hà Văn, 2006. Cho $K$ là thể lồi trơn với thể tích 1 trong $R^d$. Tồn tại hàm $\varepsilon (n)$ tiến đến 0 khi $n$ tiến đến vô cùng sao cho với mọi $x \in R$ và

$$\begin{vmatrix} P(\frac{f_i(K_n)-E(f_i(K_n))}{\sqrt{Var(f_i(Kn))}}<n)-\Phi(x))\end{vmatrix}<\varepsilon(n).$$

$0 \leq i \leq d - 1$ ta có:

Trong cả hai định lý trên, ta có thể lấy
$$\varepsilon(n)=n^{-1/(d+1)+o(1)}.$$

Môi trường Làm viêc của Vũ Hà Văn
Toán học Hungary PAUL ERDOS

Vũ Hà Văn là hạt giống toán học tốt và được gieo trồng ở Hungary, mảnh đất rất màu mỡ, là quê hưang toán học của nhiều học giả xuất chúng như G. Polya, Paul Erdos, A. Rényi, von Neumann, Halmos vân vân. Hầu hết các học giả này gốc Do Thái, làm việc ở Mỹ. Hướng nghiên cứu của Vũ Hà Văn được khởi nguồn từ các ý tưởng của ErdosRényi (xem bài "Toán học Hungary").

Bạn làm Toán của Vũ Hà Văn. Vũ Hà Văn có khá nhiều bạn làm toán ở nhiều nơi khác nhau trên thế giới. Một trong số đó là Terence Tao: “Mozart của toán học".

T. Tao sinh năm 1975 (kém Văn 5 tuổi) tại Adelaide, Australia trong một gia đình người Hoa, bố là bác sĩ, mẹ là giáo viên dạy toán. Mới han 2 tuổi, nhờ “học mót” toán và tiếng Anh qua ti-vi, Tao đã dạy lại hai môn này cho một cậu bé 5 tuổi! Đến 9 tuổi, Tao được nhận vào chương trình nghiên cứu tài năng đặc biệt của Đại học Johns Hopkins ở Mỹ. Mới 10 tuổi, Tao lọt vào đội tuyển quốc gia Australia đi dự Olympic Toán quốc tế và đoạt huy chương đồng; năm sau, đoạt huy chương bạc; rồi đến năm 13 tuổi, đoạt huy chương vàng. T. Tao là người đoạt huy chương vàng ít tuổi nhất trong lịch sử các Olympic Toán quốc tế.

17 tuổi, Tao được tặng bằng thạc sĩ tại Australia, và nhận được học bổng sang Mỹ học tiếp. 20 tuổi, Tao bảo vệ thành công luận án tiến sĩ tại Đại học Princeton danh tiếng; 25 tuổi, trở thành giáo sư. Năm 2006, mới 31 tuổi, T. Tao được tặng Huy chương Fields (được coi như Giải thưởng Nobel trong toán học), trở thành một trong mấy người trẻ tuổi nhất được nhận vinh dự cao quý ấy. Terence Tao được coi là “Mozart trong toán học”.

Vũ Hà Văn kể lại:
Năm 2003, được ông Chủ tịch Hội Toán học Mỹ giới thiệu, tôi bắt đầu làm quen với Terence Tao. Năm ấy, Tao mới 28 tuổi, chưa được tặng Huy chương Fields. Anh sống với người vợ trẻ gốc Hàn Quốc trong một cản hộ hẹp tại quận Cam, bò ra sàn nhà làm toán. Cùng mang dòng máu châu A, nên chúng tôi dễ đồng cảm. về sau, qua trao đổi email, chúng tôi cảm thấy rất dễ hiểu những ý tưởng của nhau. Từ đấy, Tao và tôi cộng tác công bố được 15 bài báo khoa học và 1 cuốn sách chuyên khảo dày 500 trang. Riêng cuốn sách chúng tôi viết mất ba năm.

Đó là cuốn Additive Combinatorics (Tổ hợp cộng tính) của hai tác giả Terence TaoVan H. Vu, do Viện Nghiên cứu toán học cao cấp Đại học Cambridge (Anh) xuất bản năm 2006. Ben Green, một nhà toán học Anh rất nổi tiếng, làm việc tại Đai học Cambridge, viết. Ben Green cho biết thuật ngữ toán học tổ hợp cộng tính chỉ mới xuất hiện gần đây, do Terence Tao đặt ra, và đã trở thành một chuyên ngành toán học phát triển nhanh, mang lại nhiều hứng thú. Còn ít nhà toán học quen với thuật ngữ này, mặc dù họ rất quen những thành tựu cột mốc của nó. Sau khi phân tích ý nghĩa của cuốn sách, Ben Green kết luận:

Tóm lại, cuốn sách là một đóng góp quan trọng cho vãn liệu toán học và đã trở thành cuốn sách mà thế hệ sinh viên mới cần đọc cũng như những chuyên gia trong các lĩnh vực gần gũi cần học hỏi thêm về toán học tổ hợp cộng tính (chẳng hạn, chương 4 có thể coi là rất hấp dẫn đối với các nhà lý thuyết về khoa học tính toán). Đây là cuốn sách viết rất đúng lúc và hai tác giả của nó rất đáng được ngợi ca vì đã thể hiện một cách đầy thuyết phục. Riêng tôi, tôi có tới ba bản in: một bản để ở nhà, một để ở nơi làm việc, và bản thứ ba dự phòng trường hợp hai bản kia bị cũ nát.

Văn có nhiều quan hệ với người Hung và người Việt. Thày hướng dẫn luận án tiến sĩ của Văn là GS. L. Lovasz, người Hungary. Ông là nhà toán học lừng danh (sinh ngày 09/03, năm 1948; ba lần liên tiếp, 1964, 1965, 1966, đoạt huy chương Vàng Olympiad thế giới; con ông năm 2008 cũng đoạt được huy chương Vàng Olympiad). Ông được trao nhiều giải danh giá:

Kyoto Prize (2010)
Hungary's Szộchenyi Grand Prize (2008)
Bolyai prize (2007)
Gordel Prize (2001)
Wolf Prize (1999)
Fulkerson Prize (1982)
Best Information Theory Paper Award (IEEE) (1981)
Polya Prize (SIAM) (1979)

Ông đã từng giữ chức Chủ Tịch Hội Toán Học Quốc Tế (2007-2010), là viện sĩ của viện Hàn Lâm Thụy Điển, và viện Hàn Lâm Hoàng Gia Anh.

Ngoài ra, Vũ Hà Văn còn cộng tác nghiên cứu và đào tạo nhiều tiến sĩ người Việt và các nước khác (xem lý lịch khoa học của Vũ Hà Văn dưới đây).


Hình đã gửi

Từ trái qua phải GS Vũ Hà Văn, GS Đặng Hùng Thắng, GS Nguyễn Hữu Việt Hưng và GS Nguyễn Duy Tiến


Blog của Vũ Hà Văn
Ngày nay các nhà toán học thường dùng blog để trao đổi toán học với nhau và những người khác. Điều này làm cho các kết quả của toán học trở nên gần gũi hơn, và thường được phổ cập nhanh chóng. NgôbảoChâu có Blog "thích học toán" rất được nhiều ban trẻ quan tâm. Vũ Hà Văn có "Van's Blog" cũng thế. Xin được trích dẫn bài số đề của GS. Vũ Hà Văn:

Số Đỏ.
Dân ta thích đỏ đen. Không biết có tự bao giờ, nhưng số đề đang là trò “đỏ đen’’ được nhiều người ưu ái nhất, chơi nhiều nhất. Đêm nằm mơ số đề đẹp, sáng ra chợ nghe bàn, trưa đi đặt số, chiều đợi radio nghe xô’ số... đề. Luật chơi đề đợi loại như sau: Sáng bạn đặt một số tiền, nói đơn giản là 1 đô cho chủ đề, vào một số từ 00 đến 99. Mục đích của người chơi đề là làm sao số này trùng vào 2 chữ số cuối cùng của gidi sổ xổ do nhà nước phát hành trong ngày đó. Khi xổ số quay, hai chữ số này được xác định gọi là “đề về”), chủ đề so số và thanh toán tiền nong. Nếu sổ của bạn trùng, bạn sẽ được 70 đô (tức 70 lẩn số tiền đẩu tư). Nếu không trúng, bạn sẽ mất 1 đô đặt cược lúc đẩu. “Ai ơi yêu lấy số đề Khi đi một chỉ, khi về bảy cây !” Đánh đề thông dụng có lẻ bởi nó đơn giản dễ hiểu, và khả năng trúng xố, trong mắt người chơi là tương đối cao (1/100). Khả năng này cao hơn nhiều so với các giải xổ sốchính thức của nhà nước, và có tác dụng tâm lý rất mạnh. Những người chơi đề lâu ngày, thường ai cũng thắng, hoặc quen biết những người đã thắng một vài lần. Tâm lý chơi đề để có một cơ hội “đổi đời” rất phổ biến, nhất là đối với những người nghèo. Chuyện về “nuôi đề”, nằm mơ thấy “đề về”, đi thăm mộ thấy số đề, vv là những chuyện nghe thấy hàng ngày. Vậy thực chất Đánh Đề có phải là một trò chơi đem lại nhiều hy vọng? Nhìn về khía cạnh toán học mà nói, thì luật chơi đề rất thiệt cho ngươi chơi, vì kỳ vọng của nó là một số âm to đùng. Giả sử ông A chơi đề ngày một lẩn, mỗi lẩn đều đặn 1 triệu đồng. Như vậy sau 6 năm, tính là 2000 ngày cho chẵn, ông A bỏ ra 2 tỷ. Mỗi lẩn chơi, xác xuất trúng là 1%. Như thế, trung bình ông A sẽ trúng 20 lấn. Mỗi lẩn được 70 triệu, 20 lấn là 1,4 tỷ, như vậy, trung bình ông A lổ 600 triệu.

Tất nhiên, ông A sẽ nói “ờ thì trung bình là vậy, nhưng nhỡ tôi may thi sao ? Xác suất may của ông A hoàn toàn có thể tính được. Nó được biểu diễn qua một định lý rất nổi tiếng và cơ bán: Định lý giới hạn trung tâm: Nếu $X_1,X_2,...X_n$ là các biến độc lập ngẫu nhiên, có cùng kỳ vọng là $E$ và phương sai là $V=\sigma ^2$. Khi đó nếu $n$ tiến đến vô cùng
$$P(\frac{\sum^n_{i=1}X_i-nE}{\sqrt{n}\sigma } \geq x) \to \Phi(x),$$
ở đây
$$\Phi(x)=\frac{1}{\sqrt{2 \pi}} \int_{ x}^{\infty} e^{-t^2/2}dt$$
là hàm phân bố Gauss.


Điều quan trọng ở đây là hàm $e^{-t^2/2}$ tiến đến $0$ rất nhanh với $t$, do đó $\Phi (x)$ cũng tiến đến 0 rất nhanh với $x$. Chẳng hạn $\Phi (1) \leq 0,16$, $\Phi (2) \leq 0,03$, $\Phi(3) \leq 0,003.$
Định lý trên có thể viết dưới dạng
$$P(\sum _{i=1}^n X_i \geq nE+x \sqrt{n} \sigma ) \approx \Phi(x).$$
Quay trở lại với ông A. Muốn ứng dụng định lý trên, ta cho $X_i$ là số tiền ông $A$ thu hoạch trong lần chơi thứ $i$ có phân bố như sau: $P(X_i=-1)=0,99$ (thua) và $P(X_i=09)=0,01$ (thắng). Ký vọng của là $-0,3$ (triệu đồng) và phương sai xấp xỉ $49=7^2$. Nếu ông $A$ không lỗ sau 2000 lần chơi thì $\sum ^n_i=1 X_i \geq 0$, tức ta phải lấy

$$x \geq \frac{n \begin{vmatrix}E \end{vmatrix}}{\sqrt{2007}} \approx 1,9$$

Vậy xác suất để ông A "may" (không lổ vốn) là độ $\Phi(1, 9)$. Xác suât này cỡ ba phẩn trăm. Nói một cách nôm na, nếu có 100 người chơi như ông, trung bình chỉ có 3 người không lỗ. (Tất nhiên bạn được nghe ba ông này tuyên truyền về "tài năng" của mình bao nhiêu lẩn lại là chuyện khác. Đây là khía cạnh tâm lý của trò chơi.) Định lý giới hạn trung tâm phản ánh một hiện tượng quan trọng và có tính ứng dụng cao: Hiện tượng (Lơrge deviation): Một biến ngẫu nhiên được xác định bởi nhiều biến ngẩu nhiên độc lập thường lấy một giá trị gần với kỳ vọng của nó. Trong trường hợp vừa rồi, biến này là , tổng số thu hoạch của ông A. Bạn có thể dùng định lý giới hạn trung tâm để so sánh sổ đề với rullet, một trò chơi thông dụng ở casino. ở cả hai trò, kỳ vọng của người chơi đều âm, nhưng kỳ vọng của rullet là một số âm nhỏ.


Lý lịch khoa học của Vũ Hà Văn

Curriculum vitae V. H. Vu Name: Van H. Vu Contact information.
Department of Mathematics, 110 Frelinghuysen Road, Piscataway, NJ 08854, USA. Email: [email protected]">[email protected] 1 Referee. Editor: SIAM Joumal of Discrete Mathematics; Joumal of Combinatorial Theory A; Communications in Contemporary Mathematics; Acta Mathematica Viet- namica.
Referee: Combinatorica, Random Structures and Algorithms, Joumal of Combinato- rial Theory, Israel Math. Joumal, Duke Math. Joumal, CiAHA. J.A.M.S, Annals of Probability, Annals of Mathematics, Acta Mathematica etc.
Panelist: NSF panels (several times). Outside referee: NSA, Israel national research foundation, ERC Advanced Research Grants (EU).
Organizing. Random matrices, workshop, American Institute of Mathematics, Dec
2010 (with T. Tao).
Additive combinatorics, mini-symposium, SIAM Discrete Math, June 2010.
Random matrices, Dimacs workshop, Spring 2008, Rutgers.
Additive combinatorics, Focus Program, Fall 2007, 1AS (Princeton), (with J. Bour- gain).
Conference in Arithmetic Combinatorics, IAS, Dec 2007 (with J. Bourgain).
Special section in Probability and Combinatorics, AMS meeting, Rutgers, Oct 2007 (with J. Kahn).
Conference in Probabilistic Combinatorics, Bank Center, Nov. 2005 (with N. Alon, B. Reed and B. Sudakov).
Additive Combinatorics, section at AMS meeting, Santa Barbara, April 2005 (with M.c. Chang).
Conference in Additive Combinatorics, AIM, Palo Alto, September 2004 (with T. Tao). Probabilistic Combinatorics, section meeting at SIAM Discrete Math. conference, Au- gust 2004 (with B. Sudakov).
Probability in combinatorics and the Internet, section at the annual meeting of the AMS, San Diego, Jan 2002 (with F. Chung).
Publications. Books:
Additive Combinatorics (with T. Tao), 512 pages, Cambridge Univ. Press, 2006. Articles:
2011 (with Tao) Random matrices: Universality of local statistics (Acta Mathematica). 2010
(with Erdos et. al.) Bulk universality for Wigner hermitian matrices with subexponen- tial decay (Mathematics Research Letters).
(with T. Tao) Random matrices: The distribution of the smallest singular values (Uni- versality at the hard Edge) (To appear in CiAHAi
(with T. Tao) Random matrices: Universality of ESD and the Circular Law (with an appendix by M. Krishnapur) (Annals of Probabiliy)
(with H. Nguyen) Squares in Sumsets (Szemeredi is 70 proceeding).
(with K. Costello) Concentration of random determinants and permanent estimator (Siam Discrete mathematics).
2009
(with L. Tran and p. Wood) On a conjecture of Alon, J. Number Theory 129 (2009), no. 11,2801-2807.
(with p. Wood) The inverse Erd?s-Heilbronn problem, Electron. J. Combin. 16 (2009), no. 1,8 pages.
(with T. Tao) From the Littlewood-Oord problem to the cừcular law: universality of the spectral distribution of random matrices, Bull. Amer. Math. Soc. 46 (2009), no. 3, 377-396.
(with H. Nguyen) Classication theorems for sumsets modulo a prime, J. Combin. The- ory Ser. A 116 (2009), no. 4, 936-959.
(with T. Tao) Inverse Littlewood-Oord theorems and the condition number of random discrete matrices, Ann. of Math. (2) 169 (2009), no. 2, 595-632. 2 (with T. Tao) On the permanent of random Bemoulli matrices, Adv. Math. 220 (2009), no. 3, 657-669.
2008.
A structural approach to subset-sum problems. Building bridges, 525-545, Bolyai Soc. Math. Stud., 19, Springer, Berlin, 2008.
(with B. Sudakov) Local resilience of graphs, Random Structures Algorithms 33 (2008), no. 4, 409-433.
Random discrete matrices. Horizons of combinatorics, 257-280, Bolyai Soc. Math. Stud., 17, Springer, Berlin, 2008.
(with K Costello) The rank of random graphs, Random Structures Algorithms 33 (2008), no. 3, 269-285.
(with T. Tao) John-type theorems for generalized arithmetic progressions and iterated sumsets, Adv. Math. 219 (2008), no. 2, 428-449.
(with A. Johansson and J. Kahn) Factors in random graphs Random Structures and Algorithms, 33 (2008) no 1, 1-28.
(with T. Tao) Random matrices: The circular law, Communications in Contemporary Mathematics, 10 (2008), 261-307.
(with J. Solymosi) Near optimal bounds for the Erdos distinct distances problem in high dimensions, Combinatorica 28 (2008) 113-125. Sum-product estimates via di- rected expanders, Math. Res. Lett. 15 (2008), no. 2, 375-388.
(with R. Richardson and L. Wu) An inscribing model for random polytopes, Discrete Comput. Geom. 39 (2008), no. 1-3, 469-499.
(with E. Szemeredi and H. Nguyen) Sumset modulo p, Acta Arithmetica (2008), no. 4, 303-316.
2007.
(with I. Barany) Central limit theorems for Gaussian polytopes, Ann. Probab. 35 (2007), no. 4, 1593-1621.
Spectral norm of random matrices, Combinatorica 27 (2007), no. 6, 721-736.
(with T. Tao) On the singularity probability of random Bemoulli matrices, Joumal of the A. M. s, 20 (2007), 603-673.
Some new results 011 subset sums, J. Number Theory 124 (2007), no. 1, 229-233. (with J H. Kim and B. Sudakov) Small subgraphs of random regular graphs, Discrete Math. 307 (2007), no. 15, 1961-1967.
2006.
Central limit theorems for random polytopes in a smooth convex set, Advances in Mathematics 207 (2006) 221-243.
(with T. Tao) Qn random (-1,1) matrices: Singularity and Determinant, Random Struc- tures and Algorithms 28 (2006), no 1, 1-23.
(with Szemeredi) Long Arithmetic Progressions in Sumsets: Thresholds and Bounds, Joumal of the A.M.S, 19 (2006), no 1, 119-169.
(with J. H. Kim) Generating random regular graphs, Combinatorica 26 (2006), no. 6, 683-708.
(with K. Costello and T. Tao) Random symmetric matrices are almost surely nonsin- gular, Duke Math. J. 135 (2006), no. 2, 395-413.
(with E. Szemeredi) Finite and Infinite Arithmetic Progressions in Sumsets, Annals of Mathematics, 163 (2006), no 1, 1-35.
2005
(with L. Wu) Improving the Gilbert-Varshamov bounds for Sumsets: q-ary codes, IEEE Transaction in lnformation Theory 51 (2005), no 9, 3200-3208.
(with B. Sudakov and E. Szemeredi) On a problem of Erdos and Moser, Duke Math. Joumal, 129 (2005), no 1, 129-155.
(with J.H. Kim and J. Matousek) Discrepancy after adding a single set, Combinatorica
25 (2005), no 4, 499-501. 3
(with T. Szabo) k-wise intersecting theorems , Graphs and Combinatorics, 21 (2005), 147-161.
(with B. Sudakov and T. Szabo) A generalization of Turan theorem, Joumal of Graph Theory 49 (2005), 187-195.
Covering codes of arbitrary alphabets, Advances in Applied Math. 34 (2005), 65-70. (with E. Szemeredi) Long arithmetic progressions in sumsets and the number of x-free sets, Proceed- ing of London Math. Society, 90 (2005), 273-296.
Sharp concentration of random polytopes, CiAHA (2005), no 6, 1284-1328.
2004.
(with J. Solymosi) Distinct distances in high dimensional homogeneous sets: Towards a theory of geometric graphs,Contemp. Math. 342, AMS 259-263.
(with J.H. Kim) Sandwiching random graphs, Advances in Mathematics 188 (2004) 444-469.
(with J.H. Kim) Devide and Conquer Martingales and the number of triangles in a random graph, Random Structures and Algorithms 24 (2004), no. 2, 166-174.
2003.
(with H. Q Ngo) Clos networks and a generalized edge-coloring problem 011 bipartite graphs, S1AM J. Comput. 32 (2003), no. 4, 1040-1049.
(with J.H. Kim) Small complete arcs in projective planes, Combinatorica 23 (2003), no. 2,311-363.
(with T. Szabo) Turan's theorem for sparse random graphs, Random Structures and Algorithms, 23 (2003), no. 3, 225-234.
(with F. Chung and L. Lu) Eigenvalues of Power Law Graphs, Annals of Combinatoris 7 (2003), 21-33.
(with F. Chung and L. Lu) The spectra of random graphs with expected degrees, Pro- ceedings of National Academy of Sciences, 100, no. 11, (2003).
(with J.H. Kim) Generating random regular graphs (2003), STOC 2003 213-222. (with M. Krivelevich and B. Sudakov) Covering codes with improved density, IEEE Transaction 011 lnformation Theory, 49 (2003) 1812-1815.
(with M. Krivelevich, B. Sudakov and N.Wormald) Qn the probability of independent sets in random graphs, Random Structures and Algorithms (2003) Nol, 1-14.
(with N. Alon, B. Bollobas and J.H. Kim) Economical covers and geometric applica- tions, Proc. London Math. Soc. (3) 86 (2003) 273-301.
2002.
On sum of dependent random variables and applications in additive number theory, Number theory for the millennium, III (Urbana, IL, 2000),341-356, A K Peters, Nat- ick, MA, 2002.
On a problem of Gowers, Annals of Combinatorics (2002) 229-233.
An upper bound 011 the list chromatic number of locally sparse graphs, Combinatorics, Probability and Computing 11 (2002), 103-111.
(with M. Krivelevich and B. Sudakov) Sharp threshold of reliability, Combinatorics, Probability and Computing 11 (2002), 465-474.
(with J. H. Kim and B. Sudakov) On the asymmetry of random graphs and random regular graphs, Random Structures and Algorithms 21(2002), 216-224.
Concentration of non-Lipschitz functions and applications, Random Structures and Al- gorithms, 20 (3) (2002), 262- 316.
(with N. Alon and M. Krivelevich) Concentration of eigenvalue of random matrices, Israel Math. Joumal, 131 (2002), 259-267. High order complementary bases of primes, Integer 2 (2002).
( with M. Krivelevich) Approximating the independent number and the chromatic num- ber in expected polynomial time, Joumal of Comb. Optimization (2002) 143-155. 2001.
A large deviation result 011 the number of small subgraphs of a random graph, Com- binatorics, Probability and Computing, 10 (2001), no. 1, 79-94.
(with M. Kirivelevich, B. Sudakov and N. Wormald) Random regular graphs of high degree, Random Structures and Algorithms 18(2001), 346-363.
(with M. Krivelevich) The weak choice number of random hypergraphs, Joumal of Combinatorial Theory, Series B, 83 (2001), no. 2, 241-257.
(with I. Pak) On mixing of certain random walks, cuto phenomenon and Sharp thresh- old of random matroid processes, Discrete Applied Mathematics, 110 (2001), 251-272. 2000.
New bounds 011 nearly perfect matchings in hypergraphs:higher codegrees do help,Random Structures and Algorithms, 17 (1)(2000), 29-63.
(with J.H. Kim) Concentration of multi-variate polynomials and its applications, Com- binatorica, 20 (3) (2000), 417-434.
On the concentration of multi- variate polynomials with small expectation, Random Structures and Algorithms, 16 (4) (2000), 344-363.
On the choice number of random hypergraphs, Combinatorics, Probability and Com- puting 9, (2000), 79-95.
(with J. Kahn, J.H. Kim and L. Lovasz) The cover time, the blanket time, and the Matthews bound, FOCS (2000), 467-476.
On arenement of Waring's problem, Duke Math. Joumal,105, (1 )(2000), 107-134.
1999.
On some degree conditions which guarantee the upper bound of chromatic (choice) number of random graphs, Joumal of Graph Theory, 31, (1999), no. 3, 201-226.
( with c. Borg, J. Chayes, A.Frieze, J.H. Kim, p. Tetali and E. Vigoda, Torpid mixing of some MCMCalgorithms in statistical physics, FOCS (1999), 218-229.
Set systems with weakly restricted intersections,Combinatorica 19, (1999), no. 4, 567¬587.
1998 and earlier (prior to mv graduation).
On the infeasibility of training neural networks with small mean squared error, IEEE Transaction 011 lnformation Theory 44, (1998), no. 7, 2892-2900.
On a theorem of Ganter, Combinatorics,Probability and Computing 6, (1997), no. 2, 247-254.
(with D.Kozlov) Coins and Cones, J. of Combinatorial Theory, series A, 78 (1997), no. 1, 1-14.
(with Noga Alon)Anti- Hadamard matrices, coin weighing, threshold gates and in- decomposable hypergraphs, J. of Combinatorial Theory, series A, 79, (1997), no. 1, 133-160.
Extremal set systems with upper bounded odd intersections, Combinatorics, Probabil- Ếứy and Com- puting 13, (1997), no. 2, 197-208.
Small strongly regular r-full graphs, Combinatorica 16,(1996), no. 2, 295-299.
On the embedding of graphs into graphs with few eigenvalues, J. of Graph Theory, 22 (1996), no. 2, 137-149.
List of invited presentations
Szemeredi 70 workshop, Budapest, August 2010.
Workshop in Pseudorandomness, 1AS, June 2010 (P. Samak, A. Wigderson, J. Bour- gain).
Seminar in Probability, Stanford, April 2010 (A. Dembo).
Colloquium, UCSD, April 2010 (J. Balogh).
Colloquium, usc, April 2010 (S. Friedlander).
Seminar in Analysis, UCLA, April 2010 (T. Tao).
Seminar in Probability, NYU, Feb 2010.
AMS-KMS joint meeting, Plenary speaker, Seoul, Dec 2009. 5 Colloquium, Princeton, Dec 2009 (S. Klainerman).
Colloquium, Yale, Oct 2009 (P. Jones). Colloquium, Lehigh, Oct 2009 (J. Yukich). Seminar (Probability), Harvard, Oct 2009 (H-T Yau). Brandeis-Harvard-MIT-Northwestem Colloquium, Oct 2009 (M. Adler).
Workshop at IPAM (Combinatorics), October 2009 (Sudakov el. al.)
Colloquium, usc, October 2009 (S. H. Teng).
Workshop in Probabilistic Combinatorics, Bank, August 2009 (Sudakov et. al.) Colloquium, NTU (Singapore), July 2009 (Opening of the School of Sciences lecture). Colloquium, Hanoi University, June 2009 (H. M. Le).
Applied Math. Colloquium, MIT, May 2009 (A. Edelman).
Workshop 011 Probabilistics Combinatorics, Montreal , May 2009 (B. Reed et. al.). Columbia-Princeton Probability Day, Plenary Speaker, May 2009.
Dimacs workshop 011 property testing, Plenary Speaker, Spring 2009.
Seminar (Combinatorics), Princeton, Spring 2009 (J. Fox).
Seminar (Combinatorics), Columbia, Spring 2009 (M. Chudnovsky).
Seminar (Geometry), NYU, Spring 2009 (R. Pollack).
Colloquium, Emory, Oct 2008 (V. Rodl).
Colloquium, CMU, Oct 2008 (A. Frieze).

Bulding Bridges (Lovasz birthday conference), Renyi Institute, Budapest, August 2008. Seminar (Probability), Microsoữ Research, June 2008 (D. Wilson). Workshop 011 Com- binatorial Number Theory, CUNY, May 2008 (M. Nathanson).
Seminar (Probability), NYU, April 2008.
Workshop 011 Additive combinatorics, Fields Institute (Toronto), April 2008 (I. Laba et. al.).
AMS meeting in Bloomington, Probability Section, April 2008 (R. Lyons et. al.). Workshop 011 expanders, IPAM (UCLA), Feb 2008 (A. Wigderson et. al.). Oberwolfach meeting 011 Combinatorics, Plenary Speaker, Jan 2008 (J. Kahn et. al.) Colloquium, NYU, Nov 2007 (A. Venkatesh).
Seminar (Discrete Math), Princeton, Nov 2007 (P. Seymour). Plenary talk, 3rd Integer Conference, West Georgia, Oct 2007. Minicourse (3 lectures) 011 additive combina- torics and random matrices, IAS, Oct 2007.
Colloquium, Hanoi Institute of Mathematics, Hanoi, Vietnam, May 2007 (T. Ngo). Erdos lecture series (3 talks), Hebrew Univ., May 2007 (G. Kalai et. al.). Plenary talk, 13rd Conference 011 Random structures and Algorithms, Tel Aviv, June 2007 (Alon et. al.)
Workshop 011 Random matrices, Utah, June 2007 (B. Rider).
Seminar(Discrete Math/TCS), IAS, Spring 2007 (A. Razborov).
Seminar (Discrete Mathematics), Princeton, Nov 2006 (B. Sudakov).
Seminar (Computer Science), Yale, Nov 2006 (D. Spielman).
Seminar (Probability), CUNY, Oct 2006 (J. Rosen) Workshop 011 Large scale Graphs, Dimacs, Oct 2006 (Lovasz-Sudakov).
Planery talk, Horizons in Combinatorics, Balaton Lake, July 2006 (Lovasz et. al.)
6 Minicourse (2 lectures) 011 Littlewood-Oord problem, Horizons in Combinatorics, Budapest, July 2006 (Lovasz et. al.) Minicourse (3 lectures) 011 random polytopes, Phenomena in High Dimensions, Paris, June 2006 (Milman et. al.). Annual confer- ence 011 Phenomena in High Dimension, Paris, June 2006 (Milman et. al.).
Colloquium, Rutgers, Spring 2006. Minicourse (4 lectures) 011 sumsets and arithmetic progressions,
Workshop 011 Additive Combinatorics, Montreal, April 2006 (A. Granville, J. Soly- mosi).
Seminar (Discrete Mathematics), IAS, Spring 2006 (A. Razborov).
Workshop 011 Probalistic combinatorics, Dimacs , April 2006 (Nesetril et. al.)
Seminar (Probability), NYU, March 2006 (Dubedat).
Colloquium, Tech. Univ. Berlin, Berlin, Jan 2006 (G. Ziegler). Planery speaker, Bian- nual meeting 011 combinatorics, Oberwolfach, Jan 2006 (L. Lovasz and H.J. Prommel). Workshop 011 Algebraic combinatorics, Dimacs, Nov 2005 (J. Nesetril, F. Roberts). Seminar (Discrete Mathematics), Princeton, Oct 2005 (M. Chudnovsky).
Seminar (Discrete Mathematics), Microsoít Research, Summer 2005 (J. H. Kim). Seminar (Discrete Mathematics), Ohio State, January 2005 (N. Robertson).
Seminar (Discrete Mathematics), IAS, December 2004 (A. Razborov).
Seminar, Theory Group, Microsoữ Research, November 2004 (A. Naor).
Seminar (Discrete mathematics), Yale, September 2004 (G. Kalai).
Colloquium, Rutgers, September 2004 (J. Kahn).
Seminar (Discrete Mathematics), Hanoi Institute of Mathematics, August 2004 (T. Ngo).
Workshop in Convex Analysis, Bank Institute, July 2004 (organizer V. Milman et. al.). 2nd South American congress of Mathematicians, Cancun, June 2004) (thematic speaker, Combinatorics Section).
Conference Siam Discrete Mathematics, Nashville, June 2004, mini-symposium in Probabilistic Combinatorics (T. Bohman and B. Sudakov).
Conference Siam Discrete Mathematics, Nashville, June 2004, mini-symposium in Ex- tremal Combinatorics (D. Mubayi).
Seminar (Discrete mathematics), Microsoữ Research, June 2004 (L. Lovasz). Conference in Additive Number Theory, May 2004, CUNY (M. Nathanson). Seminar (Geometry), NYU, May 2004 (G. Pollack).
Colloquium, Indiana, April 2004 (R. Lyons).
Colloquium, Simon Hraser. March 2004 (L. Goddyns).
Seminar (Discrete Mathematics), UBC, March 2004 (J. Solymosi).
Joint Cal Tech-UCLA Analysis seminar, Feb 2004 (T. Tao)
Seminar (Discrete Mathematics), Hanoi Insitute of Mathematics, Jan 2004 (T. Ngo). Planery speaker, Biannual conference in Combinatorics, Oberwolfach, Jan 2004 (or- ganizer L. Lovasz and H. Promel).
Colloquium, Claremont College Community, October 2003 (M. 0'Neil).
Planery Speaker, Structural and Probabilistic Methods in Coloring, Bank, September
2003 (B. Reed and p. Seymours).
Seminar (Discrete mathematics), Microsoữ Research, August 2003 (J.H. Kim).
7 Seminar (Analysis), UCLA, May 2003 (T. Tao).
Colloquium, UCR, April 2003 (M-C. Chang). Planery Speaker, CombiTexas, Texas, April 2003 (C. Yan).
Seminar (Discrete mathematics and Theoretical Computer Science), Rutgers Univ, March 2003 (M. Saks).
Seminar (Discrete mathematics and Theoretical Computer Science), IAS, March 2003 (A. Razborov).
Seminar (Discrete mathematics and Theoretical Computer Science), Princeton Univ., March 2003 (B. Sudakov).
Seminar (Probability), UCLA, February 2003 (M. Biskup).
Workshop 011 Discrete Geometry, Dimacs, Rutgers, October 2002 (J. Pach).
S0IAM Discrete Mathematics Biennial meeting, mini-symposia 011 Probabilistic Com- binatorics, San Diego, August 2002.
Workshop 011 Geometric Phenomena of Large Dimension, Vancouver, July 2003 (M. Krivelevich, V. Milman, L. Lovasz).
Workshop 011 Measure "từ cấm", Vancouver, July 2003.
Seminar (Discrete Mathematics and Theoretical Computer Science), Princeton Univ., March 2002 (B. Sudakov).
Seminar (Discrete Mathematics and Theoretical Computer Science), IAS, March 2002
B. Sudakov).
Seminar (Discrete Mathematics), March 2002 (I. Pak).
Special Section 011 Probability and Combinatorics, AMS meeting, Atlanta, March 2002 (R. Lyons).
Seminar (Geometry), Hanoi Institute of Mathematics, January 2002 (T. Ngo).
Seminar (Discrete Mathematics), Microsoữ Research, Redmond, January 2002 (J.H. Kim).
Special Section 011 Probability and the Internet Graph, AMS annual meeting, San Diego, January 2002.
Oberwolfach workshop 011 combinatorics, Germany, January 2002 (H. Prommel and L. Lovasz).
Oberwolfach workshop 011 Lnite geometry, Germany, December 2001 (A. Blockhuis). Colloquium talk, University of Illinois at Urbana-Champaign, February 2001 (D. West). Colloquium talk, University of Washington, February 2001 (V. Klee).
Colloquium talk, Camegie Melon University, November 2000 (A. Frieze).
Colloquium talk, Georgia Technical Institute, November 2000 (D. Randall). Colloquium talk, University of Michigan at Ann Arbor, October, 2000 (A. Barvinok). Probabilistic Methods in Combinatorial Optimization, BRICS, Aarhus, Denmark, Au- gust 2000 (A. Panconesi).
Research program 011 complexity, 1AS, Princeton, August 2000.
Search and Communication Complexity, Budapest, Hungary, July 2000 (G. Katona). lOth SIAM
Conference in Discrete Mathematics, Mini-symposia in coloring, Minnesota, June
2000 (M. Albertson).
Millennial Conference 011 Number Theory, Illinois, May 2000 (H. Halberstam). Probabilistic Method, Fields Institute, Toronto, Canada, February 2000 (M. MolloyB. Reed).
Combinatorics, Oberwolfach, Germany, January 2000 (H. Prommel and L. Lovasz). Probabilistic Analysis of Hard Problems, DIMACS, New Jersey, November 1999. Special section 011 the Probabilistic Method, Annual AMS meeting, Texas, January
1999 (B. Bollobas J.H. Kim).
Workshop 011 Randomized and Derandomized algorithms for Discrete Structures, In- stitute for Advance Study, November 1998.
Combinatorists of New England, Smith College, 1998 (M. Albertson).
Workshop 011 analysis and combinatorics, Stockholm, Sweden, May 1995 (A. Bjoner). Workshop 011 combinatorics and optimization, Budapest, Hungary, 1994 (R. Andras). Workshop 011 combinatorics and optimization, Comell 1992 (R. Andras).
Contributed Talks RANDOM 2009, Berkeley, August 2009.
STOC 2007, San Diego, June 2007.
STOC 2005 (2 lectures 011 random matrices), Baltimore, May 2005.
STOC, San Diego, June 2003.
FOCS, Redondo Beach, November 2001. SIAM Discrete Mathematics, Toronto 1998. FOCS, Burlington, 1997.


Tài liệu tham khảo
1) Hàm Châu, Sau Ngô Bảo Châu là nhà toán học Vũ Hà Văn?.
2) Google.
3) Vũ Quần Phương, Đợi



#323520 Về một bài toán bất đẳng thức

Đã gửi bởi Ban Biên Tập on 08-06-2012 - 22:01 trong Chuyên đề toán THPT

Bài toán 1.21 (Nguyễn Anh Khoa). Cho $a, b, c$ là ba số thực dương. Chứng minh bất đẳng thức
$$max \begin{Bmatrix} (a - 1)^2, (b - 1)^2, (c - 1)^2 \end{Bmatrix} \geq \frac{2}{3}(1 - a)(1 - b)(1 - c).$$
Lời Giải. Sử dụng bất đẳng thức
$$max \begin{Bmatrix} (a - 1)^2, (b - 1)^2, (c - 1)^2 \end{Bmatrix} \geq \frac{(a-1)^2+(b-1)^2+(c-1)^2}{3},$$
ta đưa bài toán về chứng minh bất đẳng thức mạnh hơn là
$$(a - 1)^2 + (b - 1)^2 + (c - 1)^2 \geq 2(1 - a)(1 - b)(1 - c),$$
tương đương với
$$a^2 + b^2 + c^2 + 2abc + 1 \geq 2(ab + bc + ca).$$
Bất đẳng thức cuối cùng hiển nhiên đúng. Bài toán được chứng minh xong. Đẳng thức xảy ra khi và chỉ khi a = b = c = 1 □


Bài toán 1.22 (Vasile Cirtoaje). Cho $a, b, c$ là ba số thực dương. Chứng minh rằng
$$4 (a+\frac{1}{a})(b+\frac{1}{b}) (c+\frac{1}{c}) \geq 9(a+b+c).$$
Lời Giải. Ta đặt $a=\frac{\sqrt{2}}{x}, b=\frac{\sqrt{2}}{y}, c=\frac{\sqrt{2}}{z}$ Bất đẳng thức cần chứng minh được viết lại như sau
$$(x^2 + 2)(y^2 + 2)(z^2 + 2) \geq 9(xy + yz + zx).$$
Đây chính là bất đẳngthức (1.5.1), nên ta có điều phải chứng minh.
Đẳng thức xảy ra khi và chỉ khi $a = b = c = \sqrt{2}$. □


Bài toán 1.23 (Phạm Kim Hùng). Cho $a, b, c, d$ là bốn số thực không âm thỏa mãn điều kiện $a + b + c + d = 4$. Chứng minh bất đẳng thức
$$(a^2 + 2)(b^2 + 2)(c^2 + 2)(d^2 + 2) \geq 81$$
Lời Giải. Không mất tính tổng quát của bài toán, ta có thể giả sử d = min{a, b, c, d}, khi đó ta có 0 - d - 1. Sử dụng bất đẳng thức (1.6.1), ta được
$$(a^2 + 2)(b^2 + 2)(c^2 + 2)(d^2 + 2) \geq 3(a + b + c)^2(d^2 + 2) = 3(4 - d)^2(d^2 + 2).$$
Mặt khác, ta lại có
$$3(4 - d)^2(d^2 + 2) - 81 = 3(1 - d)^3(5 - d) \geq 0, $$
vì vậy mà ta được
$$(a^2 + 2)(b^2 + 2)(c^2 + 2)(d^2 + 2) \geq 81.$$
Bài toán được chứng minh xong. Đẳng thức xảy ra khi và chỉ khi $a = b = c = d = 1.$


Bài toán 1.24 (Nguyễn Văn Huyện). Với mọi số thực $a, b, c$ thay đổi bất kỳ thỏa mãn điều kiện
$$\frac{1}{a^2+b^2+4}+\frac{1}{b^2+c^2+4}+\frac{1}{c^2+a^2+4} \geq \frac{2}{3}$$
hãy chứng minh bất đẳng thức
$$ab + bc + ca \leq \frac{3}{4}$$
Lời Giải. Do $a^2 + b^2 \geq \frac{(a+b)^2}{2}$ , nên từ giả thiết, ta có
$$\frac{1}{(a+b)^2+8}+\frac{1}{(b+c)^2+8}+\frac{1}{(c+a)^2+8} \geq \frac{1}{3}$$
Đặt $x = a + b, y = b + c, z = c + a$, ta có thể viết bất đẳng thức (1.24.1) lại thành
$$\frac{1}{x^2+8}+\frac{1}{y^2+8}+\frac{1}{z^2+8} \geq \frac{1}{3}$$
Bằng cách sử dụng kết quả "Bài toán 1.16", ta có ngay $x + y + z \leq 3$, hay là
$$a + b + c \leq \frac{3}{2}.$$
Mặt khác, theo bất đẳng thức quen thuộc $(a + b + c)^2 \geq 3(ab + bc + ca)$, ta lại có
$$\frac{9}{4} \geq (a + b + c)^2 \geq 3(ab + bc + ca),$$
từ đó suy ra được
$$ab + bc + ca \leq \frac{3}{2},$$
Bài toán được chứng minh xong. Đẳng thức xảy ra khi và chỉ khi $a = b = c = \pm 1$.


Bài toán 1.25. Cho $x,y,z$ là ba số thực không âm. Chứng minh bất đẳng thức
$$x^2 + y^2 + z^2 + \sqrt{3xyz(x + y + z)} \geq 2(xy + yz + zx).$$

Lời Giải. Bài toán có được bằng cách cộng ngược chiều hai bất đẳng thức sau đây
$$x^2 + y^2 + z^2 \geq (xy + yz + zx),$$
$$xy + yz + zx \geq \sqrt{3xyz(x + y + z)}.$$
Nếu $xyz = 0$ thì bài toán trở nên tầm thường nên ta chỉ cần xét $xyz \geq 0$. Để ý rằng bất đẳng thức cần chứng minh ở dạng thuần nhất nên ta có thể chuẩn hóa cho $xyz = 1$, và viết nó lại như sau
$$x^2 + y^2 + z^2 + \sqrt{3(x + y + z)} \geq 2(xy + yz + zx).$$
Mặt khác, theo bất đẳng thức AM-GM, thì
$$x + y + z \geq 3.$$
Vì thế để chứng minh bài toán thì ta cần chứng minh được
$$x^2 + y^2 + z^2 + 3 \geq 2(xy + yz + zx),$$
hay là
$$x^2 + y^2 + z^2 + 2xyz + 1 \geq 2(xy + yz + zx).$$
Đây chính là bất đẳng thức (1) nên ta có điều phải chứng minh. Trong trường hợp tổng quát, ta có đẳng thức xảy ra khi và chỉ khi $x = y = z$ hoặc $x = y, z = 0$ cùng các hoán vị. □


Bài toán 1.26. Cho $a, b, c$ là ba số thực dương. Chứng minh rằng
$$1+\frac{a}{b}+\frac{b}{c}+\frac{c}{a} \geq 2\sqrt{1+\frac{a}{c}+\frac{b}{a}+\frac{c}{b}}.$$
Lời Giải. Đặt
$$x=\frac{a}{b}, y=\frac{b}{c}, z=\frac{c}{a}$$
thì $x, y, z$ là các số dương và $xyz = 1$, khi đó bất đẳng thức trở thành
$$1+ x + y + z \geq 2\sqrt{1+\frac{1}{x}+\frac{1}{y}+\frac{1}{z}}.$$
hay là
$$1 + x + y + z \geq 2\sqrt{1 + yz + zx + xy.}$$
Bình phương hai vế và thu gọn lại, ta được
$$x^2 + y^2 + z^2 + 2(x + y + z) \geq 3 + 2(xy + yz + zx).$$
Vì $xyz = 1$ nên theo bất đẳng thức AM-GM, ta có
$$x + y + z \geq 3,$$
Vì thế để chứng minh bất đẳng thức trên thì ta cần chứng minh được
$$x^2 + y^2 + z^2 + x + y + z \geq 2(xy + yz + zx).$$
Đây chính là bất đẳng thức $(1.1.1)$, nên ta có điều phải chứng minh. □


Bài toán 1.27 (Trần Nam Dũng). Tìm số thực $k$ lớn nhất sao cho bất đẳng thức sau đúng với mọi $a, b, c$ dương
$$(a^2 + 2)(b^2 + 2)(c^2 + 2) \geq k(a^2 + b^2 + c^2) + (9 - k)(ab + bc + ca).$$
Lời Giải. Cho $a = b = 0,$ khi đó ta sẽ được
$$4(c^2 + 2) \geq kc^2.$$
Từ đó dễ dàng suy ra $k \leq 4$. Với $k = 4$, ta được bất đẳng thức
$$(a^2 + 2)(b^2 + 2)(c^2 + 2) \geq 4(a^2 + b^2 + c^2) + 5(ab + bc + ca),$$
tương đương với
$$a^2b^2c^2 + 2(a^2b^2 + b^2c^2 + c^2a^2) +8 \geq 5(ab + bc + ca), \ \ (1.27.1)$$
Đặt $x = ab, y = bc, z = ca$ thì bất đẳng thức $(1.27.1)$ trở thành
$$xyz + 2(x^2 + y^2 + z^2) + 8 \geq 5(x + y + z).$$
Đây chính là bất đẳng thức $(1.12.1)$ nên nó hiển nhiên đúng. Vậy $k = 4$, là giá trị lớn nhất cần tìm. □


Qua những ví dụ trên chắc hẳn các bạn cũng đã phần nào thấy được ứng dụng rộng rãi của bất đẳng thức (1), một bài toán tuy đơn giản nhưng những ứng dụng của nó thì thì quả thật không đơn giản tí nào. Chúng ta hãy cùng xem lại bài toán này một lần nữa Cho a, b, c là các số thực dương thay đổi bất kỳ. Chứng minh rằng
$$a^2 + b^2 + c^2 + 2abc + 1 \geq 2(ab + bc + ca).$$


Từ bài toán trên ta có thể sáng tạo ra hàng loạt bài toán liên quan khác, khó hơn như các bạn đã thấy. Đó chính là những bí ẩn sau vẻ đẹp của mỗi bài toán.

Trong bất đẳng thức điều kiện để một bất đẳng thức trở thành một bài toán hay thì điều đầu tiên là nó phải đẹp không có điều kiện rắc rối phức tạp và ở dạng chuẩn và ta phải Giải quyết được bài toán tổng quát của nó. Bất đẳng thức (1) cũng giống như thế, ta cũng có kết quả tổng quát của nó như sau

Tổng Quát. Với $x_i,x_2, ...,x_n (n \geq 2)$ là các số thực không âm, khi đó ta luôn có
$$(n - 1)(x_1 + x_2 + ... + x_n) + 2x_1x_2...x_n + n - 2 \geq (x_1 + x_2 + ... + x_n)^2$$



#322066 Về một bài toán bất đẳng thức

Đã gửi bởi Ban Biên Tập on 03-06-2012 - 16:45 trong Chuyên đề toán THPT

Bài toán 1.13. (Algebraic Inequalities Old and New Methods)
Cho x, y, z là ba số thực không âm thỏa mãn điều kiện $xy + yz + zx = 3$. Chứng minh bất đẳng thức
$$\frac{1}{x^2+2}+\frac{1}{y^2+2}+\frac{1}{z^2+2} \leq 1.$$
Lời Giải. Ta có bất đẳng thức cần chứng minh tương đương với
$$(x^2 + 2)(y^2 + 2)(z^2 + 2) \geq (x^2 + 2)(y^2 + 2) + (y^2 + 2)(z^2 + 2) + (z^2 + 2)(x^2 + 2).$$
Khai triển trực tiếp ra, ta được $$x^2y^2 + y^2z^2 + z^2x^2 + x^2y^2z^2 \geq 4.$$
Đặt $a = xy, b = yz$ và $c = zx$ thì $a + b + c = 3.$ Bất đẳng thức cần chứng minh trở thành $$a^2 + b^2 + c^2 + abc \geq 4.$$
Đây chính là bất đẳng thức $(1.10.1)$, nên nó hiển nhiên đúng. Bài toán được chứng minh xong. Đẳng thức xảy ra khi và chỉ khi $a = b = c = 1$, tức là $x = y = z = 1.$

Bài toán 1.14. Cho $a, b, c$ là ba số thực dương. Chứng minh bất đẳng thức:
\[\left( {{x^2} + 3} \right)\left( {{y^2} + 3} \right)\left( {{z^2} + 3} \right) \ge \frac{3}{4}{\left( {xy + yz + zx + \frac{{xyz}}{3}} \right)^2}\]
Lời Giải. Chia hai vế của bất đẳng thức cho $x^2y^2z^2$, ta có thể viết nó lại như sau
\[\left( {\frac{9}{{{x^2}}} + 3} \right)\left( {\frac{9}{{{y^2}}} + 3} \right)\left( {\frac{9}{{{z^2}}} + 3} \right) \ge 4{\left( {\frac{3}{x} + \frac{3}{y} + \frac{3}{z} + 1} \right)^2}\]
Đến đây bằng cách đặt $a = \frac{3}{x}, b = \frac{3}{y}$ và $c = \frac{3}{z}$, ta đưa bài toán về chứng minh
$$(a^2 + 3)(b^2 + 3)(c^2 + 3) \geq 4(a + b + c + 1)^2\,\,\,\,\,\, (1.14.1)$$
Khai triển trực tiếp bất đẳng thức này, ta được
$$5(a^2 + b^2 + c^2) + 3(a^2b^2 + b^2c^2 + c^2a^2) + a^2b^2c^2 + 23 \geq 8(a + b + c + ab + bc + ca)$$
Theo bất đẳng thức $(1.5.1)$ thì: $$a^2 + b^2 + c^2 + a^2b^2c^2 + 2 \geq 2(ab + bc + ca),$$
vì thế để chứng minh được bài toán ta cần chứng minh được
$$4(a^2 + b^2 + c^2) + 3(a^2b^2 + b^2c^2 + c^2a^2) +21 \geq 8(a + b + c) + 6(ab + bc + ca)$$
Bằng một số biến đổi đơn giản ta có bất đẳng thức này tương đương với
$$3 [(ab - 1)^2 + (bc - 1)^2 + (ca - 1)^2] +4 [(a - 1)^2 + (b - 1)^2 + (c - 1)^2] \geq 0,$$
là một bất đẳng thức hiển nhiên đúng. Bài toán được chứng minh xong. Đẳng thức xảy ra khi và chỉ khi $a = b = c = 1$ hay $x = y = z = 3$.

Bài toán 1.15. (Nguyễn Văn Huyện)
Với $a, b, c$ là ba số thực dương. Chứng minh rằng khi đó với mọi số dương $k \geq 2$, ta luôn có
$$(a^2 + k)(b^2 + k)(c^2 + k) \geq (k + 1)(a + b + c + k - 2)^2\,\,\,\,\,\,\,\, (1.15.1)$$
Lời Giải. Sử dụng bất đẳng thức $(1.8.1)$, ta cần chứng minh bất đẳng thức mạnh hơn là
$$\frac{(k+1)^2}{3}(a + b + c)^2 + k^3 - 3k - 2 \geq (k + 1)(a + b + c + k - 2)^2.$$
Bất đẳng thức này tương đương với
$$\frac{(k+1)^2}{3}(a + b + c)^2 + (k + 1)^3 - 3(k + 1)^2 \geq (k + 1)(a + b + c + k - 2)^2$$
$$\frac{k+1}{3}(a + b + c)^2 + (k + 1)^2 - 3(k + 1) \geq (a + b + c + k - 2)^2$$
$$\frac{k+1}{3}(a + b + c)^2 + (k + 1)^2 - 3(k + 1) \geq (a + b + c + k - 2)^2$$
$$\frac{k + 1}{3}(a + b + c)^2 + k^2 - k - 2 \geq (a + b + c)^2 + 2(k - 2)(a + b + c) + (k - 2)^2$$
$$\frac{k-2}{3}(a + b + c)^2 + 3(k - 2) \geq 2(k - 2)(a + b + c).$$
Bất đẳng thức cuối cùng đúng theo bất đẳng thức AM-GM nên ta có điều phải chứng minh.

NHẬN XÉT. Trong $(1.15.1)$ nếu ta cho $k = 2$ thì ta được $(1.6.1)$ còn nếu ta cho $k = 3$ thì ta được $(1.14.1).$

Bài toán 1.16. Với $a, b, c$ là ba số thực bất kỳ sao cho $$\frac{1}{a^2+8}+\frac{1}{b^2+8}+\frac{1}{c^2+8}=\frac{1}{3}$$
Hãy tìm giá trị lớn nhất và giá trị nhỏ nhất của biểu thức: $P = a + b + c. $

Lời Giải. Ta viết biểu thức điều kiện của bài toán lại như sau
\[\frac{1}{{{a^2} + 8}} = \left( {\frac{1}{6} - \frac{1}{{{b^2} + 8}}} \right) + \left( {\frac{1}{6} - \frac{1}{{{c^2} + 8}}} \right)\]
hay là \[\frac{1}{{{a^2} + 8}} = \frac{1}{6}\left( {\frac{{{b^2} + 2}}{{{b^2} + 8}} + \frac{{{c^2} + 2}}{{{c^2} + 8}}} \right)\]
Từ đó theo bất đẳng thức AM-GM, ta được
\[\frac{1}{{{a^2} + 8}} = \frac{1}{6}\left( {\frac{{{b^2} + 2}}{{{b^2} + 8}} + \frac{{{c^2} + 2}}{{{c^2} + 8}}} \right) \ge \frac{1}{3}\sqrt {\frac{{({b^2} + 2)({c^2} + 2)}}{{{b^2} + 8)({c^2} + 8)}}} \,\,\,\,\,\,\,{\mkern 1mu} {\mkern 1mu} {\mkern 1mu} {\mkern 1mu} {\mkern 1mu} {\mkern 1mu} {\mkern 1mu} (1.16.1)\]
Hoàn toàn tương tự, ta cũng có
$$\frac{1}{b^2+8} \geq \frac{1}{3} \sqrt{\frac{(c^2+2)(a^2+2)}{(c^2+8)(a^2+8)}}\, \, \, \,\,\,\,\,\,\,\,\,\,(1.16.2)$$
$$\frac{1}{c^2+8} \geq \frac{1}{3} \sqrt{\frac{(a^2+2)(b^2+2)}{(a^2+8)(b^2+8)}}\, \, \, \,\,\,\,\,\,\,\,\,\,(1.16.3)$$
Nhân tương ứng ba bất đẳng thức $(1.16.1), (1.16.2)$ và $(1.16.3)$ lại với nhau ta được
$$27 \geq (a^2 + 2)(b^2 + 2)(c^2 + 2).$$
Mặt khác, theo bất đẳng thức $(1.6.1)$ thì $(a^2 + 2)(b^2 + 2)(c^2 + 2) \geq 3(a + b + c)^2.$

Nên từ đó suy ra $(a + b + c)^2 \leq 9$

hay là $$-3 \leq a + b + c \leq 3\,\,\,\,\,\,\,\,\, (1.16.4)$$
Bằng tính toán trực tiếp ta thấy $P = -3$ khi và chỉ khi $(a, b, c)=(-1, -1, -1)$ và $P = 3$ khi và chỉ khi $(a, b, c)=(1,1,1)$.

Việc tìm được các giá trị cụ thể của $a, b, c$ thỏa mãn giả thiết của bài toán đồng thời bất đẳng thức $(1.16.4)$ trở thành đẳng thức cho phép ta kết luận $P_{min} = -3$ và $P_{max} = 3$.

NHẬN XÉT. Ta có bài toán tổng quát của bất đẳng thức trên như sau:

Cho $a, b, c$ là ba số thực bất kỳ và $k \geq 2$ là một số dương cho trước thỏa mãn điều kiện
$$\frac{1}{a^2+3k+2} \frac{1}{b^2+3k+2} \frac{1}{c^2+3k+2}=\frac{1}{k+1}$$
khi đó hãy tìm giá trị lớn nhất và giá trị nhỏ nhất của biểu thức $P = a + b + c.$

Lời Giải. Tương tự như trên ta viết biểu thức điều kiện của bài toán lại như sau
\[\frac{1}{{{a^2} + 3k + 2}} = \frac{1}{{2(k + 1)}}\left( {\frac{{{b^2} + k}}{{{b^2} + 3k + 2}} + \frac{{{c^2} + k}}{{{c^2} + 3k + 2}}} \right)\]
Từ đó sử dụng bất đẳng thức AM-GM, ta được
$$\frac{1}{a^2+3k+2} \geq \frac{1}{k+1} \sqrt{\frac{(b^2+k)(c^2+k)}{(b^2+3k+2)(c^2+3k+2)}}.$$
Đánh giá tương tự cho hai bất đẳng thức còn lại sau đó nhân tương ứng theo vế lại với nhau, ta thu được
$$(k + 1)^3 \geq (a^2 + k)(b^2 + k)(c^2 + k).$$
Kết hợp với bất đẳng thức $(1.8.1)$, ta có
$$(k + 1)^3 \geq \frac{(k + 1)^2}{2} (a + b + c)^2 + k^3 - k - 2$$
Biến đổi đơn giản hai vế, ta được
$$(a + b + c)^2 - 9.$$
Từ đó suy ra $P_{min} = -3$ và $P_{max} = 3$.

Bài toán 1.17. (Nguyễn Anh Khoa)
Cho $a, b, c$ là ba số thực dương thỏa mãn điều kiện $2(a^2 + b^2 + c^2) + abc = 7$. Chứng minh bất đẳng thức
$$a + b + c \leq 3.$$
Lời Giải. Biểu thức điều kiện của bài toán có thể viết lại như sau
$$15 = 2(a^2 + b^2 + c^2) + [a^2 + b^2 + c^2 + (a^2 + b^2 + c^2 + 2abc + 1)].$$
Bằng cách sử dụng bất đẳng thức $(1)$ và bất đẳng thức quen thuộc $(a^2 + b^2 + c^2) \geq \frac{1}{3}(a + b + c)^2$, ta có
$$15 \geq \frac{2}{3}(a + b + c)^2 + [a^2 + b^2 + c^2 + 2(ab + bc + ca)] = \frac{5}{3}(a + b + c)^2$$
Từ đó bằng cách lấy căn bậc hai hai vế, ta được $a + b + c \leq 3.$

Bài toán được chứng minh xong. Đẳng thức xảy ra khi và chỉ khi $a = b = c = 1$.

NHẬN XÉT. Bằng cách làm tương tự ta chứng minh được bài toán tổng quát sau đây
Nếu $a, b, c$ và $k \geq 1$ là các số thực dương thỏa mãn điều kiện $k(a^2 + b^2 + c^2) + abc = 3k + 1$, thì
$a + b + c \leq 3.$

(Nguyễn Văn Huyện)

Lời Giải. Thật vậy, biểu thức điều kiện có thể được viết lại như sau
$$3(2k + 1) = (2k - 2)(a^2 + b^2 + c^2) + [a^2 + b^2 + c^2 + (a^2 + b^2 + c^2 + 2abc + 1)],$$
Từ đó sử dụng bất đẳng thức $(1)$ và bất đẳng thức cơ bản $a^2 + b^2 + c^2 \geq \frac{1}{3}(a + b + c)^2$, ta có
$$3(2k + 1) \geq \frac{2k-2}{3} (a + b + c)^2 + (a + b + c)^2 = \frac{2k+1}{3}(a + b + c)^2\,\,\,\,\,\,\,\,\, (1.17.1)$$
Bằng cách lấy căn hai vế, ta có ngay điều phải chứng minh.

Ngoài ra ta cũng có bài toán đảo của bất đẳng thức trên như sau.

Nếu $a, b, c$ là ba số thực dương thỏa mãn điều kiện $a + b + c = 3$, khi đó với mọi số thực $ k \geq 1$ ta luôn có bất đẳng thức
$$k(a^2 + b^2 + c^2) + abc \geq 3k + 1. $$
Lời Giải. Bất đẳng thức cần chứng minh tương đương với
$$(k - 1)(a^2 + b^2 + c^2) + (a^2 + b^2 + c^2 + abc) \geq 3k + 1$$
Dễ thấy, bài toán có được bằng cách cộng hai bất đẳng thức sau lại với nhau
$$a^2 + b^2 + c^2 \geq 3$$
$$a^2 + b^2 + c^2 + abc \geq 4.$$
Những đây là những kết quả mà ta đã biết.

Bài toán 1.18. Cho $a, b, $ là ba số thực dương. Chứng minh bất đẳng thức
$$3(a^2 + b^2 + c^2) + abc + 11 \geq 7(a + b + c).$$
Lời Giải. Bất đẳng thức cần chứng minh tương đương với
$$2 [(a - 1)^2 + (b - 1)^2 + (c - 1)^2] + [a^2 + b^2 + c^2 + abc + 5 - 3(a + b + c)] \geq 0$$
là một kết quả hiển nhiên đúng theo bất đẳng thức $(1.11.1)$, nên ta có điều phải chứng minh. Đẳng thức xảy ra khi và chỉ khi $a = b = c = 1$.

Bài toán 1.19. (Nguyễn Văn Huyện)
Với $a, b, c$ là ba số thực dương. Chứng minh rằng với mọi số thực dương $ k \geq 1$, ta luôn có bất đẳng thức
$$k(a^2 + b^2 + c^2) + abc + 3k + 2 \geq (2k + 1)(a + b + c)\,\,\,\,\,\,\,\,\,\,\, (1.19.1)$$
Lời Giải. Nhân $2$ vào hai vế của bất đẳng thức, ta có thể viết nó lại như sau
$$(2k - 2)(a^2 + b^2 + c^2) + [a^2 + b^2 + c^2 + (a^2 + b^2 + c^2 + 2abc + 1)] + 3(2k + 1) \geq 2(2k + 1)(a + b + c)\,\,\,\,\,\,\,\,\,\,(1.15.1)$$
Ở $(1.17.1)$, ta đã chứng minh được
$$(2k - 2)(a^2 + b^2 + c^2) + [a^2 + b^2 + c^2 + (a^2 + b^2 + c^2 + 2abc + 1)] \geq \frac{2k+1}{3} (a + b + c)^2.$$
nên để chứng minh bất đẳng thức trên thì ta chỉ cần chỉ ra được
$$\frac{2k+1}{3}(a + b + c)^2 + 3(2k + 1) \geq 2(2k + 1)(a + b + c)$$
Là bất đẳng thức hiển nhiên đúng theo bất đẳng thức AM-GM. Chứng minh hoàn tất.

NHẬN XÉT. Tập hợp tất cả các giá trị của $k$ để $(1.19.1)$ luôn đúng là $k \geq \frac{1}{\sqrt{2}}$, tuy nhiên với $k \geq 1$ thì ta nhận được lời giải bằng cách sử dụng bất đẳng thức $(1)$. Đặc biệt với $k = \frac{1}{\sqrt{2}}$ thì ngoài trường tầm thường $a = b = c =1$ để bất đẳng thức xảy ra thì ta còn có thêm trường hợp khác nữa là $a = b = 1 + \frac{1}{\sqrt{2}}, c = 0$ cùng các hoán vi.

Bài toán 1.20. Cho $a, b, c$ là ba số thực dương. Chứng minh bất đẳng thức
$$(a^2 + 1)(b^2 + 1)(c^2 + 1) + 4 \geq 4(ab + bc + ca) + (abc - 1)^2. $$
Lời Giải. Sử dụng bất đẳng thức AM-GM, ta có:
$$(ab + 1)^2 + (bc + 1)^2 + (ca + 1)^2 \geq 4(ab + bc + ca)$$
Từ đó đưa bài toán về chứng minh bất đẳng thức mạnh hơn là
$$(a^2 + 1)(b^2 + 1)(c^2 + 1) + 4 \geq (ab + 1)^2 + (bc + 1)^2 + (ca + 1)^2 + (abc - 1)^2,$$
Tương đương với
$$a^2 + b^2 + c^2 + 2abc + 1 \geq 2(ab + bc + ca).$$
Nhưng đây chính là bất đẳng thức $(1)$ nên nó hiển nhiên đúng, tức ta có điều phải chứng minh. Đẳng thức xảy ra khi và chỉ khi $a = b = c = 1$


Còn tiếp...



#321642 Tìm số hạng tổng quát của dãy số bằng hàm Hypebolic

Đã gửi bởi Ban Biên Tập on 02-06-2012 - 07:58 trong Chuyên đề toán THPT

TÌM CÔNG THỨC TÍNH SHTQ CỦA DÃY SỐ BẰNG HÀM HYPEBOLIC

DƯ QUỐC ĐẠT

Giáo viên trường THPT Nguyễn Hữu Cầu, huyện Hốc môn, TP Hồ Chí Minh


I. Lời nói đầu:

Bài toán tìm công thức tính số hạng tổng quát của một dãy số cho dưới dạng truy hồi là một bài toán rất đa dạng và thường gặp trong những đề thi Olympic, thi học sinh giỏi quốc gia, quốc tế. Có nhiều hướng tiếp cận bài toán này và có nhiều cách giải khác nhau.


Trong bài viết này tôi giải quyết bái toán bằng cách sử dụng các hàm hypebolic .

Tác giả rất mong nhận được sự góp ý của các đồng nghiệp qua địa chỉ email: [email protected].

II. Nội dung:


II.1. Giới thiệu các hàm Hypebolic.

$\bullet $ Hàm sin hypebolic định bởi: ${\mathop{\rm sh}\nolimits} x = \frac{{{e^x} - {e^{ - \,x}}}}{2}$ có tập xác định $\mathbb{R}$ và tập giá trị $\mathbb{R}$

$\bullet $ Hàm cos hyperbolic định bởi: ${\mathop{\rm ch}\nolimits} x = \frac{{{e^x} + {e^{ - \,x}}}}{2}$ có tập xác định $\mathbb{R}$ và tập giá trị $\left[ {1, + \infty } \right)$

$\bullet $ Hàm tan hyperbolic định bởi: ${{\mathop{\rm th}\nolimits} x} = \frac{{{\mathop{\rm sh}\nolimits} x}}{{{\mathop{\rm ch}\nolimits} x}} = \frac{{{e^x} - {e^{ - \,x}}}}{{{e^x} + {e^{ - \,x}}}}$ có tập xác định $\mathbb{R}$ và tập giá trị $\left( { - 1,1} \right)$

$\bullet $ Hàm cot hyperbolic định bởi: $\coth x = \frac{{{\mathop{\rm ch}\nolimits} x}}{{{\mathop{\rm sh}\nolimits} x}} = \frac{{{e^x} + {e^{ - \,x}}}}{{{e^x} - {e^{ - \,x}}}}$ có tập xác định $\mathbb{R}\backslash \left\{ 0 \right\}$ và tập giá trị $\left( { - \,\infty \,,\, - 1} \right) \cup \left( {1\,,\, + \,\infty } \right)$

Các tính chất sau bạn đọc có thể tự chứng minh:

1. $c{h^2}x - s{h^2}x = 1$

2. $ch\left( {x \pm y} \right) = chx.chy \pm shx.shy$

3. $sh\left( {x \pm y} \right) = chx.shy \pm chy.shx$

4. $sh\left( {2x} \right) = 2shx.chx$

5. $sh\left( {3x} \right) = 4s{h^3}x + 3shx$

6. $ch\left( {2x} \right) = c{h^2}x + s{h^2}x = 2c{h^2}x - 1 = 2s{h^2}x + 1$

7. $ch\left( {3x} \right) = 4c{h^3}x - 3chx$

8. $ch\left( {4x} \right) = 8c{h^4}x - 8c{h^2}x + 1 = 8s{h^4}x + 8s{h^2}x + 1$

9. $sh\left( {4x} \right) = 8s{h^3}x.chx + 4shx.chx = 8s{h^3}x\sqrt {1 + s{h^2}x} + 4shx\sqrt {1 + s{h^2}x} $

10. $th\left( {2x} \right) = \frac{{2thx}}{{1 + t{h^2}x}}$

11. $th\left( {3x} \right) = \frac{{t{h^3}x + 3thx}}{{1 + 3t{h^2}x}}$

II.2 Các dạng thường gặp.

1. Dạng 1: $\left\{ \begin{array}{l} \mathbf{\textbf{Cho}\,\,\,{u_1}}\\ \mathbf{{u_{n + 1}} = 2u_n^2 - 1;\,\,\forall n \in \mathbb{N^*}} \end{array} \right.$

Ta xét các trường hợp sau:

* Trường hợp 1:

Nếu ${u_1} = 1$ thì ta được ${u_n}=1,\,\,\,\forall n \in \mathbb{{N}^*}$

Nếu ${u_1} = -1$ thì ta được ${u_n}=1,\,\,\,\forall n \ge 2$

* Trường hợp 2:

Nếu $-1<{u_1}<1$ thì ta đặt ${u_1}= \cos\alpha;\,\,\,\,\alpha \in \left( {0,\pi } \right)$

Chứng minh quy nạp ta được: ${u_n}=\cos\left({{2^{n-1}}\alpha} \right).$

* Trường hợp 3: Nếu ${u_1}>1$ thì ta đặt ${u_1}=ch \alpha$. Chứng minh quy nạp ta được ${u_n} = ch\left( {{2^{n - 1}}\alpha } \right)$

Lưu ý: ${u_1} = ch\alpha \Leftrightarrow {u_1} = \frac{{{e^\alpha } + {e^{ - \,\alpha }}}}{2}$. Giải phương trình ta chỉ cần chọn một nghiệm $e^{\alpha}$.

Khi đó: ${u_n} = \frac{{{{\left( {{e^\alpha }} \right)}^{{2^{n - 1}}}} + {{\left( {{e^{ - \,\alpha }}} \right)}^{{2^{n - 1}}}}}}{2}$

* Trường hợp 4: Nếu ${u_1}<-1$ thì ta đặt ${u_1}=-ch \alpha$. Chứng minh quy nạp ta được ${u_n} = ch\left( {{2^{n - 1}}\alpha } \right),\,\,\,\,\forall n \ge 2$

Các dạng dãy số quy về dạng 1

1.1. Dạng: $\left\{ \begin{array}{l} \mathbf{\textbf{Cho}\,\,{u_1}}\\ \mathbf{{u_{n + 1}} = u_n^2 - 2\,,\,\,\forall n \in \mathbb{N^*}} \end{array} \right.$

Đặt ${u_n}=2 {x_n}$ ta được: $\left\{ \begin{array}{l}
{x_1} = \frac{1}{2}{u_1}\\ {x_{n + 1}} = 2x_n^2 - 1,\,\forall n \in \mathbb{N^*} \end{array} \right.$

1.2. Dạng: $\left\{ \begin{array}{l} \mathbf{\textbf{Cho}\,\,{u_1}}\\ \mathbf{{u_{n + 1}} = au_n^2 - \frac{2}{a}\,\,,\,\,\forall n \in \mathbb{N^*}} \end{array} \right.\,\,\,\,\,\,\,\mathbf{\left( {a \ne 0} \right)}$

Đặt ${u_n}= \dfrac{2}{a} {x_n}$ ta được: $\left\{ \begin{array}{l} {x_1} = \frac{a}{2}{u_1}\\ {x_{n + 1}} = 2x_n^2 - 1\,,\,\,\forall n \in \mathbb{N^*} \end{array} \right.$

1.3. Dạng: $\left\{ \begin{array}{l} \mathbf{ \text{Cho}\,\,{u_1}}\\ \mathbf{{u_{n + 1}} = au_n^2 + b{u_n} + c\,,\,\,\forall n \in \mathbb{N^*}} \end{array} \right.\,\,\,\,\,\,\,\,\,\,\,\,\,\mathbf{\left( {a \ne 0,\,c = \frac{{{b^2} - 2b - 8}}{{4a}}} \right)}$

Ta đặt: ${u_n}=p {x_n} +q$, thế vào giả thiết và tìm $p,q$ để ta được: ${x_{n + 1}} = 2x_n^2 - 1$

1.4. Dạng: $\left\{ \begin{array}{l} \mathbf{ \text{Cho}\,\,{u_1}}\\ \mathbf{{u_{n + 1}} = u_n^2 - 2{a^{{2^n}}}\,\,,\,\,\forall n \in \mathbb{N^*}} \end{array} \right. \,\,\,\,\,\, \mathbf{(a \ne 0)}$

Đặt ${u_n} = 2{a^{{2^{n - 1}}}}{x_n}$ ta được: $\left\{ \begin{array}{l} {x_1} = \frac{1}{{2a}}{u_1}\\
{x_{n + 1}} = 2x_n^2 - 1\,,\,\forall n \in \mathbb{N^*} \end{array} \right.$

1.5. Dạng: $\left\{ \begin{array}{l} \mathbf{ \text{Cho}\,\,{u_1}}\\ \mathbf{{u_{n + 1}} = 2{a^{{2^n}}}u_n^2 - {a^{\left( {n + 1} \right){2^n}}}\,\,,\,\,\forall n \in \mathbb{N^*}} \end{array} \right.\,\,\,\,\,\,\, \mathbf{\left( {a \ne 0} \right)}$

Đặt ${u_n} = {a^{n{2^{n - 1}}}}{x_n}$ ta được: $\left\{ \begin{array}{l} {x_1} = \frac{1}{a}{u_1}\\ {x_{n + 1}} = 2x_n^2 - 1 \end{array} \right.$

1.6. Dạng: $\left\{ \begin{array}{l} \mathbf{ \text{Cho}\,\,\,{u_1} \ge - 2}\\ \mathbf{{u_{n + 1}} = \sqrt {2 + {u_n}} \,;\,\,\forall n \in \mathbb{N^*}} \end{array} \right.$

Ta xét các trường hợp sau:

* Trường hợp 1: Nếu $ - 2 \le {u_1} \le 2$ thì ta đặt ${u_1}=2\cos \alpha;\,\,\, \alpha \in \left({0, \pi} \right)$ rồi áp dụng công thức $1 + \cos \alpha = 2{\cos ^2}\frac{\alpha }{2}$, chứng minh quy nạp ta được: ${u_n} = 2\cos \frac{\alpha }{{{2^{n - 1}}}}\,,\,\,\forall n \in \mathbb{N^*}$

* Trường hợp 2: Nếu ${u_1}>2$ thì ta đặt ${u_1} = 2ch\alpha $ rồi áp dụng công thức $1 + ch\left( {2\alpha } \right) = 2c{h^2}\alpha $, chứng minh quy nạp ta được: ${u_n} = 2ch\frac{\alpha }{{{2^{n - 1}}}}\,,\,\,\forall n \in \mathbb{N^*}$

1.7. Dạng: $\left\{ \begin{array}{l} \mathbf{ \text{Cho}\,\,{u_1}}\\ \mathbf{{u_{n + 1}} = 2{u_n}\sqrt {1 + u_n^2} \,,\,\,\forall n \in \mathbb{N^*}} \end{array} \right.$

Đặt $sh \alpha = {u_1}$, giải phương trình để tìm $e^\alpha$

Chứng minh quy nạp ta được: $${u_n} = sh\left( {{2^{n - 1}}\alpha } \right) = \dfrac{{{{\left( {{e^\alpha }} \right)}^{{2^{n - 1}}}} - {{\left( {{e^{ - \,\alpha }}} \right)}^{{2^{n - 1}}}}}}{2}$$
Dạng mở rộng: $\left\{ \begin{array}{l} \text{Cho}\,\,{u_1}\\ {u_{n + 1}} = b\left( {{u_n} + c} \right)\sqrt {u_n^2 + 2c{u_n} + {c^2} + {a^2}} - c\,,\,\,\forall n \in \mathbb{N^*} \end{array} \right.$ Với $a,\,b >0 $ và $ab=2$

1.8. Dạng: $\left\{ \begin{array}{l} \mathbf{ \text{Cho}\,\,{u_1} > 1}\\ \mathbf{{u_{n + 1}} = a.{u_n} + b\sqrt {u_n^2 - 1} \,,\,\,\forall n \in \mathbb{N^*}} \end{array} \right.\,\,\,\,\textbf{với}\,\,\, \mathbf{a>0}\,\,\,\textbf{ và}\,\,\,\,\mathbf{ a^2 - b^2 =1}$

Đặt $\left\{ \begin{array}{l} ch\beta = a\\ sh\beta = b \end{array} \right.$, giải hệ để tìm ${e^\beta }$ và đặt $\left\{ \begin{array}{l} ch\alpha = {u_1}\\ sh\alpha = \sqrt {u_1^2 - 1} \end{array} \right.$, giải hệ để tìm ${e^\alpha }$

Ta được: ${u_2} = ch\alpha .ch\beta + sh\alpha .sh\beta = ch\left( {\alpha + \beta } \right)$

Chứng minh quy nạp ta được: $${u_n} = ch\left[ {\alpha + \left( {n - 1} \right)\beta } \right]\, = \frac{{\left( {{e^\alpha }} \right){{\left( {{e^\beta }} \right)}^{n - 1}} + \left( {\frac{1}{{{e^\alpha }}}} \right){{\left( {\frac{1}{{{e^\beta }}}} \right)}^{n - 1}}}}{2},\,\,\forall n \in \mathbb{N^*}$$
Dạng mở rộng: $\left\{ \begin{array}{l}\text{Cho}\,\,{u_1}\\ {u_{n + 1}} = \frac{{{u_n}}}{{a + \sqrt {cu_n^2 + b} }}\,\,\,,\,\,\,\forall n \in \mathbb{N^*} \end{array} \right.$ với $\alpha > 0,\,\,a > 0$ và $a^2 - b =1$

Biến đổi: $\frac{1}{{{u_{n + 1}}}} = \frac{{a + \sqrt {cu_n^2 + b} }}{{{u_n}}} = a\left( {\frac{1}{{{u_n}}}} \right) + \sqrt {c + b\left( {\frac{1}{{u_n^2}}} \right)} $. Đặt ${x_n} = \frac{1}{{{u_n}}}$ ta đưa về dạng trên

1.9. Dạng: $\left\{ \begin{array}{l} \mathbf{ \text{Cho}\,\,{u_1}}\\ \mathbf{{u_{n + 1}} = \frac{{2{u_n}}}{{1 + u_n^2}}\,,\,\,\forall n \in \mathbb{N^*}} \end{array} \right.$

Dạng này ta chia trường hợp như sau:

* Nếu ${u_1}=1$ hay ${u_1}=-1$ thì ${u_n}=1$ hay ${u_n}=-1$

* Nếu $-1 < {u_1} < 1$ thì ta đặt ${u_1}=th \alpha$

Chứng minh quy nạp ta được: $${u_n} = th\left( {{2^{n - 1}}\alpha } \right) = \frac{{{{\left( {{e^\alpha }} \right)}^{{2^{n - 1}}}} - {{\left( {{e^{ - \,\alpha }}} \right)}^{{2^{n - 1}}}}}}{{{{\left( {{e^\alpha }} \right)}^{{2^{n - 1}}}} + {{\left( {{e^{ - \,\alpha }}} \right)}^{{2^{n - 1}}}}}} = \frac{{{{\left( {{e^{2\alpha }}} \right)}^{{2^{n - 1}}}} - 1}}{{\left( {{e^{2\alpha }}} \right) + 1}}\,,\,\,\forall n \in \mathbb{N^*}$$
* Nếu ${u_1} < -1$ hay ${u_1} > 1$ thì ta đặt ${u_1} = \coth \alpha = \frac{1}{{th\alpha }}$

Chứng minh quy nạp ta được: $${u_n} = th\left( {{2^{n - 1}}\alpha } \right) = \frac{{{{\left( {{e^{2\alpha }}} \right)}^{{2^{n - 1}}}} - 1}}{{\left( {{e^{2\alpha }}} \right) + 1}}\,,\,\,\forall n \ge 2$$
1.10. Dạng: $\left\{ \begin{array}{l} \mathbf{ \text{Cho}\,\,{u_1}}\\ \mathbf{{u_{n + 1}} = 4u_n^3 - 3{u_n}\,,\,\,\forall n \in \mathbb{N^*}} \end{array} \right.\,\,\,\,\,\,\,\,\mathbf{\left( a \right)}$ và $\left\{ \begin{array}{l}
\mathbf{\text{Cho}\,\,{u_1}}\\ \mathbf{{u_{n + 1}} = 4u_n^3 + 3{u_n}\,,\,\,\forall n \in \mathbb{N^*}} \end{array} \right.\,\,\,\,\,\,\,\mathbf{\left( b \right)}$

Với dạng 1.10a) Ta xét các trường hợp sau:

* Trường hợp 1: Nếu ${u_1}=1$ hay ${u_1}=-1$ thì ta được ${u_n}=1$ hay ${u_n}=-1,\,\,\,\,\,\,\forall \in \mathbb{N^*}$

* Trường hợp 2: Nếu $-1 < {u_1} <1$ thi ta đặt ${u_1}=\cos \alpha;\,\,\, \alpha \in \left({0; \pi} \right)$

Chứng minh quy nạp ta được: ${u_n} = \cos \left( {{3^{n - 1}}\alpha } \right)$

* Trường hợp 3: Nếu ${u_1}>1$ thì ta đặt ${u_1}=ch \alpha$.

Chứng minh quy nạp ta được: $${u_n} = ch\left( {{3^{n - 1}}\alpha } \right) = \frac{{{{\left( {{e^\alpha }} \right)}^{{3^{n - 1}}}} + {{\left( {{e^\alpha }} \right)}^{{3^{n - 1}}}}}}{2}$$
* Trường hợp 4: Nếu ${u_1}<-1$ thì ta đặt $-{u_1}=ch \alpha$

Chứng minh quy nạp ta được: $${u_n} = - \,ch\left( {{3^{n - 1}}\alpha } \right) = - \,\frac{{{{\left( {{e^\alpha }} \right)}^{{3^{n - 1}}}} + {{\left( {{e^\alpha }} \right)}^{{3^{n - 1}}}}}}{2}$$
Với dạng 1.10b) ta đặt ${u_1}=sh \alpha$. Chứng minh quy nạp ta được: $${u_n} = sh\left( {{3^{n - 1}}\alpha } \right) = \,\frac{{{{\left( {{e^\alpha }} \right)}^{{3^{n - 1}}}} - {{\left( {{e^\alpha }} \right)}^{{3^{n - 1}}}}}}{2}$$

Các dạng dãy số quy về dạng 1.10.

1.10.1. Dạng: $\left\{ \begin{array}{l} \text{Cho}\,\,{u_1}\\ {u_{n + 1}} = au_n^3\, \pm 3{u_n}\,\,;\,\,\forall n \in \mathbb {N^*} \end{array} \right.\,\,\,\,\,\,\left( {a > 0} \right)$

Nhận xét: Ta tìm cách quy về dạng trên như sau: Đặt ${u_n}=b{v_n}$.

Thế vào giả thiết ta được: $b{v_{n + 1}} = a{b^3}v_n^3 \pm 3b{v_n} \Rightarrow {v_{n + 1}} = a{b^2}v_n^3 \pm 3{v_n}$

Ta cần tìm $b$ sao cho $a{b^2} = 4 \Leftrightarrow b = \frac{2}{{\sqrt a }}$

1.10.2. Dạng: $\left\{ \begin{array}{l} {u_1} = \alpha \\ {u_{n + 1}} = au_n^3 + bu_n^2 + c{u_n} + d\,,\,\,\forall n \in \mathbb{N^*} \end{array} \right.$

với $a>0,\,c = \frac{{{b^2}}}{{3a}}\,,\,\,d = \frac{{b\left( {c - 3} \right)}}{{9a}}\,,\,\,\alpha > - \,\frac{b}{{3a}}$ hay $a>0,\,c = \frac{{{b^2} + 9a}}{{3a}}\,,\,\,d = \frac{{{b^3} + 18ab}}{{27{a^2}}}\,,\,\,\alpha > \frac{2}{{\sqrt a }} - \frac{b}{{3a}}$

Đặt ${u_n}=a{v_n}+b$, thế vào giả thiết rồi tìm hai số $a, b$ sao cho ${v_{n + 1}} = 4v_n^3 \pm 3{v_n}$

1.11. Dạng: $\left\{ \begin{array}{l}
\mathbf{ \text{Cho}\,\,{u_1}}\\
\mathbf{{u_{n + 1}} = 8u_n^4 - 8u_n^2 + 1\,,\,\,\forall n \in \mathbb{N^*}}
\end{array} \right.\,\,\,\,\,\,\mathbf{\left( a \right)}$ và $\left\{ \begin{array}{l}
\mathbf{ \text{Cho}\,\,{u_1}}\\
\mathbf{{u_{n + 1}} = \left( {8u_n^3 + 4{u_n}} \right)\sqrt {1 + u_n^2} \,,\,\,\forall n \in \mathbf{N^*}}
\end{array} \right.\,\,\,\,\,\,\mathbf{\left( b \right)}$

Với dạng 1.11a) Ta xét các trường hợp sau:

* Trường hợp 1:

Nếu ${u_1} = 1$ thì ta được ${u_n} = 1,\,\,\forall \in \mathbb{N^*}$

Nếu ${u_1} = – 1$ thì ta được ${u_n} = 1,\,\,\forall n \le 2$

* Trường hợp 2: Nếu $– 1 < {u_1} < 1$ thì ta đặt ${u_1} = \cos \alpha;\,\,\,\alpha \in \left({0; \pi} \right)$.

Chứng minh quy nạp ta được: ${u_n} = \cos \left( {{4^{n - 1}}\alpha } \right)$

* Trường hợp 3 : Nếu ${u_1} > 1$ thì ta đặt ${u_1} = ch \alpha$.

Chứng minh quy nạp ta được: $${u_n} = ch\left( {{4^{n - 1}}\alpha } \right) = \frac{{{{\left( {{e^\alpha }} \right)}^{{4^{n - 1}}}} + {{\left( {{e^\alpha }} \right)}^{{4^{n - 1}}}}}}{2}$$
* Trường hợp 4: Nếu ${u_1}< – 1$ thì ta đặt $– {u_1} = ch \alpha$.

Chứng minh quy nạp ta được: $${u_n} = \,ch\left( {{4^{n - 1}}\alpha } \right) = \frac{{{{\left( {{e^\alpha }} \right)}^{{4^{n - 1}}}} + {{\left( {{e^\alpha }} \right)}^{{4^{n - 1}}}}}}{2}\,,\,\,\forall n \ge 2$$
Với dạng 1.11b) ta đặt ${u_1} = sh \alpha$. Chứng minh quy nạp ta được: $${u_n} = sh\left( {{4^{n - 1}}\alpha } \right) = \,\frac{{{{\left( {{e^\alpha }} \right)}^{{4^{n - 1}}}} - {{\left( {{e^\alpha }} \right)}^{{4^{n - 1}}}}}}{2}$$

1.12. Dạng: $\left\{ \begin{array}{l}
\mathbf{ \text{Cho}\,\,{u_1}}\\
\mathbf{{u_{n + 1}} = \frac{{u_n^3 + 3{u_n}}}{{1 + 3u_n^2}}\,,\,\,\forall n \in \mathbb{N^*}}
\end{array} \right.$

Ta xét các trường hợp sau:

* Nếu ${u_1} = 1$ hay ${u_1} = – 1$ thì ${u_n} = 1$ hay ${u_n} = – 1$

* Nếu $– 1 < {u_1} < 1$ thì ta đặt $th \alpha ={u_1}$.

Chứng minh quy nạp ta được: $${u_n} = th\left( {{3^{n - 1}}\alpha } \right) = \frac{{{{\left( {{e^{2\alpha }}} \right)}^{{3^{n - 1}}}} - 1}}{{{{\left( {{e^{2\alpha }}} \right)}^{{3^{n - 1}}}} + 1}}$$
* Nếu ${u_1} < – 1$ hay ${u_1} > 1$ thì ta đặt $coth \alpha ={u_1}$ .

Chứng minh quy nạp ta được: $${u_n} = \coth \left( {{3^{n - 1}}\alpha } \right) = \frac{{{{\left( {{e^{2\alpha }}} \right)}^{{3^{n - 1}}}} + 1}}{{{{\left( {{e^{2\alpha }}} \right)}^{{3^{n - 1}}}} - 1}}$$

Bây giờ, chúng ta hãy xét một số ví dụ sau.

Ví dụ 1: Tìm số hạng tổng quát của dãy số $({u_n})$ định bởi: $\left\{ \begin{array}{l}
{u_1} = \frac{1}{3}\\
{u_{n + 1}} = 3u_n^2 - \frac{2}{3}\,,\,\,\forall n \in \mathbb{N^*}
\end{array} \right.$

Giải: Đặt ${u_n} = \frac{2}{3}{x_n}$ ta được dãy số $({x_n})$ định bởi: $\left\{ \begin{array}{l}
{x_1} = \frac{1}{2}\\
{x_{n + 1}} = 2x_n^2 - 1,\,\,\forall n \in \mathbb{N^*}
\end{array} \right.$

Ta có: ${x_1} = \cos \frac{\pi }{3}$. Giả sử ${x_n} = \cos \left( {{2^{n - 1}}\frac{\pi }{3}} \right)$. Khi đó ${x_{n + 1}} = 2{\cos ^2}\left( {{2^{n - 1}}\frac{\pi }{3}} \right) - 1 = \cos \left( {{2^n}\frac{\pi }{3}} \right)$

Vậy: ${x_n} = \cos \left( {{2^{n - 1}}\frac{\pi }{3}} \right)$ suy ra ${u_n} = \frac{2}{3}\cos \left( {{2^{n - 1}}\frac{\pi }{3}} \right)$

Ví dụ 2: Tìm số hạng tổng quát của dãy số $({u_n})$ định bởi: $\left\{ \begin{array}{l}
{u_1} = \frac{{15}}{2}\\
{u_{n + 1}} = u_n^2 - {2.3^{{2^n}}},\,\,\forall n \in \mathbb{N^*}
\end{array} \right.$

Giải: Đặt ${u_n} = {2.3^{{2^{n - 1}}}}{x_n}$ ta được dãy số $({x_n})$ định bởi: $\left\{ \begin{array}{l}
{x_1} = \frac{5}{4}\\
{x_{n + 1}} = 2x_n^2 - 1,\,\,\forall n \in \mathbb{N^*}
\end{array} \right.$

Đặt $ch\alpha = \frac{5}{2} \Leftrightarrow \frac{{{e^\alpha } + {e^{ - \,\alpha }}}}{2} = \frac{5}{4}$, giải phương trình ta chọn nghiệm ${e^\alpha } = 2$

Ta có: ${x_1} = ch\alpha $. Giả sử ${x_n} = ch\left( {{2^{n - 1}}\alpha } \right)$. Khi đó: ${x_{n + 1}} = 2c{h^2}\left( {{2^{n - 1}}\alpha } \right) - 1 = ch\left( {{2^n}\alpha } \right)$

Vậy: ${x_n} = ch\left( {{2^{n - 1}}\alpha } \right) = \frac{{{{\left( {{e^\alpha }} \right)}^{{2^{n - 1}}}} + {{\left( {{e^{ - \,\alpha }}} \right)}^{{2^{n - 1}}}}}}{2} = \frac{{{2^{{2^{n - 1}}}} + {{\left( {\frac{1}{2}} \right)}^{{2^{n - 1}}}}}}{2}$ suy ra ${u_n} = {6^{{2^{n - 1}}}} + {\left( {\frac{3}{2}} \right)^{{2^{n - 1}}}}$

Ví dụ 3: Tìm số hạng tổng quát của dãy số $({u_n})$ định bởi: $\left\{ \begin{array}{l}
{u_1} = 3\\
{u_{n + 1}} = \sqrt {2 + {u_n}} \,,\,\,\forall n \in \mathbb{N^*}
\end{array} \right.$

Giải: Đặt ${u_1} = 2ch\alpha \Leftrightarrow 3 = {e^\alpha } + {e^{ - \,\alpha }}$. Giải phương trình ta chọn nghiệm ${e^\alpha } = \frac{{3 + \sqrt 5 }}{2}$

Giả sử ${u_n} = 2ch\frac{\alpha }{{{2^{n - 1}}}}$. Khi đó: $${u_{n + 1}} = \sqrt {2 + 2ch\frac{\alpha }{{{2^{n - 1}}}}} = \sqrt {2\left( {1 + ch\frac{\alpha }{{{2^{n - 1}}}}} \right)} = \sqrt {4c{h^2}\frac{\alpha }{{{2^n}}}} = 2c{h^2}\frac{\alpha }{{{2^n}}}$$
Vậy: $${u_n} = 2ch\frac{\alpha }{{{2^{n - 1}}}} = {\left( {{e^\alpha }} \right)^{\frac{1}{{{2^{n - 1}}}}}} + {\left( {\frac{1}{{{e^\alpha }}}} \right)^{\frac{1}{{{2^{n - 1}}}}}} = {\left( {\frac{{3 + \sqrt 5 }}{2}} \right)^{\frac{1}{{{2^{n - 1}}}}}} + {\left( {\frac{{3 - \sqrt 5 }}{2}} \right)^{\frac{1}{{{2^{n - 1}}}}}}\,,\,\,\forall n \in \mathbb{N^*}$$

Còn tiếp...




#319239 Về một bài toán bất đẳng thức

Đã gửi bởi Ban Biên Tập on 25-05-2012 - 00:02 trong Chuyên đề toán THPT

VỀ MỘT BÀI TOÁN BẤT ĐẲNG THỨC

NGUYỄN VĂN HUYỆN

Sinh viên CN10B trường Đại học Giao thông vận tải tp.Hồ Chí Minh

[email protected]


1. Lời nói đầu:
Bất đẳng thức không thuần nhất là một phần quan trọng, hay và tương đối khó trong bất đẳng thức vì chúng ta không có một phương pháp thực sự “tốt” nào để giải quyết loạt các bài toán này. Những lời giải cho những bất đẳng thức dạng này thường mang những ý tưởng khá hay, độc đáo và thường là những phương pháp không mẫu mực. Ở bài viết này tác giả xin được giới thiệu đến bạn đọc một bài toán bất đẳng thức không thuần nhất tương đối đơn giản nhưng lại có nhiều ứng dụng trong việc giải quyết các bất đẳng thức thuần nhất và không thuần nhất khác, thậm chí là những bất đẳng thức trong đề thi học sinh giỏi quốc gia và quốc tế.

(Bài viết được trích ra từ bài viết cùng tên của tác giả đăng trong chuyên đề Toán học số 9 của trường Phổ Thông Năng Khiếu Đại học Quốc gia Tp.HCM và được tác giả bổ sung, điều chỉnh.)

2. Nội dung:
Bài toán gốc: Cho $a, b, c$ là các số thực dương thay đổi bất kì. Chứng minh rằng:
$$a^2+b^2+c^2+2abc+1\geq 2(ab+bc+ca) \ \ \ (1)$$
Lời giải 1:
Ta sẽ sử dụng phương pháp tam thức bậc hai để chứng minh bài toán. Bất đẳng thức được chuyển về dạng tam thức bậc hai như sau:
$$f(a)=a^2+2a(bc-b-c)+(b-c)^2+1\geq 0$$
* Nếu $bc\geq b+c$ thì ta có ngay điều phải chứng minh.
* Xét trường hợp ngược lại $bc\leq b+c$, và điều này tương đương với $(b-1)(c-1)\leq 1$. Khi đó ta tính được biệt thức $\Delta '$ của $f(a)$ là:
$$\Delta '=(bc-b-c)^2-(b-c)^2-1=bc(b-2)(c-2)-1$$
Ta xét hai trường hợp:
Trường hợp 1: Có đúng một trong hai số $b, c$ lớn hơn $2$, số còn lại không lớn hơn $2$. Trong trường hợp này ta có $(b-2)(c-2)\leq 0$ từ đó suy ra $\Delta '\leq 0$.
Trường hợp 2: Cả hai số $b, c$ đều không lớn hơn 2. Khi đó theo bất đẳng thức AM-GM, ta có :
$$\Delta '=bc(2-b)(2-c)-1\leq \left[\frac{b+c+(2-b)+(2-c)}{4}\right]^4-1= 0.$$

Tóm lại trong mọi trường hợp ta đều có $\Delta '\leq 0$. Tức $f(a)\geq 0$ và đây là điều phải chứng minh. Đẳng thức xảy ra khi và chỉ khi $a=b=c=1.$

Lời giải 2: Theo nguyên lý Dirichlet, ta thấy rằng trong ba số $a, b, c$ sẽ có hai số hoặc cùng $\geq 1$ hoặc cùng $\leq 1$. Giả sử hai số đó là $a, b$ khi đó:
$$(a-1)(b-1)\geq 0.$$
Từ đây, bằng cách sử dụng hằng đẳng thức:
$$a^2+b^2+c^2+2abc+1-2(ab+bc+ca)=(a-b)^2+(c-1)^2+2c(a-1)(b-1)\geq 0$$
Ta thu được ngay bất đẳng thức (1), phép chứng minh hoàn tất.

Lời giải 3: Ta sẽ sử dụng phương pháp dồn biến để chứng minh bài toán. Giả sử $c$ là số bé nhất và đặt:
$$f(a,b,c)=a^2+b^2+c^2+2abc+1-2(ab+bc+ca)$$
Ta có:
$$f(a,b,c)-f(\sqrt{ab},\sqrt{ab},c)=(\sqrt{a}-\sqrt{b})^2(a+b+2\sqrt{ab}-2c)\geq 0$$
Do đó $f(a,b,c)\geq f(\sqrt{ab},\sqrt{ab},c)$, vậy ta chỉ cần chứng minh $f(\sqrt{ab},\sqrt{ab},c)\geq 0$.
Thật vậy, nếu đặt $t=\sqrt{ab}$ thì ta có:
$$f(t,t,c)=2t^2+c^2+2t^2c-2(t^2+2tc)+1=(c-1)^2+2c(t-1)^2\geq 0$$
Bài toán được chứng minh xong.

Lời giải 4: Sử dụng lần lượt bất đẳng thức AM-GM, ta có:
$$2abc+1=abc+abc+1\geq 3\sqrt[3]{a^2b^2c^2}\geq \frac{9abc}{a+b+c}$$
Do đó, ta chỉ cần chứng minh:
$$a^2+b^2+c^2+\frac{9abc}{a+b+c}\geq 2(ab+bc+ca)$$
Thực hiện phép khi triển trực tiếp, ta có bất đẳng thức tương đương với:
$$a^3+b^3+c^3+3abc\geq ab(a+b)+bc(b+c)+ca(c+a)$$
Đúng vì đây chính là bất đẳng thức Schur dạng bậc ba nên ta có điều phải chứng minh.

Bất đẳng thức $(1)$ được Darij Grinberg đề xuất trên diễn đàn toán học Mathlinks.ro vào năm 2004. Mặc dù chỉ là một kết quả đơn giản nhưng bất đẳng thức này lại có nhiều ứng dụng vào việc chứng minh các bất đẳng thức ba biến. Sau đây, chúng ta sẽ đi vào xét các bài toán cụ thể để hiểu rõ vì sao chúng tôi lại nói như vậy.

Bài toán 1.1. (Moscow Mathematical Olympiad 2000). Cho $a, b, c$ là các số dương thỏa mãn $abc = 1$. Chứng minh bất đẳng thức sau
$$a^2 + b^2+c^2+a+b+c \geq 2(ab + bc + ca) \ \ \ (1.1.1)$$

Lời Giải.
Sử dụng bất đẳng thức AM-GM, ta có
$$a + b + c \geq 3\sqrt[3]{abc} = 3 = 2abc + 1$$.
Vì thế để chứng minh bài toán, ta chỉ cần chứng minh
$$a^2+b^2+c^2+2abc+1 \geq 2(ab+bc+ca)$$
Đây chính là bất đẳng thức (1) nên ta có ngay điều phải chứng minh.
Đẳng thức xảy ra khi và chỉ khi $a = b = c = 1.$

Bài toán 1.2. (Viet Nam Mathematical Olympiad 2006). Tìm hằng số $k$ lớn nhất để bất đẳng thức
$$\frac{1}{a^2}+\frac{1}{b^2}+\frac{1}{c^2}+3k \geq (k+1)(a+b+c)$$
luôn đúng với mọi $a, b, c$ dương thỏa mãn $abc = 1.$

Lời Giải.
Cho $a = b = t (t > 0, t = 1)$ và $c =\frac{1}{t^2}$, khi đó $a, b, c$ là các số dương và $abc = 1$. Do đó, theo yêu cầu của bài toán ta phải có
$$\frac{2}{t^2}+t^4+3k \geq (k+1)(2t+\frac{1}{t^2})$$
Bất đẳng thức này tương đương với mỗi bất đẳng thức trong dãy các bất đẳng thức sau đây
$$\frac{2}{t^2}+t^4-3 \geq (k+1)(2t+\frac{1}{t^2}-3),$$
$$\frac{t^6-3t^2+2}{t^2} \geq \frac{(k+1)(2t^3-3t^2+1)}{t^2}$$
$$\frac{t^2-1)^2(t^2+2)}{t^2} \geq \frac{(k+1)(t-1)^2(2t+1)}{t^2}$$
$$\frac{(t+1)^2(t^2+2)}{2t+1} \geq k+1, \forall t>0$$
Cho $t \to 0^+$, ta được $2 \geq k+1$, suy ra $k \leq 1$. Ta sẽ chứng minh rằng 1 chính là hằng số cần tìm, tức là
$$\frac{1}{a^2}+\frac{1}{b^2}+\frac{1}{c^2}+3 \geq 2(a+b+c).$$
Đặt $x = \frac{1}{a}, y= \frac{1}{b}, z=\frac{1}{c}$ thì ta có $xyz=1, 3=2xyz+1$ và
$$x^2+y^2+z^2+2xyz+1 \geq 2(xy+yz+zx)$$
hiển nhiên đúng theo (1). Vậy ta có kết luận $k_{max}=1.$

Bài toán 1.3. (Mircea Lascu, Romania Junior Team Selecsion Test 2005).
Cho các số dương $ a, b, c$ thỏa mãn điều kiện $(a + b)(b + c)(c + a) = 1.$ Chứng minh rằng
$$ab+bc+ca \leq \frac{3}{4}.$$
Lời Giải.
Đặt $x = a + b, y = b + c, z = c + a$ thì ta có $xyz = 1$ và
$$a=\frac{z + x -y }{2}, b=\frac{x+y-z}{2}, c=\frac{y+z-x}{2}.$$
Bất đẳng thức cần chứng minh được viết lại như sau
$$\frac{z + x -y }{2}.\frac{x+y-z }{2}+\frac{x+y-z }{2}.\frac{y+z-x }{2}+\frac{y+z-x }{2}.\frac{z + x -y }{2} \leq \frac{3}{4}$$
Sau khi thu gọn, ta được
$$x^2+y^2+z^2+3 \geq 2(xy+yz+zx),$$
hay là
$$x^2+y^2+z^2+2xyz+1 \geq 2(xy+yz+zx).$$
Đây chính là bất đẳng thức (1) nên ta có điều phải chứng minh.
Đẳng thức xảy ra khi và chỉ khi $a = b = c = \frac{1}{2}$

Bài toán 1.4. (Gabriel Dospinescu, Marian Tetiva, Mircea Lascu).
Chứng minh rằng với mọi số thực dương a, b, c, ta đều có
$$a^2+b^2+c^2 + 2abc + 3 \geq (a + 1)(b + 1)(c + 1).$$

Lời Giải.
Sau khi khai triển và rút gọn, ta có bất đẳng thức tương đương
$$a^2+b^2+c^2+ abc + 2 \geq ab + bc + ca + a + b + c,$$
hay là
$$2(a^2+b^2+c^2) + 2abc + 4 \geq 2(ab + bc + ca + a + b + c).$$
Theo bất đẳng thức (1), ta có
$$a^2+b^2+c^2 + 2abc + 1 \geq 2(ab + bc + ca).$$
Sử dụng đánh giá này, ta đưa được bài toán về chứng minh
$$a^2+b^2+c^2+2abc+1 \geq 2(ab+bc+ca).$$
Bất đẳng thức này tương đương với bất đẳng thức hiển nhiên đúng là
$$(a-1)^2+(b-1)^2+(c-1)^2 \geq 0$$
Bài toán được chứng minh xong. Đẳng thức xảy ra khi và chỉ khi $a = b = c = 1.$

Bài toán 1.5. (Asian Pacific Mathematical Olympiad 2004).
Chứng minh bất đẳng thức sau luôn đúng với mọi số thực dương a, b, c bất kỳ
$$(a^2 + 2)(b^2+ 2)(c^2+ 2) \geq 9(ab + bc + ca), \ \ (1.5.1)$$

Lời Giải. Bất đẳng thức cần chứng minh tương đương với
$$a^2b^2c^2+3(a^2+b^2+c^2)+2(a^2b^2+b^2c^2+c^2a^2+3)+2 \geq 9(ab+bc+ca).$$
Theo bất đẳng thức AM-GM, ta có
$$3(a^2+b^2+c^2)=3(\frac{a^2+b^2}{2}+\frac{b^2+c^2}{2}+\frac{c^2+a^2}{2}) \geq 3(ab+bc+ca)$$
Và $$2(a^2b^2+b^2c^2+c^2a^2+3)=2 \begin{bmatrix} (a^2b^2+1)+(b^2c^2+1)+(c^2a^2+1) \end{bmatrix} \geq 4(ab+bc+ca).$$
Từ đó, bài toán được đưa về chứng minh
$$a^2+b^2+c^2+a^2b^2c^2+2 \geq 2(ab+bc+ca)$$
Bất đẳng thức này được viết lại thành
$$ \begin{bmatrix} a^2+b^2+c^2+2abc+1-2(ab+bc+c+a) \end{bmatrix}+(abc-1)^2 \geq 0$$
hiển nhiên đugns theo (1). Đẳng thức xảy ra khi và chỉ khi $a=b=c=1.$

NHẬN XÉT: Bài toán còn đúng trong trường hợp $a,b,c$ là các số thực bát kì. Thật vậy, từ chứng mnih trên ta thấy rằng
$$(a^2+2)(b^2+2)(c^2+2)=( \begin{vmatrix}a \end{vmatrix}^2+2)( \begin{vmatrix}b \end{vmatrix}^2+2)( \begin{vmatrix}c \end{vmatrix}^2+2) \geq 9(\begin{vmatrix}a \end{vmatrix} \begin{vmatrix}b \end{vmatrix}+ \begin{vmatrix}b \end{vmatrix} \begin{vmatrix}c \end{vmatrix}+ \begin{vmatrix}c \end{vmatrix} \begin{vmatrix}a \end{vmatrix}) \geq 9(ab+bc+ca).$$

Bài toán 1.6. (Crux Mathematicorum).
Chứng minh bất đẳng thức sau luôn đúng với mọi số thực dương $a, b, c$ bất kỳ
$$(a^2+2)(b^2+2)(c^2+2) \geq 3(a+b+c)^2$$

Lời Giải. Tương tự như trên, ta cũng sử dụng phép khai triển trực tiếp và viết lại bất đẳng thức dưới dạng
$$a^2b^2c^2+a^2+b^2+c^2+2(a^2b^2+b^2c^2+c^2a^2+3)+2 \geq 6(ab+bc+ca).$$
Đến đây, ta cũng sử dụng bất đẳng thức AM-GM để thu được
$$2(a^2b^2+b^2c^2+c^2a^2+3) \geq 4(ab+bc+ca),$$
và từ đó đưa được bất đẳng thức về chứng minh
$$a^2+b^2+c^2+a^2b^2c^2+2 \geq 2(ab+bc+ca).$$
Đây chính là bất đẳng thức (1.5.1) đã được chứng minh ở phần trên. Bài toán được chứng minh xong. Đẳng thức xảy ra khi $a = b = c = 1.$

Nhận Xét. Vì $(a+b+c)^2 \geq 3(ab+bc+ca),$ nên từ bài toán này ta có thể dễ dàng suy ra
$$(a^2+2)(b^2+2)(c^2+2) \geq 9(ab+bc+ca)$$
Vậy bài toán này chính là một kết quả mạnh hơn của bất đẳng thức Asian Pacific Mathematical Olypiad 2004. Ngoài ra ta còn có thể làm chặt bất đẳng thức này hơn nữa, ta cùng xét bài toán sau đây.

Bài toán 1.7. (Nguyễn Đình Thi) Cho $a, b, c$ là các số thực dương. Chứng minh rằng
$$(a^2+2)(b^2+2)(c^2+2) \geq 3(a+b+c)^2+(abc-1)^2.$$
.
Lời Giải. Sau khi khai triển và rút gọn, ta được bất đẳng thức tương đương
$$2(a^2b^2+b^2c^2+c^2a^2+3)+a^2+b^2+c^2+2abc+1 \geq 6(ab+bc+ca).$$
Theo bất đẳng thức AM-GM thì
$$2(a^2b^2+b^2c^2+c^2a^2+3) \geq 4(ab+bc+ca).$$
Do đó ta chỉ cần chứng minh
$$a^2+b^2+c^2+2abc+1 \geq 2(ab+bc+ca).$$
Đây chính là bất đẳng thức (1).
Bài toán được chứng minh xong. Đẳng thức xảy ra khi và chỉ khi $a = b = c = 1.$

Bài toán 1.8. (Nguyễn Văn Huyện).
Với mọi số thực dương $a, b, c$ và $k \geq 2$ là một số thực bất kỳ, khi đó ta luôn có bất đẳng thức
$$(a^2+k)(b^2+k)(c^2+k) \geq \frac{(k+1)^2}{3}(a+b+c)^2+k^3-3k-2, \ \ (1.8.1)$$

Lời Giải. Ta cũng thực hiện phép khai triển và viết bất đẳng thức trên thành
$$a^2b^2c^2+k(a^2b^2+b^2c^2+c^2a^2)+k^2(a^2+b^2+c^2)+3k+2 \geq \frac{(k+1)^2}{3}(a+b+c)^2.$$
Sử dụng bất đẳng thức (1.5.1), ta được
$$a^2+b^2+c^2+(a^2+b^2+c^2+a^2b^2c^+2) \geq a^2+b^2+c^2+2(ab+bc+ca)=(a+b+c)^2,$$
từ đó chỉ cần chứng minh được
$$k(a^2b^2+b^2c^2+c^2a^2)+(k^2-2)(a^2+b^2+c^2)+3k \geq \frac{k^2+2k+2}{3}(a+b+c)^2.$$
Theo bất đẳng thức AM-GM, thì
$$(a^2b^2+1)+(b^2c^2+1)+(c^2a^2+1) \geq 2(ab+bc+ca)$$
Suy ra
$$k(a^2b^2+b^2c^2+c^2a^2)+(k^2-2)(a^2+b^2+c^2)+3k \geq 2k(ab+bc+ca)+(k^2-2)(a^2+b^2+c^2)$$
$$=(k^2-k-2)(a^2+b^2+c^2)+k(a+b+c)^2$$
$$ \geq \frac{k^2-k-2}{3}(a+b+c)^2+k(a+b+c)^2$$
$$=\frac{k^2+2k-2}{3}(a+b+c)^2.$$
Bài toán được chứng minh xong. Đẳng thức xảy ra khi và chỉ khi $a = b = c = 1.$

Nhận Xét. Trong (1.8.1) nếu cho $k = 2$, thì ta thu được (1.6.1).

Bài toán 1.9. (Iran Mathematical Olympiad 2002).
Cho các số dương $a, b, c$ thỏa mãn $ a^2+b^2+c^2 + abc = 4.$ Chứng minh rằng
$$a+b+c \leq 3.$$
Lời Giải. Từ giả thiết sử dụng bất đẳng thức $(1)$, ta có
$$9 = 2(a^2+b^2+c^2 + abc) + 1 = a^2+b^2+c^2 + (a^2+b^2+c^2 + 2abc + 1)$$
$$\geq a^2+b^2+c^2+ 2(ab + bc + ca)$$
$$= (a + b + c)2,$$
Vì $a, b, c$ là các số dương nên sau khi lấy căn hai vế, ta được
$$a + b + c \leq 3.$$
Chứng minh hoàn tất. Đẳng thức xảy ra khi và chỉ khi $a = b = c = 1$.

Bài toán 1.10. (Mathematical Reflections 4/2006).
Cho $a, b, c$ là ba số thực dương thỏa mãn điều kiện $a + b + c = 3$. Chứng minh bất đẳng thức
$$a^2+b^2+c^2 + abc \geq 4 \ \ \ (1.10.1)$$

Lời Giải. Nhân 2 vào hai vế của bất đẳng thức, ta được
$$2(a^2+b^2+c^2) + 2abc \geq 8.$$
Bây giờ bằng cách sử dụng bất đẳng thức (1), ta có
$2(a^2+b^2+c^2) + 2abc = $[$a^2+b^2+c^2+ (a^2+b^2+c^2 + 2abc + 1)$]$-1$
$\geq $[$a^2+b^2+c^2+2(ab+bc+ca)$]$-1$
$$=(a+b+c)^2-1$$
$$=8.$$
Bài toán được chứng minh xong. Đẳng thức xảy ra khi và chỉ khi $a = b = c = 1$.

Bài toán 1.11. (Nguyễn Văn Huyện).
Cho $a, b, c$ là ba số thực dương. Chứng minh bất đẳng thức
$$2(a^2+b^2+c^2)+2abc+10 \geq 6(a+b+c), \ \ \ (1.11.1)$$

Lời Giải. Nhân 2 và hai vế của bất đẳng thức, ta được
$$2(a^2+b^2+c^2) + 2abc +10 \geq 6(a + b + c).$$
Bất đẳng thức cần chứng minh tương đương với mỗi bất đẳng thức sau đây
$$[a^2 + b^2 + c^2 + 2(ab + bc + ca)] + a^2 + b^2 + c^2 + 2abc + 10 \geq 6(a + b + c) + 2(ab + bc + ca),$$
$$(a + b + c)2 + a^2 + b^2 + c^2 + 2abc +10 \geq 6(a + b + c) + 2(ab + bc + ca),$$
$$[(a + b + c)2 — 6(a + b + c) + 9] + a^2 + b^2 + c^2 + 2abc +1 \geq 2(ab + bc + ca),$$
$$(a + b + c - 3)2 + [a^2 + b^2 + c^2 + 2abc + 1 - 2(ab + bc + ca)] \geq 0.$$
Là kết quả hiển nhiên đúng theo bất đẳng thức (1) nên ta có điều phải chứng minh.
Bài toán được chứng minh xong. Đẳng thức xảy ra khi $a = b = c = 1.$

Nhận Xét. Từ bài này nếu cho $a + b + c = 3$, ta sẽ nhận được "Bài toán 1.10", còn nếu ta cho $a^2 + b^2 + c^2 + abc = 4$ thì ta sẽ nhận được "Bài toán 1.9".

Bài toán 1.12. (Trần Nam Dũng, Hello IMO 2007, Toán Học và Tuổi Trẻ).
Chứng minh rằng với mọi $a, b, c$ dương, ta luôn có
$$abc + 2(a^2 + b^2 + c^2) + 8 \geq 5(a + b + c), \ \ (1.12.1)$$

Lời Giải 1. Sử dụng bất đẳng thức AM-GM, ta có
$$a+b+c=\frac{1}{3}.3.(a+b+c) \leq \frac{1}{6}[9+(a+b+c)^2].$$
Do đó ta chỉ cần chứng minh
$$12(a^2 + b^2 + c^2) + 6abc + 48 \geq 5 [(a + b + c)2 + 9] .$$
Bất đẳng thức này tương đương với
$$7(a^2 + b^2 + c^2) + 6abc + 3 \geq 10(ab + bc + ca),$$
$$4(a^2 + b^2 + c^2 - ab - bc - ca) + 3 [a^2 + b^2 + c^2 + 2abc +1 - 2(ab + bc + ca)] \geq 0,$$
đúng vì ta có $a^2 + b^2 + c^2 \geq ab + bc + ca$ (theo AM-GM) và
$$a^2 + b^2 + c^2 + 2abc +1 \geq 2(ab + bc + ca)$$ (theo (1)).
Bài toán được chứng minh xong. Đẳng thức xảy ra khi và chỉ khi $a = b = c = 1$

Lời Giải 2. Ta có bất đẳng thức cần chứng minh tương đương với
$$(a - 1)^2+ (b -1)^2 + (c -1)^2 + [a^2 + b^2 + c^2 + abc + 5 - 3(a + b + c)] \geq 0,$$
là một kết quả hiển nhiên đúng theo bất đẳng thức (1.11.1), nên ta có điều phải chứng minh.

Còn nữa ....



#318610 Phương pháp đại số (phương pháp gien)

Đã gửi bởi Ban Biên Tập on 22-05-2012 - 22:50 trong Tài liệu, chuyên đề, phương pháp về Số học

8. Phương pháp đại số (Phương pháp gien)
Nếu từ một nghiệm của phương trình đã cho ta có quy tắc để xây dựng ra một nghiệm mới thì quy tắc đó chính là gien. Phương pháp gien là phương pháp dựa vào gien để tìm tất cả các nghiệm của phương trình đã cho từ các nghiệm cơ sở. Để tìm các nghiệm cơ sở, ta áp dụng bước lùi, tức là quy tắc ngược của quy tắc tiến nói trên. Minh họa tốt nhất cho ý tưởng này là phương trình Pellphương trình Markov. Ta bắt đầu bằng phương trình Pell.

Phương trình Pell cổ điển là phương trình dạng, $x^2 – Dy^2 = 1$ trong đó $D$ là số nguyên dương không chính phương. Nếu $D = k^2$ thì từ phân tích $(x – ky)(x + ky) = 1$ ta suy ra phương trình đã cho có các nghiệm nguyên duy nhất là $(±1, 0)$. Trong trường hợp $D$ bất kỳ thì $(±1, 0)$ cũng là nghiệm của phương trình Pell. Ta gọi nghiệm này là nghiệm tầm thường.


Với số nguyên dương $D$ không chính phương cho trước, đặt
$$S = \{ (x, y) \in \mathbb{N}^* \times \mathbb{N}^* | x^2 – Dy^2 = 1 \}$$
là tập hợp tất cả các nghiệm nguyên dương của phương trình Pell
$$x^2 – Dy^2 = 1, \ \ (1)$$.


Ta có định lý quan trọng sau
Định lý 8.1. Nếu $D$ là số nguyên dương không chính phương thì $S \ne \varnothing$, tức là phương trình $(1)$ có nghiệm không tầm thường.

Chứng minh định lý này khá phức tạp, dựa vào lý thuyết liên phân số hoặc phương pháp hình học. Tuy nhiên, về mặt ứng dụng (trong các bài toán phổ thông), định lý này là không thực sự cần thiết vì với $D$ cho trước, ta có thể tìm ra một nghiệm nguyên dương của $(1)$ bằng phương pháp thử và sai. Ta bỏ qua định lý này và chuyển sang định lý mô tả tất cả các nghiệm của $(1)$ khi biết nghiệm cơ sở.


Với $(x, y), (x’, y’) \in S$ ta có nếu $x > x’$ thì $y > y’$. Do đó có thể định nghĩa $(x, y) > (x’ y’) \Leftrightarrow x > x’$. Với thứ tự này, $S$ là một tập sắp thứ tự tốt. Gọi $(a, b)$ là phần tử nhỏ nhất của $S$ theo thứ tự trên. Ta gọi $(a, b)$ là nghiệm cơ sở của $(1)$.

Định lý 8.2. Nếu $(a, b)$ là nghiệm cơ sở của $(1)$ và $(x, y)$ là một nghiệm nguyên dương tùy ý của $(1)$ thì tồn tại số nguyên dương $n$ sao cho $x + y \sqrt{D} = (a + b \sqrt{D})n$ và từ đó mọi nghiệm của $(1)$ đều có thể tìm được bởi công thức
\[x = \frac{{{{(a + b\sqrt D )}^n} + {{(a - b\sqrt D )}^n}}}{2},\;y = \frac{{{{(a + b\sqrt D )}^n} - {{(a - b\sqrt D )}^n}}}{{2\sqrt D }}\ \ \ (2)\].
Chứng minh: Nhận xét rằng nếu $(x, y)$ là nghiệm của $(1)$ thì
$$x’ = ax – Dby ; y’ = ay – bx $$
cũng là nghiệm của $(1)$ (có thể không nguyên dương)

Trước hết, do $\sqrt{D}$ vô tỷ nên nếu $x + y\sqrt{D} = (a + b\sqrt{D})n$ thì $x - y\sqrt{D} = (a - b\sqrt{D})n$ và từ đó $x^2 – Dy^2 = (a + b\sqrt{D})n(a - b\sqrt{D})n = (a^2 – Db^2)n = 1$, suy ra $(x, y)$ là nghiệm của $(1)$ và ta có công thức như trên.

Tiếp theo, giả sử không phải nghiệm nào của $(1)$ cũng có dạng $(2)$. Gọi $(x^*, y^*)$ là nghiệm nhỏ nhất không có dạng $(2)$ thì rõ ràng $x^* > a, y^* > b$.
Theo nhận xét:
$$x’ = ax^* - Dby^* ; y’ = ay^* - bx^* \ \ (3)$$
là nghiệm của $(1)$

Dễ dàng kiểm tra được rằng
$1) x^* > x’ > 0$
$2) y^* > y’ > 0$.

Từ đó, do tính nhỏ nhất của $(x^*, y^*)$, tồn tại $n$ nguyên dương sao cho $x’+ y’\sqrt{D} = (a+b\sqrt{D})n$. Giải hệ $(3)$ với ẩn là $(x^*, y^*)$, ta được (chú ý $a^2 – Db^2 = 1$)
$$x^* = ax’ + Dby’, y^* = ay’ + bx’$$
Từ đó
$$ x^*+ y^*\sqrt{D} = ax’ + Dby’ + (ay’+bx’)\sqrt{D} = (a+b\sqrt{D})(x’+y’\sqrt{D}) = (a+b\sqrt{D})^{n+1}$$
mâu thuẫn!.
Vậy điều giả sử là sai và $(2)$ là tất cả các nghiệm của $(1)$.

Tiếp theo, ta xét phương trình dạng Pell, tức là phương trình dạng$$x^2 – Dy^2 = k \ \ (4)$$
trong đó $D$ không chính phương và $k \in \{ 0, 1 \}$.
Ta có một số nhận xét sau
+ Không phải với cặp $D, k$ nào phương trình $(4)$ cũng có nghiệm. Ví dụ: phương trình $x^2 – 3y^2 = -1$.
+ Nếu phương trình $(4)$ có nghiệm nguyên dương thì nó có vô số nghiệm nguyên dương. Lý do: nếu $(x, y)$ là nghiệm của $(4)$ thì
$$x’ = ax + Dby ; y’ = ay + bx$$
cũng là nghiệm của $(4)$, trong đó $(a, b)$ là nghiệm cơ sở của phương trình.

Như thường lệ, ta đặt
$$S = \{ (x, y) \in \mathbb{N}^* \times \mathbb{N}^* | x^2 – Dy^2 = k \}$$
và gọi $(a, b)$ là nghiệm cơ sở của phương trình Pell tương ứng $x^2 – Dy^2 = 1$. Nghiệm $(x_0, y_0)$ thuộc S được gọi là nghiệm cơ sở của $(4)$ nếu không tồn tại $(x’, y’) \in S$ sao cho
$$x = ax’ + Dby’ ;y = ay’ + bx’$$
Gọi $S_0$ là tập hợp tất cả các nghiệm cơ sở. Ta có định lý quan trọng sau:


Định lý 8.3. Với mọi $D, k$ ta có $|S_0| < \infty$.
Chứng minh: Nếu $S_0 = \varnothing$ thì $|S_0| = 0 < \infty$. Tiếp theo giả sử $S_0 \neq \varnothing $. Gọi $(x, y)$ là một nghiệm cơ sở nào đó của $(4)$. Xét hệ
$$\left\{\begin{matrix}ax’ + Dby’ = x\\ ay’ + bx’ = y\end{matrix}\right.$$
có nghiệm $x’ = ax – Dby, y’ = ay – bx$. Dễ dàng chứng minh được $(x’)^2 – D(y’)^2 = 1$. Vì $(x, y) \in S_0$ nên theo định nghĩa $(x’, y’) \notin S$. Điều này xảy ra khi và chỉ khi
\[\left[ \begin{array}{l} x' \le 0\\ y' \le 0\end{array} \right.\; \Leftrightarrow \left[ \begin{array}{l} ax \le Dby\\ ay \le bx \end{array} \right. \Leftrightarrow \left[ \begin{array}{l} {x^2} \le - kD{b^2}\ \ \ (5)\\ {y^2} \le k{b^2}\ \ \ \ \ \ (6) \end{array} \right.\]

Nếu $(5)$ xảy ra thì ta có đánh giá $y^2 = \frac{x^2-k}{D} \leq - \frac{k(Db^2+1)}{D}$
Nếu $(6)$ xảy ra thì ta có $x^2 = Dy^2 + k \leq Dkb^2 + D$.
Trong cả hai trường hợp, ta có $|S_0| < \infty$.

Cuối cùng, chú ý rằng từ một nghiệm $(x, y)$ bất kỳ của $(4)$ không thuộc $S_0$, bằng cách đi ngược xuống bằng công thức $x’ = ax – Dby, y’ = ay – bx$ ta luôn có thể đi đến một nghiệm cơ sở của $(4)$. Như vậy, với định lý trên, phương trình dạng Pell đã được giải quyết hoàn toàn. Dưới đây chúng ta xem xét một ví dụ:

Ví dụ 8.1. Tìm tất cả các nghiệm nguyên dương của phương trình $x^2 – 5y^2 = –4, \ \ (1)$.
Lời giải: Bằng phép thử tuần tự, ta tìm được nghiệm cơ sở của phương trình $x^2 – 5y^2 = 1$ là $(9, 4)$. Theo phép chứng minh định lý 8.3, nghiệm cơ sở của $(1)$ thỏa mãn
$$x^2 \leq 4.5.42 ,y^2 \leq \frac{4.5.42 + 4}{5} \Rightarrow x < 17, y < 9$$
Dùng phép thử tuần tự, ta tìm được hai nghiệm cơ sở là $(1, 1)$ và $(11, 5)$. Từ hai nghiệm này, bằng công thức
$$x’ = 9x + 20y, y’ = 4x +9y$$
ta tìm được tất cả các nghiệm của $(1)$.

Với phép giải phương trình dạng Pell, trên thực tế ta đã có thể giải tất cả các phương trình Diophant bậc 2, tức là phương trình dạng
$$ax^2 + bxy + cy^2 + dx + ey + f = 0$$
Dựa vào lý thuyết đường cong bậc 2, ta có thể đưa phương trình trên về một trong các dạng chính sau
+ Dạng ellip: $ax^2 + by^2 = c (a, b, c > 0)$ – có hữu hạn nghiệm, giải bằng phương pháp thử và sai
+ Dạng parabol: $ax^2 + by + c$ - giải bằng đồng dư bậc 2
+ Dạng hypebol: $ax^2 – by^2 = c$ - phương trình dạng Pell
Ngoài ra còn có các dạng suy biết như hai đường thẳng cắt nhau, hai đường thẳng song song, ellip ảo … Dưới đây, ta xét một ví dụ áp dụng:

Ví dụ 8.2: Tìm tất cả các cặp số nguyên dương $(m, n)$ thỏa mãn phương trình
$$m(m+1) + n(n+1) = 3mn$$
Lời giải: Xét phương trình đã cho như phương trình bậc 2 theo $m$
$$m^2 – (3n-1)m + n(n+1) = 0$$
Phương trình này có nghiệm nguyên dương khi và chỉ khi $\Delta$ là số chính phương, tức là
$$(3n-1)2 – 4n(n+1) = y^2 \Leftrightarrow y^2 – 5(n-1)^2 = - 4 $$
ta thu được phương trình dạng Pell mà ta đã biết cách giải.

Đặc biệt với $k = -1$, phương trình $x^2 – Dy^2 = -1$ được gọi là phương trình Pell loại 2. Định lý dưới đây cho chúng ta điều kiện có nghiệm của phương trình này.

Định lý 8.4. Cho d là một số nguyên dương không chính phương. Xét phương trình
$$x^2 – dy^2 = -1 \; \; \ (1)$$
Gọi $(a, b)$ là nghiệm nhỏ nhất của phương trình Pell liên kết với phương trình $(1$), tức là phương trình
$$x^2 – dy^2 = 1 \; \; (1’)$$
Khi đó phương trình $(1)$ có nghiệm khi và chỉ khi hệ sau có nghiệm nguyên dương
$$\left\{ \begin{array}{l} a = x^2 + dy^2 \; \; \; (2) \\ b = 2xy \end{array} \right.$$.
Chứng minh. Giả sử $(x_0; y_0)$ là nghiệm nguyên dương của hệ phương trình $(2)$, vì $(a ; b)$ là nghiệm nguyên dương của phương trình Pell liên kết nên
$$a^2 – db^2 = 1 $$
Do vậy, từ $(2)$ ta có
$$(x_0 + dy_0)^2 – d(2x_0y_0)^2 = 1 \Leftrightarrow (x_0^2 – dy_0^2)^2 = 1 \Leftrightarrow x_0^2 – dy_0^2 = 1 \vee x_0^2 – dy_0^2 = -1$$.
Nếu $x_0^2 – dy_0^2= 1$ thì $(x_0, y_0)$ là nghiệm nguyên dương của phương trình Pell liên kết. Do $(a, b)$ là nghiệm nhỏ nhất của $(1’)$ nên $x_0 \ge a, y_0 \ge b$, mà $a = x_0^2 + dy_0^2 \Rightarrow x_0 \ge x_0^2 + dy_0^2$, vô lý!
Nếu $x_0^2 – dy_0^2 = - 1$ thì $(x_0; y_0)$ là nghiệm nguyên dương của phương trình $(1)$.

Đảo lại: Giả sử phương trình $(1)$ có nghiệm nguyên dương. Khi đó gọi $(x_0; y_0)$ là nghiệm nguyên dương nhỏ nhất của $(1)$. Ta sẽ chứng minh $(x_0; y_0)$ chính là nghiệm nguyên dương của hệ $(2)$.

Thật vậy, đặt $u = x_0^2 + dy_0^2$, $v = 2u_0v_0$ thì $u^2 – dv^2 = (x_0^2 + dy_0^2)^2 – d(2x_0y_0)^2 = (x_0^2-dy_0^2)^2 = 1$ (vì $x_0^2 – dy_0^2 = -1$). Suy ra $u, v$ là nghiệm của phương trình $(1’)$. Do $(a; b)$ là nghiệm nguyên dương nhỏ nhất của $(1’)$ nên $u \ge a, v \ge b$.

Ta chứng minh $u = a, v = b$.
Giả sử trái lại $u > a, v > b$ (vì $u > a \Leftrightarrow v > b$). Ta có $0 < a - b\sqrt d < (a - b\sqrt d )(a + b\sqrt d ) = {a^2} - d{b^2} = 1$. Do đó
\[(a - b\sqrt d )({x_0} + {y_0}\sqrt d ) < {x_0} + {y_0}\sqrt d \Leftrightarrow a{x_0} - bd{y_0} + (a{y_0} - b{x_0})\sqrt d < {x_0} + {y_0}\sqrt d \]
Từ $u > a, v > b$ suy ra $a + b\sqrt d < u + v\sqrt d = {({x_0} + {y_0}\sqrt d )^2}$. Do vậy
$$- \left( ax_0-bdy_0 \right) + \left( ay_0-bx_0 \right) \sqrt{d} = \left( a+b \sqrt{d} \right) \left( -x_0+y_0 \sqrt{d} \right) < \left( x_0+y_0 \sqrt{d} \right)^2 \left( -x_0+y_0 \sqrt{d} \right)$$
Hay $$- \left( ax_0-bdy_0 \right)+ \left(ay_0 -bx_0 \right) \sqrt{d} < \left( dy_0^2-x_0^2 \right) \left( x_0+y_0 \sqrt{d} \right) = x_0+y_0 \sqrt{d}.$$
Đặt $s = ax_0 – bdy_0$; $t = ay_0 – bx_0$, ta có
$$\begin{array}{l} s+t \sqrt{d} < x_0+y_0 \sqrt{d} \; \; \; \; \; (3) \\ -s+t \sqrt{d} < x_0+y_) \sqrt{d} \; \; \; (4) \end{array}$$
Ta thấy
$$\begin{aligned} s^2 – dt^2 & = (ax_0^2 – bdy_0)^2 – d(ay_0-bx_0)^2 \\ & = a^2(x_0^2-dy_0^2) + b^2d(dy_0^2-x_0^2) \\ & = -a^2 + db^2 \\ & = -1. \end{aligned}$$
Vậy $s^2 – dt^2 = -1$, suy ra $s \ne 0$ (vì nếu $s = 0$ thì $dt^2 = 1$ suy ra $d = 1$, vô lý).

Ta chứng minh $t > 0$.
Ta có $$t > 0 \Leftrightarrow ay_0 > bx_0 \Leftrightarrow a^2y_0^2 > b^2x_0^2 \Leftrightarrow (1+db^2)y_0^2 > b^2(dy_0^2-1) \Leftrightarrow y_0^2 > -b^2.$$ Bất đẳng thức cuối cùng đúng do đó $t > 0$.

Do $s \ne 0$ nên $s > 0$ hoặc $–s > 0$.
Nếu $s > 0$ thì $(s; t)$ là nghiệm nguyên dương của phương trình $(1)$. Mà $(x_0; y_0)$ là nghiệm nguyên dương bé nhất của phương trình này nên $s \ge x_0, t \ge y_0$ suy ra $$s+ t \sqrt{d} \ge x_0+y_0 \sqrt{d}$$ trái với $(3)$.

Trường hợp $–s > 0$ thì $(-s; t)$ là nghiệm nguyên dương của phương trình $(1)$ và lý luận tương tự, ta đi đến một bất đẳng thức mâu thuẫn với $(4)$.
Vậu $(u; v) = (a; b)$.


Nhận xét. Thực tế là ta đã chứng minh kết quả sau : Nếu $(a ; b)$ là nghiệm nhỏ nhất của $(1’)$ và $(m, n)$ là nghiệm nhỏ nhất của $(1)$ thì $a = m^2 + dn^2, b = 2mn.$

Một điều thú vị khác nữa là phương trình dạng $Ax^2 – By^2 = 1$ cũng có mối liên quan chặt chẽ đến phương trình $x^2 – Aby^2 = 1$. Ta có kết quả sau:

Định lý 8.5. Cho phương trình $$Ax^2 – By^2 = 1 \; \; \; \; (1)$$
với $A$ và $AB$ không chính phương. Gọi $(a, b$) là nghiệm nhỏ nhất của phương trình Pell kết hợp
$$x^2 – ABy^2 = 1 \; \; \; \; (2)$$
Giả sử phương trình $(1)$ có nghiệm và $(x_0; y_0)$ là nghiệm nhỏ nhất của nó thì $(x_0; y_0)$ là nghiệm duy nhất của hệ phương trình
$$\left\{ \begin{array}{l} a = Ax^2 + By^2 \\ b = 2xy \end{array} \right.$$


Chứng minh. Giả sử $(x_0; y_0)$ là nghiệm nhỏ nhất của $(1)$. Đặt $u = Ax_0^2 + By_0^2, v = 2x_0y_0$ thì ta có
$$u^2 – ABv^2 = (Ax_0^2 + By_0^2)^2 – AB(2x_0y_0)^2 = (Ax_0^2 – By_0^2)^2 = 1.$$
Chứng tỏ $(u; v)$ là nghiệm của phương trình $(2)$. Mà $(a; b)$ là nghiệm nhỏ nhất của phương trình này nên $u \ge a, v \ge b.$
Ta chứng minh $u = a, v = b.$
Thật vậy, giả sử trái lại $u > a ; v > b.$

\[a - b\sqrt {AB} < (a - b\sqrt {AB} )(a + b\sqrt {AB} ) = {a^2} - AB{b^2} = 1\]
\[ \Rightarrow (a - b\sqrt {AB} )(\sqrt A {x_0} + \sqrt B {y_0}) < (\sqrt A {x_0} + \sqrt B {y_0})\]
\[ \Rightarrow (a{x_0} - Bb{y_0})\sqrt A + (a{y_0} - Ab{x_0})\sqrt B < (\sqrt A {x_0} + \sqrt B {y_0})\]
Lại có
\[\begin{array}{l}(a + b\sqrt {AB} ) < (u + v\sqrt {AB} ) = {\left( {\sqrt A {x_0} + \sqrt B {y_0}} \right)^2}\\ \Rightarrow (a{x_0} - Bb{y_0})\sqrt A - (a{y_0} - Ab{x_0})\sqrt B = (a + b\sqrt {AB} )(\sqrt A {x_0} - \sqrt B {y_0}) \\ < {(\sqrt A {x_0} + \sqrt B {y_0})^2}(\sqrt A {x_0} - \sqrt B {y_0}) = (\sqrt A {x_0} + \sqrt B {y_0}) \end{array}\]
Đặt $s = ax_0 – Bby_0, t = ay_0 – Abx_0$ thì các bất đẳng thức trên có thể viết lại thành
\[\begin{array}{l}s\sqrt A + t\sqrt B < {x_0}\sqrt A + {y_0}\sqrt B \ \ \ (3)\\ s\sqrt A - t\sqrt B < {x_0}\sqrt A + {y_0}\sqrt B \ \ \ (4)\end{array}\]
Tiếp theo, ta có $$(As^2 – Bt^2) = A(ax_0 – Bby_0)^2 – B(ay_0 – Abx_0)^2= (a^2 – ABb^2)(Ax_0^2 – By_0^2) = 1.1 = 1.$$
Ta thấy $s > 0$ vì
$$s > 0 \Leftrightarrow ax_0 > Bby_0 \Leftrightarrow a^2x_0^2 > B^2b^2y_0^2 \Leftrightarrow a^2x_0^2 > Bb^2(Ax_0^2-1) \Leftrightarrow (a^2-ABb^2)x_0^2 > - Bb^2 \Leftrightarrow x_0^2 > - Bb^2.$$
Bất đẳng thức cuối cùng đúng, do đó $s > 0$.
Ta thấy $t \ne 0$ vì
$$t = 0 \Leftrightarrow ay_0 = Abx_0 \Leftrightarrow a^2y_0^2 = A^2b^2x_0^2 \Leftrightarrow (ABb^2+1)y_0^2 = Ab^2(By_0^2+1) \Leftrightarrow y_0^2 = Ab^2$$
Điều này không thể xảy ra do $A$ không chính phương.

Nếu $t > 0$ thì $(s; t)$ là nghiệm nguyên dương của (1), mà (x0 ; y0) là nghiệm nhỏ nhất của $(1)$ $s \ge x_0 ; t \ge y_0$, do vậy $$s \sqrt{A}+ t \sqrt{B} \ge \sqrt{A}x_0+ \sqrt{B}y_0$$
Điều này mâu thuẫn với $(3)$.

Tương tự, với $t < 0$ thì $(s ; -t)$ là nghiệm nguyên dương của $(1)$ và ta cũng dẫn đến một bất đẳng thức mâu thuẫn với $(4)$.
Vậy $u = a, v = b$ hay $(x_0 ;y_0)$ là nghiệm của hệ trên.

Ví dụ 8.3. (Vietnam TST 2009) Cho $a, b$ là các số nguyên dương không chính phương sao cho $a.b$ cũng không chính phương. Chứng minh rằng ít nhất một trong hai phương trình
$$ax^2 – by^2 = 1$$ và $$ax^2 – by^2 = –1. $$
không có nghiệm nguyên dương.


Lời giải. Giả sử cả hai phương trình
$$ax^2 – by^2 = 1 \; \; \; \; \; (1)$$

$$bx^2 – ay^2 = 1 \; \; \; \; \; (2)$$
có nghiệm .
Gọi $(m ; n)$ là nghiệm nhỏ nhất của phương trình $x^2 – aby^2 = 1$, $(x_1;y_1)$ là nghiệm nhỏ nhất của phương trình $(1)$ và $(x_2,y_2)$ là nghiệm nhỏ nhất của phương trình $(2)$. Áp dụng định lý 8.5, ta có
$$\left\{ \begin{array}{l} m = ax_1^2 + by_1^2 \\ n = 2x_1y_1 \end{array} \right.$$
và $$\left \{ \begin{array}{l} m = bx_2^2+ ay_2^2 \\ n = 2x_2y_2 \end{array} \right.$$
Do $ax_1^2 = by_1^2 + 1$ và $ay_2^2 = bx_2^2 – 1$ nên từ đây ta suy ra
$$ax_1^2 + by_1^2= bx_2^2 + ay_2^2 \Leftrightarrow 2by_1^2 + 1 = 2bx_2^2 – 1\Leftrightarrow b(x_2^2-y_1^2) = 1$$
Điều này không thể xảy ra do $b > 1$


Phương trình Markov cổ điển là phương trình dạng
$$x_1^2+x_2^2+ \cdots + x_n^2 = kx_1 \cdots x_n \; \; \; \; (1)$$
Ở đây $n$ và $k$ là các tham số nguyên dương. Trường hợp riêng khi $n = k = 3$ – phương trình
$$x^2+y^2+z^2 = 3xyz \; \; \; \; (2)$$
được nghiên cứu chi tiết trong bài báo của A.A Markov về dạng toàn phương dương đăng ở Báo cáo VHL KH Liên Xô năm 1951; “dạng Markov”, liên quan chặt chẽ đến phương trình dạng $(2)$ được sử dụng trong lý thuyết xấp xỉ các số vô tỷ bằng các số hữu tỷ.

Đầu tiên, ta chú ý đến một tính chất thú vị của phương trình Markov. Nếu phương trình $(1)$ có một nghiệm thì nó sẽ có rất nhiều nghiệm và có thể tạo ra các nghiệm đó bằng cách sau đây. Ta sẽ coi một biến, chẳng hạn $x_n$, là “ẩn số”, còn tất cả các biến khác như các tham số. Khi đó, vì phương trình
$$x^2 – kx_1 \cdots x_{n-1}x + x_1^2+…+ x_{n-1}^2 = 0$$
là phương trình bậc 2 theo x và có nghiệm $x = x_n$, nên nó có nghiệm nguyên thứ hai $x’_n = u$; theo định lý Viet ta có
$$u = kx_1 \cdots x_{n-1} – x_n = (x_1^2+ \cdots + x_{n-1}^2)/x_n \; \; \; \; (3)$$
Chú ý rằng $u < x_n$ khi và chỉ khi
$$x_1^2+ \cdots + x_{n-1}^2 kx_1 \cdots x_{n-1} \; \; \; \; (4)$$
Quá trình này có thể thực hiện với mọi biến số $x_j$ trong vai trò của $x_n$. Nhưng chỉ đối với một biến - biến lớn nhất là có thể xảy ra $(4)$ và ta thu được nghiệm mới $(x_1,x_2, …, x’_n)$ “nhỏ hơn” nghiệm cũ (thứ tự theo tổng các biến); như vậy, theo đa số là các nghiệm tăng lên và ta có cây nghiệm.

Tiếp theo, trừ những trường hợp đặc biệt, ta sẽ giả sử rằng $x_1 \le x_2 \le \cdots \le x_n$. Ta sẽ nói nghiệm $(x_1,x_1, \cdots , x_n)$ là nghiệm gốc (nghiệm cơ sở), nếu
$$x_1^2+ \cdots + x_{n-1}^2\ge x_n^2 \Leftrightarrow 2x_n \le kx_1 \cdots x_{n-1} \; \; \; (5)$$
(từ nghiệm này, tất cả các nhánh cây đi đến các nghiệm bên cạnh, đều tăng)


Bổ đề 8.4. Nếu phương trình $(1)$ có nghiệm nguyên dương thì nó có nghiệm gốc.

Bổ đề 8.5. Nếu $n > 2$, $(x_1,x_2, \cdots, x_n)$) là nghiệm gốc, ngoài ra, $x_1 \le x_2 \le \cdots \le x_n$. Khi đó
$$x_1 \cdots x_{n-2} \le \dfrac{2(n-1)}{k}.$$


Chứng minh:
$$\begin{aligned} kx_1 \cdots x_{n-2}x_{n-1}^2 & \le kx_1 \cdots x_{n-2}x_{n-1}x_n \\ & = x_1^2+ \cdots + x_{n-1}^2+ x_n^2 \\ & \le 2(x_1^2+ \cdots + x_{n-1}^2) \\ & \le 2(n-1)x_{n-1}^2 \end{aligned}$$

Bổ đề 8.6. Nếu $x_1 \le x_2 \le \cdots \le x_n$ là các số nguyên dương bất kỳ thoả mãn điều kiện $1 < x_n^2 \le x_1^2+ \cdots + x_{n-1}^2$, thì tỷ số $R = \dfrac{x_1^2+ \cdots + x_n^2}{x_1x_2 \cdots x_n}$ không vượt quá $\dfrac{n+3}{2}.$.

Định lý 8.7. Nếu phương trình $(1)$ có nghiệm và $n \ne k$, thì $n \ge 2k – 3$ khi $n \ge 5$ và $n > 4k – 6$ khi $n = 3, n = 4.$

Từ các định lý và bổ đề trên, với $n$ cho trước, việc tìm tất cả các giá trị $k$ sao cho $(1)$ có nghiệm thực hiện được dễ dàng. Hơn nữa, phương pháp gien được sử dụng ở trên có thể áp dụng cho các phương trình dạng tương tự, ví dụ phương trình $(x+y+z)^2 = kxyz$.

Cuối cùng là một ví dụ khác về ứng dụng của gien

Ví dụ 8.3: (Iran 2001) Giả sử $x, y, z$ là các số nguyên dương thỏa mãn điều kiện xy = z2 + 1. Chứng minh rằng tồn tại các số $a, b, c$ và d sao cho $x = a^2+b^2, y = c^2+d^2$ và $z = ac+ bd.$

Bài tập
8.1 (Ailen 1995) Tìm tất cả các số nguyên a sao cho phương trình $$x^2 + axy + y^2 = 1$$ có vô số nghiệm nguyên phân biệt $x, y.$
8.2. (Đài Loan 1998) Tồn tại hay không nghiệm của phương trình
$$x^2+y^2+z^2+u^2+v^2 = xyzuv – 65$$
trong tập hợp các số nguyên lớn hơn $1998$?
8.3. (Việt Nam 1999) Cho hai dãy số $(x_n)$, $(n=0, 1, 2, …)$ và $(y_n)$, $(n=0, 1, 2, …)$ được xác định
8.4. (Việt Nam 2002) Tìm tất cả các số nguyên dương n sao cho phương trình
$$x+y+z+t=n \sqrt{xyzt}$$có nghiệm nguyên dương.
8.5. (Ba Lan 2002) Tìm tất cả các cặp số nguyên dương $x, y$ thỏa mãn phương trình $$(x+y)^2 – 2(xy)^2 = 1.$$
8.6. (Mỹ 2002) Tìm tất cả các cặp sắp thứ tự các số nguyên dương $(m, n)$ sao cho $mn – 1$ chia hết $m^2+n^2.$
8.7. (Việt Nam 2002, vòng 2) Chứng minh rằng tồn tại số nguyên $m \ge 2002$ và m số nguyên dương phân biệt $a_1,a_2, \cdots a_m$ sao cho
$$\prod_{i=1}^{m}a_i^2-4\sum_{i=1}^{m}a_i^2$$ là số chính phương.
8.8. (Việt Nam 2002, vòng 2) Tìm tất cả các đa thức $p(x)$ với hệ số nguyên sao cho đa thức
$$q(x) = (x^2+6x+10)(p(x))^2 – 1 $$
là bình phương của một đa thức với hệ số nguyên.
8.9. (THTT 6/181) Với giá trị nguyên dương nào của p thì phương trình $x^2+y^2 + 1 = pxy$ có nghiệm nguyên dương?
8.10. (THTT 4/202) Cho ba số nguyên $a, b, c; \ a > 0$, $ac – b^2 = p_1p_2 \cdots p_m$ trong đó $p_1p_2 \cdots p_m$ là các số nguyên tố khác nhau. Gọi $M(n)$ là số các cặp số nguyên $(x, y)$ thỏa mãn
$$ax^2 + 2bxy + cy^2 = n.$$
8.11. (Đề đề nghị IMO 95) Tìm số nguyên dương $n$ nhỏ nhất sao cho $19n+1$ và $95n+1$ đều là các số chính phương.
8.12. Tam giác với cạnh $3, 4, 5$ và tam giác với cạnh $13, 14, 15$ có các cạnh là các số nguyên liên tiếp và có diện tích nguyên. Hãy tìm tất cả các tam giác có tính chất như vậy.
8.13. Chứng minh rằng nếu cả $3n+1$ và $4n+1$ đều là các số chính phương thì $n$ chia hết cho $56$.
8.14*. Trong các hàng của tam giác Pascal, hãy tìm hàng có chứa ba số hạng liên tiếp lập thành một cấp số cộng.
8.15. (Mỹ 1986) Tìm số nguyên dương $n > 1$ nhỏ nhất sao cho trung bình bình phương của $n$ số nguyên dương đầu tiên là một số nguyên.
8.16. (IMO 88) Nếu $a, b, q = \dfrac{a^2+b^2}{ab+1}$ là các số nguyên dương thì $q$ là số chính phương.
8.17. (MOCP 03) Tìm tất cả giá trị $n$ sao cho phương trình $(x+y+z)^2 = nxyz$ có nghiệm nguyên dương.
8.18. (PTNK 03). Tìm tất cả các số nguyên dương $k$ sao cho phương trình $$x^2 - (k^2-4)y^2 = - 24$$ có nghiệm nguyên dương.
8.19. Chứng minh rằng phương trình $(k^2-4)x^2 – y^2 = 1$ không có nghiệm nguyên với mọi $k>3$.
8.20. (Mathlinks) Cho A là tập hợp hữu hạn các số nguyên dương. Chứng minh rằng tồn tại tập hợp hữu hạn các số nguyên dương B sao cho $A \in B$ và $\prod _{x \in \boldsymbol{B}}x=\sum_{x \in \boldsymbol{B}}x^2.$
8.21. (AMM 1995) Cho x, y là các số nguyên dương sao cho $xy + x$ và $xy + y$ là các số chính phương. Chứng minh rằng có đúng một trong hai số $x, y$ là số chính phương.


Tài liệu tham khảo
1. Jean-Marie Monier. Đại số 1 – giáo trình toán tập 5, NXBGD-Dunod 1999.
2. Hà Huy Khóai - Phạm Huy Điển. Số học thuật toán, NXB ĐHQG HN 2003.
3. Lê Hải Châu. Các bài thi học sinh giỏi Toán PTTH toàn quốc, NXBGD 1994.
4. Nguyễn Sinh Nguyên, Nguyễn Văn Nho, Lê Hoành Phò. Tuyển tập các bài dự tuyển Olympic Toán học Quốc tế 1991-2001, NXBGD 2003.
5. Nguyễn Văn Nho. Olympic Toán học châu Á – Thái Bình Dương 1989-2002, NXBGD 2003.
6. Tập thể tácgiả. Tuyển tập 5 năm Tạp chí Toán học và Tuổi trẻ, NXBGD 2003.
7. Arthur Engel. Problem Solving Strategies, Springer 1998
8. Goeoge Polya. Gabor Szego, Problems and Theorems in Analysis II, Springer 1976
9. Harvey Cohn. Advanced Number Theory, Dover Publications 1980
10. Titu Andreescu, Juming Feng. Mathematical Olympiads 1999-2000:Olympiads Problems from Around the World, MMA 2000.
11. Titu Andreescu, Juming Feng, Hojoo Lee. Mathematical Olympiads 2001-2002: Olympiads Problems from Around the World, MMA 2002.
12. Titu Andreescu & Razvan Gelca. Mathematical Olympiads Challenge. Bikhauser 2000.
13. Walter Mientka & others. Mathematical Olympiads 1996-1997: Olympiads Problems from Around the World, MMA 1997.
14. Walter Mientka & others. Mathematical Olympiads 1997-1998: Olympiads Problems from Around the World, MMA 1998.
15. Б.О. Бугаенко. Уравнения Пелля, Москва 2001.
16. Д.Ф. Базылев. Диофантовы уравнения, Минск 1999.
17. А.О. Гельфонд. Решение уравнений в целых числах, Наука 1978.
18. В. Серпинский. О решений уравнений в целых числах, ФМЛ 1961.
19. В. Серпинский. 250 задач по элементарной теории чисел, ФМЛ 1968. Có bản dịch đề bài tiếng Việt do Sở GD Tp HCM in năm 1982.
20. Дэвенпорт. Высшая арифметика, Наука 1956.
21. Các tạp chí Kvant, AMM, Toán học & Tuổi trẻ, Toán học trong nhà trường.
22. Tư liệu Internet, đặc biệt là các website: www.mccme.ruwww.mathlinks.ro.


:)



#318210 Một số điều nên và không nên trong giảng dạy toán

Đã gửi bởi Ban Biên Tập on 21-05-2012 - 08:43 trong Dành cho giáo viên các cấp

Trong loạt bài này, tôi sẽ viết dần một số quan điểm của tôi về những điều nên và không nên trong giảng dạy. Những quan điểm này được rút ra từ kinh nghiệm bản thân, việc nghiên cứu các liệu về giáo dục, sự trao đổi với đồng nghiệp và sinh viên, và những suy nghĩ để làm sao dạy học tốt hơn. Tất nhiên có những quan điểm của tôi có thể còn phiến diện. Xin mời mọi người trao đổi, viết lên những quan điểm và kinh nghiệm của mình.
Hình đã gửi
Tôi sẽ chủ yếu nói về việc dạy toán, tuy rằng nhiều điểm áp dụng được cho hầu hết các môn học khác. Tôi sẽ dùng từ “giảng viên” để chỉ cả giảng viên đại học lẫn giáo viên phổ thông, từ “học sinh” (student) để chỉ học sinh sinh viên hay học viên ở mọi cấp học, từ phổ thông cho đến sau đại học. Tôi viết không theo thứ tự đặc biệt nào.
Hình đã gửi
1) Nên: Thỉnh thoảng thay đổi môn dạy nếu có thể. Nếu dạy một môn nhiều lần, thì cải tiến thường xuyên phương pháp và nội dung dạy môn đó.
Không nên: Dạy mãi năm này qua năm khác một môn, với giáo trình nhiều năm không thay đổi.
Các chức vụ quản lý lãnh đạo thường có nhiệm kỳ, và thường có nguyên tắc là không ai làm quá 2 nhiệm kỳ ở cùng 1 vị trí. Lý do là để tạo sự thay đổi cải tiến thường xuyên, tránh sự trì trệ. Ngay trong việc dạy học cũng vậy: một người mà dạy quá nhiều năm cùng một thứ, thì dễ dẫn đến nhàm chán trì trệ. Để tránh chuyện đó, có những cơ sở đại học có qui định là các môn học cũng có nhiệm kỳ: ai mà dạy môn nào đó được 4-5 năm rồi thì phải giao cho người khác đảm nhiệm, trừ trường hợp không tìm được người thay thNhiều khoa toán có phân chia việc dạy các môn cho các tổ bộ môn, ví dụ môn “phương trình vi phân” thì chỉ dành cho người của tổ bộ môn phương trình vi phân dạy. Việc phân chia như vậy có cái lợi là đảm bảo chất lượng dạy,đặc biệt là trong điều kiện trình độ giảng viên nói chung còn thấp, phải “chuyên môn hóa” trong việc dạy để đảm bản chất lượng tối thiểu. Tuy nhiên nó có điểm hạn chế, là nó tạo ra xu hướng người của tổ bộ môn nào sẽ chỉ biết chuyên ngành hẹp đấy, tầm nhìn không mở rộng ra. Ỏ một số trường đại học tiên tiến, nơi có nhiều giảng viên trình độ cao (và với nguyên tắc là đã là giáo sư hay giảng viên cao cấp thì đủ trình độ để dạy bất cứ môn nào trong các môn toán bắt buộc ở bậc cử nhân), công việc giảng dạy không phân chia theo tổ bộ môn hẹp như vậy, mà giảng viên (cao cấp) nào cũng có thể đăng ký dạy bất cứ môn nào ở bậc cử nhân.
Tất nhiên, việc thay đổi môn dạy đòi hỏi các giảng viên phải cố gắng hơn trong việc chuẩn bị bài giảng (mỗi lần đổi môn dạy, là một lần phải chuẩn bị bài giảng gần như từ đầu), nhưng đổi lại nó làm tăng trình độ của bản thân giảng viên, giúp cho giảng viên tìm hiểu những cái mới (mà nếu không đổi môn dạy thì sẽ không tìm hiểu, do sức ỳ). Đặc biệt là các môn ở bậc cao học: việc chuẩn bị bài giảng cho một môn cao học mới có thể giúp ích trực tiếp cho việc nghiên cứu khoa học của giảng viên.
Tôi có một số kinh nghiệm cá nhân về việc này. Ví dụ như một lần năm 1999 tôi nhận dạy 1 học kỳ cao học về hệ động lực Hamilton, và trong quá trình đọc tài liệu để chuẩn bị bài giảng cho môn đó, tôi phát hiện ra một số vấn đề cơ bản liên quan đến dạng chuẩn địa phương của hệ động lực chưa được nghiên cứu, và điều đó thúc đẩy tôi nghiên cứu được một số kết quả khá tốt. Năm 2008 tôi nhận dạy môn đại số (mở rộng trường và một ít đại số giao hoán) cho sinh viên toán năm thứ 4, tuy rằng trước đó tôi hầu như không đụng chạm đến những thứ đó. Việc dạy môn đại số đã giúp tôi nắm chắc thêm được một số kiến thức về đại số, ví dụ như hiểu thêm ý nghĩa của tính chất Noether (đây là tính chất đặc trưng của “đại số”, đối ngược với “giải tích”).
Tất nhiên có nhiều người, do điều kiện công việc, phải dạy cùng một môn (ví dụ như môn Toán lớp 12) trong nhiều năm. Để tránh trì trệ trong trường hợp đó, cần thường xuyên cải tiến phương pháp và nội dung giảng dạy (đưa vào những ví dụ minh họa mới và bài tập mới từ thực tế hiện tại, sử dụng những công nghệ mới và công cụ học tập mới, tìm các cách giải thích mới dễ hiểu hơn, v.v.)
2) Nên: Dạy và kiểm tra kiến thức học sinh theo lối “học để hiểu”
Không nên: Tạo cho học sinh thói quen học vẹt, chỉ nhớ mà không hiểu
Các nhà giáo dục học và thần kinh học trên thế giới đã làm nhiều phân tích và thí nghiệm cho thấy, khi bộ óc con người “hiểu” một cái gì đó (tức là có thể “make sense” cái đó, liên tưởng được với những kiến thức và thông tin khác đã có sẵn trong não) thì dễ nhớ nó (do thiết lập được nhiều “dây nối” liên quan đến kiến thức đó trong mạng thần kinh của não — một neuron thần kinh có thể có hàng chục nghìn dây nối đến các neuron khác), còn khi chỉ cố nhồi nhét các thông tin riêng lẻ vào não (kiểu học vẹt) mà không liên hệ được với các kiến thức khác đã có trong não, thì thông tin đó rất khó nhớ, dễ bị não đào thải.
Thực ra thì môn học nào cũng cần “hiểu” và “nhớ”, tuy rằng tỷ lệ giữa “hiểu” và “nhớ” giữa các môn khác nhau có khác nhau: ví dụ như ngoại ngữ thì không có gì phức tạp khó hiểu lắm nhưng cần nhớ nhiều (tất nhiên để nhớ được các câu chữ ngoại ngữ thì cũng phải liên tưởng được các câu chữ đó với hình ảnh hay ỹ nghĩa của chúng và với những thứ khác có trong não), nhưng toán học thì ngược lại: không cần nhớ nhiều lắm, nhưng phải hiểu được các kiến thức, và quá trình hiểu đó đòi hỏi nhiều công sức thời gian. Có những công thức và định nghĩa toán mà nếu chúng ta quên đi chúng ta vẫn có thể tự tìm lại được và dùng được nếu đã hiểu bản chất của công thức và định nghĩa đó, còn nếu chúng ta chỉ nhớ công thức và định nghĩa đó như con vẹt mà không hiểu nó, thì cũng không dùng được nó, và như vậy thì cũng không hơn gì người chưa từng biết nó. Ví dụ như công thức tính Christoffel symbol cho liên thông Riemann của một Riemannian metric là một công thức hơi dài, và tôi chẳng bao giờ nhớ được chính xác nó lâu tuy “mang tiếng” là người làm hình học vi phân: cứ mỗi lần đụng đến thì xem lại, nhớ được một lúc, rồi lại quên. Nhưng điều đó không làm tôi băn khoăn, vì tôi hiểu bản chất của Christoffel symbol và các tính chất cơ bản của liên thông Riemann, từ đó có thể tự nghĩ ra lại được công thức nếu cần thiết (tốn một vài phút) hoặc tra trên internet ra ngay.
Sinh viên ngày nay (là những chuyên gia của ngày mai) có thể tra cứu rất nhanh mọi định nghĩa, công thức, v.v., nhưng để hiểu chúng thì vẫn phải tự hiểu, không có máy móc nào hiểu hộ được. Cách đây 5-10 năm, theo thông lệ của những người dạy trước tôi, tôi thường không cho phép sinh viên mang tài liệu vào phòng thi trong các kỳ thi cuối học kỳ, và đề bài thi hay có 1 câu hỏi lý thuyết (tức là phát biểu đúng 1 định nghĩa hay định lý gì đó thì được điểm). Nhưng trong thời đại mới, việc nhớ y nguyên các định nghĩa và định lý có ít giá trị, mà cái chính là phải hiểu để mà sử dụng được chúng. Bởi vậy những năm gần đây, trong các kỳ thi tôi dần dần cho phép học sinh mang bất cứ tài liệu nào vào phòng thi, và đề thi không còn các câu hỏi “phát biểu định lý” nữa. Thay vào đó là những bài tập (tương đối đơn giản, và thường gần giống các bài có trong các tài liệu nhưng đã thay tham số) để kiểm tra xem học sinh có hiểu và sử dụng được các kiến thức cơ bản không.
Về mặt hình thức, chương trình học ở Việt Nam (kể cả bậc phổ thông lẫn bậc đại học) khá nặng, nhưng là nặng về “nhớ” mà nhẹ về “hiểu”, và trình độ trung bình của học sinh Việt Nam thì yếu so với thế giới (tất nhiên vẫn có học sinh rất giỏi, nhưng tỷ lệ học sinh giỏi thực sự rất ít, và cũng khó so được với giỏi của phương Tây). Vấn đề không phải là do người Việt Nam sinh ra kém thông minh, mà là do điều kiện và phương pháp giáo dục, chứ trẻ em gốc Việt Nam lớn lên ở nước ngoài thường là thành công trong đường học hành. Hiện tượng rất phổ biến ở Việt Nam là học sinh học thuộc lòng các “kiến thức” trước mỗi kỳ kiểm tra, rồi sau khi kiểm tra xong thì “chữ thầy trả thầy”. Việt Nam rất cần cải cách chương trình giáo dục theo hướng tăng sự “hiểu” lên, và giảm sự “học gạo”, “nhớ như con vẹt”. Tôi có phỏng vấn nhiều sinh viên tốt nghiệp loại giỏi ngành toán ở Việt Nam, nhưng khi hỏi một số kiến thức khá cơ bản thì nhiều em lại không biết. Lỗi không phải tại các em mà có lẽ tại hệ thống giáo dục. Nhiều thầy cô giáo chỉ khuyến khích học sinh làm bài kiểm tra giống hệt lời giải mẫu của mình, chứ làm kiểu khác đi, tuy có thể thú vị hơn cách của thầy thì có khi lại bị trừ điểm. Tôi đã chứng kiến trường hợp sinh viên chỉ đạt điểm thi 7-8 lại giỏi hơn sinh viên đạt điểm thi 9-10 vì kiểu chấm thi như vậy. Kiểu chấm điểm như thế chỉ khuyến khích học vẹt chứ không khuyến khích sự sáng tạo hiểu biết.
3) Nên: Dạy những cái cơ bản nhất, nhiều công dụng nhất
Không nên: Mất nhiều thời giờ vào những thứ ít hoặc không dùng đến
Trên đời có rất nhiều cái để học, trong khi thời gian và sức lực của chúng ta có hạn, và bởi vậy chúng ta luôn phải lựa chọn xem nên học (hay dạy học) cái gì. Nếu chúng ta phung phí quá nhiều thời gian vào những cái ít công dụng (hoặc thậm chí phản tác dụng, ví dụ như những lý thuyết về chính trị hay kinh tế trái ngược với thực tế), thì sẽ không còn đủ thời gian để học (hay dạy học) những cái quan trọng hơn, hữu ích hơn.
Tất nhiên, mức độ “quan trọng, hữu ích” của từng kiến thức đối với mỗi người khác nhau thì khác nhau, và phụ thuộc vào nhiều yếu tố như thời gian, hoàn cảnh, sở trường, v.v. Ví dụ như học nói và viết tiếng Việt cho đàng hoàng là không thể thiếu với người Việt, nhưng lại không cần thiết với người Nga. Những người muốn làm nghề toán thì phải học nhiều về toán, còn sinh viên đại học các ngành khác nói chung chỉ cần học một số kiến thức toán cao cấp cơ bản nhất mà sẽ cần trong công việc của họ. Những người muốn làm toán ứng dụng, thì ngoài các môn toán, cần phải học các môn mà họ định mang toán ứng dụng vào đó.
Ngay trong các môn toán, không phải các kiến thức nào cũng quan trọng như nhau. Và “độ quan trọng” và “độ phức tạp” là hai khái niệm khác nhau: không phải cái gì quan trọng cũng phức tạp khó hiểu, và không phải cái gì rắm rối khó hiểu cũng quan trọng. Giảng viên cần tránh dẫn dắt học sinh lao đầu vào những cái rắm rối phức tạp nhưng ít công dụng. Thay vào đó, cần dành nhiều thời gian cho những cái cơ bản, nhiều công dụng nhất. Nếu là cái vừa cơ bản và vừa khó, thì lại càng cần dành đủ thời gian cho nó, vì khí nắm bắt được nó tức là nắm bắt được một công cụ mạnh.
Một ví dụ là đạo hàm và tích phân. Đây là những khái niệm cơ bản vô cùng quan trọng trong toán học. Học sinh cần hiểu định nghĩa, bản chất và công dụng của chúng, và nắm được một số nguyên tắc cơ bản và công thức đơn giản, ví dụ như nguyên tắc Leibniz cho đạo hàm của một tích, hay công thức “đạo hàm của sin x bằng cos x”. Tuy nhiên nếu bắt học sinh học thuộc hàng trăm công thức tính đạo hàm và tích phân khách nhau, thì sẽ tốn thời gian vô ích vì phần lớn các công thức thức đó sẽ không dùng đến sau này, hoặc nếu dùng đến thì có thể tra cứu được dễ dàng. Một lần tôi thấy có một sách tiếng Việt về tính tích phân cho học sinh, dày hơn 150 trang, với rất nhiều công thức phức tạp dài dòng (ví dụ như công thức tính tính phân của một hàm số có dạng thương của hai biểu thức lượng giác), mà ngay những người làm toán chuyên nghiệp cũng rất hiếm khi cần đến. Thay vì tốn nhiều thời gian vào những công thức phức tạp mà không cần dùng đó, học những thứ cơ bản khác sẽ có ích hơn.
Một lần nhà xuất bản Springer có lần nhờ tôi làm phản biện cho 1 quyển sách về hình học vi phân và ứng dụng. Tôi đã khuyên Springer không in sách đó, và một trong các lý do là quyển sách chứa quá nhiều khái niệm mà ngay trong sách đó cũng không dùng đến. Ví dụ như khái niệm “không gian Lindeloff” được đưa vào ngay ở đầu sách, phát biểu thành 1 định nghĩa có đánh số hẳn hoi (chứ không phải là chỉ nhắc qua nó trong một “remark”), nhưng không dùng đến nó lúc nào trong sách, tôi không hiểu người viết sách đưa định nghĩa đó vào trong sách để làm gì.
Một ví dụ khác: các bất đẳng thức. Có những bất đẳng thức “có tên tuổi”, không phải vì nó “khó”, mà là vì nó có ý nghĩa (nó xuất hiện trong các vấn đề hình học, số học, phương trình vi phân, v.v.). Chứ nếu học một đống hàng ngàn bất đẳng thức mà không biết chúng dùng để làm gì, thì khá là phí thời gian. Phần lớn các bất đẳng thức (không kể các bất đẳng thức có tính tổ hợp) có thể được chứng minh khá dễ dàng bằng một phương pháp cơ bản, là phương pháp dùng đạo hàm hoặc sai phân. Phương pháp này học sinh phổ thông có thể học được, nhưng thay vào đó học sinh lại được học các kiểu mẹo mực để chứng minh bất đẳng thức. Các mẹo mực có ít công dụng, chỉ dùng được cho bài toán này nhưng không dùng được cho bài toán khác (bởi vậy mới là “mẹo mực” chứ không phải “phương pháp”). “Mẹo mực” có thể làm cho cuộc sống thêm phong phú, nhưng nếu mất quá nhiều thời gian vào “mẹo mực” thì không còn thời gian cho những cái cơ bản hơn, giúp tiến xa hơn. Như là trong công nghệ, có cải tiến cái đèn dầu đến mấy thì nó cũng không thể trở thành đèn điện.
Hồi còn nhỏ, có lần tôi đi thi học sinh giỏi (lớp 6 ?), có bài toán tìm cực đại. Tôi dùng đạo hàm tính ngay ra điểm cực đại, và có bạn khác cùng lớp cũng biết làm như vậy. Cách làm đó là do chúng tôi tự đọc sách mà ra chứ không được dạy. Nhưng khi viết lời giải thì lại phải giả vờ “đoán mò” điểm cực đại, rồi viết hàm số dưới dạng một số (giá trị tại điểm đó) cộng với một biểu thức hiển nhiên là không âm (ví dụ như vì có dạng bình phương) thì mới được điểm, chứ nếu viết đạo hàm thì mất hết điểm. Nếu như thầy giáo trừ điểm học sinh, vì học sinh giải bài thi bằng một phương pháp “cơ bản” nhưng “không có trong sách thầy”, thì điều đó sẽ góp phần làm cho học sinh học mẹo mực, thiếu cơ bản.
Qua phỏng vấn một số sinh viên đại học và cao học ngành toán của Việt Nam, tôi thấy họ được học nhiều môn “cao cấp”, nhưng vẫn thiếu kiến thức cơ bản. Ví dụ như họ học giải tích hàm, với những định lý trừu tượng khá là khó. Nhưng họ lại không biết công thức Parceval cho chuỗi Fourier là gì, trong khi chuỗi Fourier là một trong những khái niệm giải tích cơ bản và nhiều ứng dụng nhất của toán. Tôi không có ý nói giải tích hàm là “không cơ bản”. Nó là thứ cần thiết. Nhưng nếu những khái niệm và định lý của giải tích hàm chỉ được học một cách hình thức, không có liên hệ với chuỗi Fourier hay với các ví dụ cụ thể khác, thì đó là học “trên mây trên gió”.

Còn tiếp ... Theo http://zung.zetamu.net




#315429 Toán học trong dự báo thời tiết

Đã gửi bởi Ban Biên Tập on 09-05-2012 - 21:45 trong Toán học lý thú

Khí quyển của chúng ta, một cách cơ bản nhất, chính là một hệ khí động lực mà sự tiến triển của nó được mô tả bởi hệ các phương trình chuyển động chất lỏng Navier-Stokes cùng với các phương trình trạng thái và bảo toàn, goi tắt là hệ PES (1). Với một điều kiện ban đầu và điều kiện biên cho trước, bài toán dự báo thời tiết có thể được coi một cách hình thức là bài toán tìm nghiệm phụ thuộc tường minh vào không thời gian của hệ PES nói trên.

Hình đã gửi




Hình đã gửi
Hệ PES này áp dụng cho tất cả các quá trình trong khí quyển từ những xoáy bụi nhỏ mà chúng ta đôi khi bắt gặp trên các con phố, các cụm mây có phân bố như những gợn sóng nhỏ trên bầu trời, cho đến các cơn bão nhiệt đới với quy mô không gian khoảng 1000 km hay các quá trình khí hậu với quy mô thời gian nhiều năm. Tuy nhiên, gần 100 năm kể từ khi bài toán dự báo khí tượng được đặt ra một cách tường minh như vậy bởi Richarson, tất cả nhưng gì chúng ta có thể làm được lại chỉ là một bài toán dự báo thời tiết chỉ với hạn dài nhất đến thời điểm này là 2 tuần với một độ chính xác rất khiêm tốn (2) (tất cả chúng ta hẳn đã có không ít lần bắt gặp các bản tin dự báo sai, ngay cả với các trung tâm dự báo hàng đầu thế giới của Mỹ hay Châu Âu). Mấu chốt của tính dự báo rất khiêm tốn này nằm ở hai khó khăn chính. Thứ nhất, hệ phương trình Navier-Stokes chưa có lời giải chính xác cho đến tại thời điểm hiện tại sau gần 200 năm đưa ra bởi Navier năm 1822. Đây chính là bài toán thiên niên kỷ số ba của viện toán học Clay mà tầm quan trọng của nó đã đi vào tất cả các lĩnh vực sâu sắc nhất của đời sống hàng ngày. Khó khăn thứ hai là do quy mô không thời gian của các quá trình thời tiết biến đổi trong khoảng quá rộng từ các xoáy rối cỡ vài mm cho đến các quá trình sóng hành tinh với quy mô hàng ngàn km mà không một hệ thống thám sát nào có thể ghi nhận đầy đủ để chúng ta cỏ thể đo đạc và tìm hiểu chi tiết.

Cách tiếp cận lý thuyết duy nhất của các nhà khí tượng học đến thời điểm này là sử dụng một tập các giả thiết xấp xỉ gần đúng cho một hiện tượng nào đó để đơn giản hóa hệ PES cho hiện tượng này. Một ví dụ điển hình là các nghiên cứu lý thuyết về bão nhiệt đới. Sư phức tạp của hệ PES cũng như các hiểu biết vật lý‎ còn hết sức khiêm tốn về các quá trình chi tiết trong bão khiến các nhà nghiên cứu khí tượng không thể mô tả một cách giải tích tất cả các quá trình trong bão một cách đầy đủ. Thay vào đó, bão sẽ được chia ra làm các pha phát triển khác nhau mà tương ứng với nó là các xấp xỉ cho pha đó. Ví dụ, trong giai đoạn chín muồi của bão mà cường độ của chúng gần như không thay đổi theo thời gian, nhà khí tượng học Emanuel (Viện công nghệ Massachusetts) năm 1986 đã đề xuất một mô hình trong đó bão được xem như là một chu trình Carnot nhiệt với một cấu trúc đối xứng trục. Cụ thể, bão lấy nhiệt từ bề mặt đại dương với nhiệt độ cao (khoảng 300 K) và sau đó trao đổi với tầng nhiệt độ lạnh hơn ở tầng độ cao khoảng 18 km (khoảng 220 K) trong quá trình chuyển hóa năng lượng ẩn nhiệt có được từ bốc hơi nước sang động năng của bão. Như thế, cường độ cực đại mà một cơn bão có được sẽ được quy đinh bởi nhiệt độ bề mặt biển tương ứng. Đây là một kết luận rất quan trọng cho phép khảo sát mối liên hệ giữa sự ấm nóng toàn cầu và cường độ bão trong tương lai. Trong pha phát triển của bão, chúng ta tuy nhiên lại không thể giả thiết các cơn bão là ở trạng thái dừng như ở trên. Thay vào đó, chúng ta phải xấp xỉ hệ PES theo một cách khác mà cho phép chúng ta tìm được nghiệm phụ thuộc tường minh vào thời gian. Các mô hình lý‎ thuyết khác nhau cho pha phát triển này của bão đều cho thấy bão có thể xem như một hệ động lực không cân bằng và sự phát triển của bão chính là một quá trình hồi tiếp dương của một hệ bất ổn định. Nhưng ngay cả trong những xấp xỉ này, phần lớn các hệ đơn giản hóa vẫn quá phức tạp đến nỗi chưa ai chứng minh được nghiệm giải tích liệu có tồn tại, và nếu có tồn tại liệu nghiệm đó có duy nhất. Chỉ trong một vài tình huống rất đặc biệt lý‎ tưởng hóa thì chúng ta mới thu được các kết quả giải tích. Trong các trường hợp còn lại hệ phương trình thu được vẫn quá phức tạp để chúng ta có thể hi vọng vào một nghiệm đầy đủ.

Các bài toán khí tượng lý‎ thuyết một mặt là rất cần thiết bởi chúng cho phép chúng ta hiểu sâu hơn về quá trình động lực học trong khí quyển. Nhưng mặt khác rõ ràng là các khó khăn cả về mặt lý thuyết lẫn thực tế nêu ở trên lại không cho phép chúng ta tìm hiểu chi tiết bản chất của khí quyển như chúng ta trông đợi. Vậy dự báo thời tiết mà chúng ta theo dõi hàng ngày đang diễn ra bằng cách nào? Quy trình dự báo thời tiết nghiệp vụ hiện đại là tổ hợp của rất nhiều các cấu thành bao gồm các bộ phận xử lý ảnh vệ tinh, thu phát dữ liệu thám sát, kiểm tra đánh giá, v.v., nhưng cốt lõi nhất của tất cả các trung tâm dự báo hiện nay là một mô hình máy tính giải hệ PES nêu ở trên bằng phương pháp số, gọi tắt là dự báo thời tiết số (NWP). Nói một cách đơn giản, chúng ta chia khí quyển liên tục của chúng ta thành một khí quyển rời rạc được mô tả bởi một mạng các điểm nút theo các phương ngang và thẳng đứng với một độ phân giải cho trước. Hệ PES khi đó được chuyển thành một hệ các phương trình cho các điểm nút này, và chúng ta khi đó sẽ mô phỏng khí quyển rời rạc này trên các siêu máy tính. Các kết quả thu được từ các mô phỏng sẽ được xem như là trạng thái khí quyển trong tương lai và từ đó đưa ra các bản tin dự báo. Theo nghĩa này, NWP chính là bài toán tích phân số hệ phương trình đạo hàm riêng từ một điều kiện ban đầu và điều kiện biên cho trước. Do đặc thù của hệ PES này phụ thuộc mạnh vào điều kiện ban đầu, một cách rõ ràng là các mô hình dự báo thời tiết được xây dựng dựa trên hệ PES phải có được điều kiện ban đầu (biên) tốt nhất có thể. Các nghiên cứu của Lorenz từ đầu nhưng năm 1960 đã chỉ ra rằng một sự thay đổi nhỏ của điều kiện ban đầu có thể dẫn đến một sự thay đổi rất lớn của trạng thái tích phân theo thời gian. Sự phụ thuộc mạnh của dự báo thời tiết vào điều kiện ban đầu này đôi khi dẫn đến các kết quả hoàn toàn sai lệch so với trạng thái thực của khí quyển chỉ do một vài sai số quan trắc nào đó (3). Đây chính là giới hạn lớn nhất của bài toán dự báo thời tiết số vì dường như chúng ta sẽ không bao giờ có thể có được một hệ thống thám sát toàn cầu đủ chính xác với một độ phân giải tuỳ ý. Các nghiên cứu lý thuyết của Lorenz từ năm 1960 đã đặt ra một giới hạn trên cho hạn dự báo của các mô hình số là vào khoảng 2 tuần, và đây được xem như là điểm tới hạn của bài toán dự báo thời tiết NWP.

Song song với bài toán tạo trường ban đầu tốt nhất, lớp bài toán có tầm quan trọng không kém trong các mô hình dự báo số chính là các thuật toán sai phân hóa hệ PES. Đây là nguồn gốc của các sai số nội tại của mô hình mà trong suốt các thập kỷ vừa qua hàng loạt các thuật toán sai phân hữu hạn hay tích phân phổ đã được liên tục phát triển và thử nghiệm để giảm tối đa các nghiệm phi vật lý của thuật toán. Cùng với đòi hỏi độ phân giải ngày càng cao của các mô hình để có thể chi tiết hóa các quá trình vật lý thì các đòi hỏi về khả năng tính toán cũng tăng lên rất nhanh đến mức độ phân giải khả thi nhất của chúng ta hiện nay cũng chỉ vào khoảng 10 km đối với các bài toán nghiệp vụ, một độ phân giải rất thô cho các quá trình mưa hay lốc xoáy. Thêm vào đó, sự hiểu biết không đầy đủ về các quá trình vật lý khí quyển cũng ngăn cản chúng ta có được nhưng biểu diễn chính xác của các quá trình vật lý trong mô hình.

Tất cả nhưng khó khăn về mặt lý thuyết cũng như khả năng quan trắc ở trên đã và đang hạn chế nghiêm trọng đến năng lực dự báo thời tiết của con người mà hiện tại chúng ta vẫn chưa vượt qua được. Bài toán dự báo thời tiết do đó vẫn đang là một bài toán chưa có lời giải.

----------
1) PES trong bài này là viết tắt của Primitive Equation Systems. Một cách tổng quát, hệ PES trong khí tượng bao gồm phương trình chuyển động Navier-Stokes, phương trình trạng thái, phương trình bảo toàn khối lượng, và phương trình bảo toàn năng lượng. Trong một số tài liệu, hệ PES có thể bao gồm các phương trình Navier-Stokes và phương trình bảo toàn khối lượng.
2) Dự báo thời tiết trong khuôn khổ đề cập ở đây là các dự báo dựa trên hệ động lực PES. Các dự báo bằng phương pháp thống kê nói chung có thể cho các bản tin dự báo xa hơn (với độ chính xác tuy nhiên kém hơn). Kiểu dự báo thống kê này tuy nhiên lại không mang thuộc tính động lực học của khí quyển và sẽ không được đề cập ở đây.
3) Một ví dụ minh hoạ của Lorenz cho sự phụ thuộc vào điều kiện ban đầu này chính là hiệu ứng “con bướm” mà một sự vỗ cánh nhỏ của nó tại Brazil được mường tượng là có thể tạo ra các cơn lốc xoáy ở Texas, Bắc Mỹ.



Theo Tia Sáng




#315427 Ngôn ngữ phổ quát của các con số

Đã gửi bởi Ban Biên Tập on 09-05-2012 - 21:43 trong Toán học lý thú

Vâng, thành thật xin lỗi bạn, chúng tôi đã sửa lại. Cảm ơn bạn